Eureka 2010

Page 1

CONTEÚDO AOS LEITORES

2

XV OLIMPÍADA DE MATEMÁTICA DE MAIO Problemas e Resultado Brasileiro

3

XX OLIMPÍADA DE MATEMÁTICA DO CONE SUL Problemas e Resultado Brasileiro

7

L OLIMPÍADA INTERNACIONAL DE MATEMÁTICA (IMO) Problemas e Resultado Brasileiro

9

XXIV OLIMPÍADA IBEROAMERICANA DE MATEMÁTICA Problemas e Resultado Brasileiro

11

ARTIGOS PAR OU ÍMPAR? EIS A QUESTÃO Samuel Barbosa Feitosa e Einstein do Nascimento Júnior

13

GEOMETRIA DO TRIÂNGULO: FATOS E PROBLEMAS Carlos Yuzo Shine

28

SÉRIE HARMÔNICA DE NÚMEROS PRIMOS Lenimar Nunes de Andrade

45

COMO É QUE FAZ?

50

SOLUÇÕES DE PROBLEMAS PROPOSTOS

52

PROBLEMAS PROPOSTOS

59

AGENDA OLÍMPICA

61

COORDENADORES REGIONAIS

62


Sociedade Brasileira de Matemática

AOS LEITORES Por mais um ano consecutivo estamos iniciando a realização da Olimpíada Brasileira de Matemática. A Olimpíada Brasileira de Matemática – OBM tem crescido substancialmente nos últimos anos, contando, em 2009, com a adesão ao evento de mais de 3.700 escolas, sendo 2.180 da rede pública e 1.608 da rede privada de ensino, o que implicou em uma participação efetiva de cerca de 180.000 jovens estudantes e seus professores. Além disso, a iniciativa contou com a colaboração de professores universitários em 155 instituições de ensino superior: eles participaram de todas as atividades, inclusive aquelas referentes à OBM Nível Universitário em atividades de coordenação, divulgação, treinamento de alunos, aperfeiçoamento de professores e aplicação das distintas fases da Olimpíada Brasileira de Matemática. Paralelamente, o projeto apóiou a realização de Olimpíadas Regionais de Matemática, contando com a participação de 165.148 estudantes das escolas públicas e privadas em todo o Brasil nas competições estaduais. No que se refere à participação em competições internacionais, os resultados foram excelentes: Em particular, Henrique Ponde conquistou a oitava medalha de Ouro do Brasil na IMO. Além disso, nesta IMO, todos os alunos da equipe brasileria ganharam medalhas. Durante 2009 a CAPES e o CNPq lançaram o Programa de Iniciação Científica – Mestrado (PICME) para medalhistas da OBMEP e OBM, beneficiando 19 estudantes premiados na Olimpíada Brasileira de Matemática – OBM, com o objetivo de aumentar o número de matemáticos no país, e oferecer uma formação matemática mais sólida a jovens profissionais de outras áreas científicas e tecnológicas. Todos estes resultados nacionais e internacionais demonstram que, além de influenciar positivamente o ensino da Matemática nas instituições de ensino fundamental, médio e superior, conseguimos detectar jovens muito talentosos que são estimulados a seguir uma carreira científica, o que é fundamental para o crescimento da Ciência e Tecnologia no país. A Olimpíada Brasileira de Matemática é um projeto conjunto da Sociedade Brasileira de Matemática, do Instituto Nacional de Matemática Pura e Aplicada (IMPA) e conta com o apoio do Conselho Nacional de Desenvolvimento Científico e Tecnológico (CNPq) e do Instituto Nacional de Ciência e Tecnologia de Matemática (INCTMat).

Os editores

EUREKA! N°31, 2010

2


Sociedade Brasileira de Matemática

XV OLIMPÍADA DE MAIO PRIMEIRO NÍVEL PROBLEMA 1

A cada número natural de dois algarismos associamos um dígito da seguinte forma: Multiplicam-se seus algarismos. Se o resultado é um dígito, este é o dígito associado. Se o resultado é um número de dois dígitos, multiplicam-se estes dois algarismos, e se o resultado é um dígito, este é o dígito associado. Caso contrario, repetimos a operação. Por exemplo, o dígito associado a 32 é o 6 pois 3 ⋅ 2 = 6; o dígito associado a 93 é o 4 pois 9 ⋅ 3 = 27, 2 ⋅ 7 = 14, 1 ⋅ 4 = 4. Encontre todos os números de dois algarismos aos que se associa o dígito 8. PROBLEMA 2

Encontre números primos p, q, r para os quais p + q 2 + r 3 = 200 . Diga todas as possibilidades. Obs: Lembre-se que o número 1 não é primo. PROBLEMA 3

Temos 26 cartões e cada um tem escrito um número. Há dois com o número 1, dois com o número 2, dois com o 3, e assim por diante até dois com o 12 e dois com o 13. Deve-se distribuir os 26 cartões em pilhas de maneira que sejam cumpridas as duas condições a seguir: • Se dois cartões têm o mesmo número estão na mesma pilha. • Nenhuma pilha contém um cartão cujo número é igual à soma dos números de dois cartões dessa mesma pilha. Determine qual é o número mínimo de pilhas que temos que formar. Dê um exemplo com a distribuição dos cartões para esse número de pilhas e justifique por quê é impossível ter menos pilhas. PROBLEMA 4

Três circunferências são tangentes entre si, tal como mostramos na figura. A região do círculo exterior que não está coberta pelos dois círculos interiores tem área igual a 2π. Determine o comprimento do segmento PQ.

EUREKA! N°31, 2010

3

P

Q


Sociedade Brasileira de Matemática

PROBLEMA 5

Pelas linhas de um tabuleiro quadriculado formado por 55 linhas horizontais e 45 linhas verticais caminha uma formiga. Queremos pintar alguns trechos de linhas para que a formiga possa ir de qualquer cruzamento até outro cruzamento qualquer, caminhando exclusivamente pelos trechos pintados. Se a distância entre linhas consecutivas é de 10 cm, qual é a menor quantidade possível de centímetros que deverão ser pintados? SEGUNDO NÍVEL PROBLEMA 1

Inicialmente no quadro está escrito o número 1. Em cada passo, apaga-se o número do quadro e se escreve outro, que é obtido aplicando alguma das seguintes operações:

1 . 2

Operação A: Multiplicar o número escrito no quadro por

Operação B: Trocar o número escrito no quadrado pela diferença entre 1 e ele.

Por exemplo, se no quadro está escrito o número

3 podemos substituí-lo por 8

1 3 3 3 5 ou por 1 − = . ⋅ = 2 8 16 8 8 Encontre uma sequência de passos ao fim dos quais o número do quadro seja

2009 . 2 2009 PROBLEMA 2

Seja ABCD um quadrilátero convexo tal que o triângulo ABD é equilátero e o

µ = 90o . Se E é o ponto médio do lado AD, triângulo BCD é isósceles, com C µ . determine a medida do ângulo CED PROBLEMA 3

Na seguinte soma: 1 + 2 + 3 + 4 + 5 + 6, se suprimirmos os dois primeiros sinais de “+” obtemos a nova soma 123 + 4 + 5 + 6 = 138. Suprimindo três sinais de “+” podemos obter 1 + 23 + 456 = 480. Consideremos agora a soma 1 + 2 + 3 + 4 + 5 + 6 + 7 + 8 + 9 + 10 + 11 + 12 + 13, na qual serão suprimidos alguns sinais de “+”. Quais são os três menores múltiplos de 100 que podemos obter desta forma? EUREKA! N°31, 2010

4


Sociedade Brasileira de Matemática

PROBLEMA 4

Cada casa de um tabuleiro 5 × 5 é pintada de vermelho ou de azul, de tal forma que seja cumprida a seguinte condição: “Para quaisquer duas filas e duas colunas, das 4 casas que estão em suas interseções, há 4, 2 ou 0 pintadas de vermelho.” De quantas formas podemos pintar o tabuleiro? PROBLEMA 5

Um jogo de paciência se inicia com 25 cartas em fila. Algumas estão viradas para cima, e outras viradas para baixo. Em cada movimento devemos escolher uma carta que esteja virada para cima, retirá-la e virar as cartas vizinhas à que foi retirada (se houver). Ganha-se o jogo de paciência quando conseguimos, repetindo este movimento, retirar as 25 cartas da mesa. Se inicialmente há n cartas viradas para cima, encontre todos os valores de n para os quais se pode ganhar o jogo. Explique a estratégia vencedora, independentemente da localização inicial das cartas viradas para cima, e justifique por quê é impossível ganhar para os outros valores de n. Duas cartas são vizinhas quando uma está imediatamente ao lado de outra, à direita ou à esquerda. Por exemplo: a carta marcada com A tem duas cartas vizinhas e a marcada com B apenas uma. Depois de retirar uma carta fica um espaço, de C B A D modo que a marcada com C tem unicamente uma carta vizinha, e a marcada com D não tem nenhuma.

EUREKA! N°31, 2010

5


Sociedade Brasileira de Matemática

RESULTADO BRASILEIRO 2009: Nível 1 (até 13 anos) Pontos Prêmio

Nome

Cidade - Estado

Luis Fernando Veronese Trivelatto

Cascavel - PR

30

Medalha de Ouro

Lucas Carvalho Daher

Anápolis - GO

29

Medalha de Prata

Guilherme Renato Martins Unzer

São Paulo - SP

27

Medalha de Prata

Elias Brito Oliveira

Brasília - DF

26

Medalha de Bronze

Lucas Cardoso Zuccolo

São Paulo - SP

25

Medalha de Bronze

Gustavo Lima Lopes

Barra de São Fco. - ES

24

Medalha de Bronze

Rafael Rodrigues Rocha de Melo

Fortaleza - CE

24

Medalha de Bronze

Igor Albuquerque Araújo

Rio de Janeiro - RJ

23

Menção Honrosa

Liara Guinsberg

São Paulo - SP

23

Certificado

23

Certificado

Fellipe Sebastiam da Silva Paranhos Pereira Rio de Janeiro - RJ

2009: Nível 2 (até 15 anos) Nome

Cidade - Estado

João Lucas Camelo Sá

Fortaleza - CE

50

Medalha de Ouro

César Ilharco Magalhães

Barbacena - MG

42

Medalha de Prata

Bruno Silva Mucciaccia

Vitória - ES

40

Medalha de Prata

Daniel dos Santos Bossle

Porto Alegre - RS

40

Medalha de Bronze

Gustavo Haddad Francisco e Sampaio Braga S.J. dos Campos - SP

38

Medalha de Bronze

Otávio Araújo de Aguiar

Fortaleza - CE

38

Medalha de Bronze

Gabriel Militão Vinhas Lopes

Fortaleza - CE

35

Medalha de Bronze

Lara Timbó Araújo

Fortaleza - CE

33

Menção Honrosa

Artur A. Scussel

Fortaleza - CE

32

Menção Honrosa

Bruno Ferri de Moraes

São Paulo - SP

31

Menção Honrosa

EUREKA! N°31, 2010

6

Pontos Prêmio


Sociedade Brasileira de Matemática

XX OLIMPÍADA DE MATEMÁTICA DO CONE SUL Enunciados e resultado brasileiro A XX Olimpíada de Matemática do Cone Sul foi realizada na cidade de Mar del Plata, Argentina entre os dias 16 e 17 de abril de 2009. A equipe foi liderada pelos professores Pablo Rodrigo Ganassim, de São Paulo – SP e Alex Correa Abreu, de Niterói – RJ. RESULTADOS DA EQUIPE BRASILEIRA BRA1 BRA2 BRA3 BRA4

Deborah Barbosa Alves Gabriel Militão Vinhas Lopes Matheus Barros de Paula Matheus Secco Torres da Silva

Medalha de Prata Medalha de Bronze Medalha de Prata Medalha de Bronze

PROBLEMA 1

Os quatro círculos da figura determinam 10 regiões limitadas. Nessas regiões são escritos 10 números inteiros positivos distintos cuja soma é 100, um número em cada região. A soma dos números contidos em cada círculo é igual a S (a mesma para os quatro círculos). Determine o maior e o menor valor possível de S.

PROBLEMA 2

Um corchete é composto por três segmentos de comprimento 1, que formam dois ângulos retos como mostra a figura.

É dado um quadrado de lado n dividido em n2 quadradinhos de lado 1 por meio de retas paralelas aos seus lados. Corchetes são colocados sobre esse quadrado de modo que cada segmento de um corchete cubra um lado de algum quadradinho. Dois segmentos de corchete não podem ficar sobrepostos. Determine todos os valores de n para os quais é possível cobrir os lados dos n2 quadradinhos. EUREKA! N°31, 2010

7


Sociedade Brasileira de Matemática

PROBLEMA 3

Sejam A, B e C três pontos tais que B é ponto médio do segmento AC e seja P um ponto tal que ∠PBC = 60º. São construídos o triângulo equilátero PCQ tal que B e Q estão em semiplanos diferentes em relação a PC, e o triângulo equilátero APR tal que B e R estão no mesmo semiplano em relação a AP. Seja X o ponto de interseção das retas BQ e PC; seja Y o ponto de interseção das retas BR e AP. Demonstre que XY e AC são paralelos. PROBLEMA 4

Ana e Beto jogam em um tabuleiro de 11 linhas e 9 colunas. Primeiro Ana divide o tabuleiro em 33 zonas. Cada zona é formada por 3 casas adjacentes alinhadas vertical ou horizontalmente, como mostra a figura.

Depois, Beto escreve em cada casa um dos números 0, 1, 2, 3, 4, 5, de modo que a soma dos números de cada zona seja igual a 5. Beto ganha se a soma dos números escritos em cada uma das 9 colunas do tabuleiro é um número primo; caso contrário, Ana ganha. Demonstre que Beto tem uma estratégia vencedora. PROBLEMA 5

Dada uma sequência S de 1001 números reais positivos não necessariamente distintos, e dado um conjunto A de números inteiros positivos distintos, a operação permitida é: eleger um k ∈ A (k = 1001), selecionar k números de S, calcular a média dos k números (média aritmética) e substituir cada um dos k números selecionados por essa média. Se A é um conjunto tal que para cada S pode-se conseguir, mediante uma sucessão de operações permitidas, que os números sejam todos iguais, determine o menor valor possível do maior elemento de A. PROBLEMA 6

Pablo tem uma certa quantidade de retângulos cujas áreas somam 3 e cujos lados são todos menores ou iguais a 1. Demonstre que com esses retângulos é possível cobrir um quadrado de lado 1 de modo que os lados dos retângulos sejam paralelos aos lados do quadrado. Nota: Os retângulos podem estar sobrepostos e podem sair parcialmente do quadrado.

EUREKA! N°31, 2010

8


Sociedade Brasileira de Matemática

L OLIMPÍADA INTERNACIONAL DE MATEMÁTICA (IMO) Enunciados e resultado Brasileiro A L Olimpíada Internacional de Matemática (IMO) foi realizada na cidade de Bremen, Alemanha entre os dias 14 e 21 de julho de 2009. A equipe foi liderada pelos professores Carlos Yuzo Shine, de São Paulo – SP e Ralph Costa Teixeira, de Niterói – RJ. RESULTADOS DA EQUIPE BRASILEIRA BRA1 BRA2 BRA3 BRA4 BRA5 BRA6

Henrique Ponde de Oliveira Pinto Renan Henrique Finder Marcelo Tadeu de Sá Oliveira Sales Matheus Secco Torres da Silva Marco Antonio Lopes Pedroso Davi Lopes Alves de Medeiros

Medalha de Ouro Medalha de Prata Medalha de Prata Medalha de Prata Medalha de Bronze Medalha de Bronze

PRIMEIRO DIA PROBLEMA 1

Seja n um inteiro positivo e sejam a1 ,...,ak ( k ≥ 2 ) inteiros distintos do conjunto

{1,...,n}

tais que n divide ai ( ai +1 − 1) , para i = 1,...,k − 1. Demonstre que n não

divide ak ( a1 − 1) . PROBLEMA 2

Seja ABC um triângulo cujo circuncentro é O. Sejam P e Q pontos interiores dos lados CA e AB, respectivamente. Sejam K, L, e M os pontos médios dos segmentos BP, CQ e PQ, respectivamente, e Γ a circunferência que passa por K, L, e M. Suponha que a recta PQ é tangente à circunferência Γ . Demonstre que OP = OQ. PROBLEMA 3

Seja s1 ,s2 ,s3 ,... uma sucessão estritamente crescente de inteiros positivos tal que as subsucessões ss1 ,ss2 ,ss3 ,... e ss1 +1 ,ss2 +1 ,ss3 +1 ,... são ambas progressões aritméticas. Demonstre que a sucessão s1 ,s2 ,s3 ,... também é uma progressão aritmética. EUREKA! N°31, 2010

9


Sociedade Brasileira de Matemática

SEGUNDO DIA PROBLEMA 4

Seja ABC um triângulo com AB = AC. As bissectrizes dos ângulos ∠CAB e ∠ABC intersectam os lados BC e CA em D e E, respectivamente. Seja K o incentro do triângulo ADC. µ = 45°. Determine todos os possíveis valores de C µ Suponha que BEK AB. PROBLEMA 5

Determine todas as funções f do conjunto dos inteiros positivos no conjunto dos inteiros positivos tais que, para todos os inteiros positivos a e b, existe um triângulo não degenerado cujos lados medem, a, f (b) e f (b + f(a) – 1). (Um triângulo é não degenerado se os seus vértices não são colineares). PROBLEMA 6

Sejam a1 ,a2 ,...,an inteiros positivos distintos e M um conjunto de n – 1 inteiros positivos que não contém o número s = a1 + a2 + ... + an . Um gafanhoto pretende saltar ao longo da recta real. Ele começa no ponto 0 e dá n saltos para a direita de comprimentos a1 ,a2 ,...,an , em alguma ordem. Prove que essa ordem pode ser escolhida de modo que o gafanhoto nunca caia num ponto de M.

EUREKA! N°31, 2010

10


Sociedade Brasileira de Matemática

XXIV OLIMPÍADA IBEROAMERICANA DE MATEMÁTICA Enunciados e resultado Brasileiro A XXIV Olimpíada Iberoamericana de Matemática foi realizada na cidade de Santiago de Queretaro, México no período de 17 a 27 de setembro de 2009. A equipe brasileria foi liderada pelos professores Onofre Campos, de Fortaleza – CE e Luzinalva Miranda de Amorim, de Salvador – BA.

RESULTADOS DA EQUIPE BRASILEIRA BRA1 BRA2 BRA3 BRA4

Renan Henrique Finder Matheus Secco Torres da Silva Marco Antonio Lopes Pedroso Marcelo Tadeu de Sá Oliveira Sales

Medalha de Ouro Medalha de Ouro Medalha de Prata Medalha de Prata

PRIMEIRO DIA PROBLEMA 1

Seja n um natural maior que 2. Suponhamos que n ilhas estejam localizadas ao redor de um círculo e que entre cada duas ilhas vizinhas haja duas pontes, como na figura:

x2

x1

x3

xn

xj

xn – 1

Partindo da ilha x1 , de quantas maneiras se podem percorrer as 2n pontes passando por cada ponte exatamente uma vez? EUREKA! N°31, 2010

11


Sociedade Brasileira de Matemática

PROBLEMA 2

Para cada inteiro positivo n definimos an = n + m, onde m é o maior inteiro tal que 22 ≤ n 2n . Determinar quais inteiros positivos não aparecem na sequência an . m

PROBLEMA 3

Sejam C1 e C2 duas circunferências de centros O1 e O2 , com o mesmo raio, que se intersectam em A e B. Seja P um ponto sobre o arco AB de C2 que está dentro de C1 . A reta AP intersecta C1 em C, a reta CB intersecta C2 em D e a bissetriz de ∠CAD intersecta C1 em E e C2 em L. Seja F o simétrico do ponto D em relação ao ponto médio de PE. Demonstrar que existe um ponto X que satisfaz ∠XFL = ∠XDC = 30° e CX = O1O2 . SEGUNDO DIA PROBLEMA 4

Seja ABC um triângulo com AB ≠ AC. Sejam I o incentro de ABC e P o outro ponto de interseção da bissetriz externa do ângulo A com o circuncírculo de ABC. A reta PI intersecta o circuncírculo de ABC no ponto J. Demonstrar que os circuncírculos dos triângulos JIB e JIC são tangentes às retas IC e IB, respectivamente. PROBLEMA 5

A sequência an está definida por 1 , para todo inteiro k ≥ 1. a2 k Demonstrar que todo número racional positivo aparece exatamente uma vez nesta sequência. a1 = 1, a2 k = 1 + ak e a2 k +1 =

PROBLEMA 6

Ao redor de uma circunferência marcam-se 6000 pontos, cada um dos quais se pinta com uma de 10 cores dadas, de modo que entre quaisquer 100 pontos consecutivos sempre figuram as 10 cores. Achar o menor inteiro k com a seguinte propriedade: para toda coloração deste tipo existem k pontos consecutivos nos quais se encontram as 10 cores.

EUREKA! N°31, 2010

12


Sociedade Brasileira de Matemática

PAR OU ÍMPAR? EIS A QUESTÃO Einstien do Nascimento Jr e Samuel Barbosa Feitosa Paridade Quando duas pessoas estão indecisas sobre uma escolha, muitas vezes elas utilizam uma brincadeira chamada Par ou ímpar para se decidirem. Por trás desse simples critério, podem se resolver problemas que parecem ser bastante complicados. Dizemos que um númreo tem paridade par se ele for par, e paridade ímpar, se ele for ímpar. Observar a paridade de um número é algo bem simples mas com aplicações fantásticas em problemas de olimpíadas. Vejamos um exemplo: Paridade como invariante Vamos começar com um problema bastante famoso que já foi utilizado até em entrevistas para grandes empresas de computação. Problema 1. 100 pessoas são postas em uma fila e cada uma delas recebe um chapéu, que pode ser preto ou branco. Cada pessoa só consegue ver os chapéus das pessoas que estão a sua frente. É pedido que cada uma delas tente adivinhar a cor do seu chapéu. Qual o máximo número de acertos que se pode garantir, dado que as pessoas podem combinar uma estratégia antes de recebê-los. Solução: Facilmente consegue-se 50 acertos. Podemos dividir as pessoas em pares: (100,99), (98, 97),...(2, 1) e assim o maior número de cada par falar a cor da pessoa da frente. Que apenas precisa repeti-lo, para garantir 1 acerto por par. De uma forma um pouco mais elaborada, se garante 66 acertos. Separando em trios: (100, 99, 98),...(4, 3, 2). O maior número de cada trio pode falar BRANCO caso os dois da sua frente tenham a mesma cor e PRETO, caso as cores sejam distintas. Assim, após o maior número falar, o número do meio pode acertar sua cor e em seguida, o primeiro do trio pode acertar a dele. Curiosamente esse número pode chegar a 99 acertos utilizando esse poderoso argumento que é a paridade. Notemos que ninguém sabe a cor do último da fila. Então não importa a estratégia de ordem das pessoas, nenhuma informação pode ser obtida para esse chapéu. O que não ocorre com os 99 chapéus restantes. Note ainda que a diferença de conhecimentos entre a pessoa e a pessoa que encontra atrás dela é apenas o seu chapéu. Então, basta seguir a estratégia: As cores serão faladas das pessoas de trás para as da frente. E a última pessoa vai falar BRANCO caso a quantidade de chapéus brancos a sua frente seja par e PRETO, caso contrário. Como a 99ª. pessoa sabe a paridade da quantidade de chapéus brancos estritamente à sua frente, e a paridade da EUREKA! N°31, 2010

13


Sociedade Brasileira de Matemática

quantidade de chapéus brancos à sua frente, incluindo ela mesma, que foi informada pela 100ª. pessoa, ela acertará o seu chapéu. A 98ª., computando ambas as informações pode acertar o dela, e assim sucessivamente. Problema 2. Em cada casa de um tabuleiro de 5 x 5 está escrito 1 ou –1. Em cada passo troca-se o número de cada uma das 25 casas pelo resultado da multiplicação dos números de todas as suas casas vizinhas. Inicialmente se tem o tabuleiro da figura. Mostre como fica o tabuleiro ao final de 2004 passos. Observação: Duas casas são vizinhas se tiverem um lado em comum. 1 1 1 1 1

1 1 1 1 1

–1 1 1 1 1

1 1 1 1 1

1 1 1 1 1

Dica: Muitas vezes, quando não se tem ideia de como será a solução de uma questão, pode-se obter várias pistas fazendo alguns casos iniciais do enunciado, esperando observar algum padrão. Meus números da sorte são 5 e 9. Ao achar um padrão repetitivo, basta analisar em que caso cairá o número 2004. Problema 3. Em cada um dos 10 degraus de uma escada existe uma rã. Cada rã pode, de um pulo, colocar-se em outro degrau, mas quando uma rã faz isso, ao mesmo tempo, uma outra rã pulará a mesma quantidade de degraus em sentido contrário: uma sobe e outra desce. Conseguirão as rãs colocar-se todas juntas num mesmo degrau? Solução: Uma maneira muito utilizada para atacar problemas onde é dada uma condição inicial e um conjunto de operações para manipulá-la é tentar procurar o que não muda, independentemente dos movimentos que utilizamos. Note que se uma rã vai de um degrau par para um ímpar (muda de paridade), a outra rã que se movimenta com ela também pulará um número ímpar de degraus, mudando também a paridade. Caso a primeira não mude, a sua parceira de movimento também permanecerá num degrau de mesma paridade. UM INVARIANTE: Paridade da quantidade de rãs em degraus de número par (comprove testando os movimentos possíveis). Como na posição inicial há 5 rãs nos degraus de posição par e na posição final há ou dez ou zero rãs nos degraus de posição par, a posição final NÃO pode ser obtida da posição inicial apenas fazendo essas operações permitidas. EUREKA! N°31, 2010

14


Sociedade Brasileira de Matemática

Essa estratégia de invariantes é utilizada principalmente para provar a impossibilidade de ocorrer algum evento. Definiremos uma peça príncipe (que não existe no jogo de xadrez) como uma que só pode andar na horizontal e vertical, uma casa por vez. Um jeito comum de fazer notações em um tabuleiro de xadrez é nomear as colunas da esquerda para a direita de a a h e as linhas de baixo para cima de 1 a 8 tomando o referencial da pessoa que joga com as casas brancas. Problema 4. Sobre um tabuleiro de xadrez, um príncipe começa do quadrado a1 e retorna após fazer alguns movimentos. Mostre que o príncipe fez um número par de movimentos. Solução: veja que em cada movimento, o príncipe muda para uma casa de cor oposta. Como a casa a1 é preta, após um número ímpar de movimentos o príncipe estará numa casa da cor branca. Para ele ter retornado até a casa preta do início, ele deverá ter feito um número par de movimentos. Problema 5. Pode um príncipe começar do quadrado a1 de um tabuleiro de xadrez, ir até o quadrado h8, visitando cada um dos quadrados restantes exatamente uma vez? Solução: A resposta é não. Em cada movimento, o príncipe pula para um quadrado da cor oposta. Como o príncipe tem que fazer 63 movimentos, o último movimento irá deixá-lo em uma casa da cor oposta a cor de a1. Entretanto, a1 e h8 têm a mesma cor. Isto é um absurdo. O último problema nos conduz a um tipo muito importante de demonstração: prova por absurdo. Suponha que lhe perguntaram se é possível somar cinco números ímpares e obter o número 100. Após algumas tentativas você começa a desconfiar que isto não é possúvel. Mas como provar que não é possível? Se realmente fosse possível somar 5 números ímpares e obter 100 o que aconteceria? Como a soma de cinco números ímpares é sempre ímpar obteríamos que 100 é um número ímpar. Mas 100 não é ímpar! Logo não é possível existirem tais 5 números. Para provar que algo não é possível, basta supormos que é possível e chegarmos a um absurdo. Problema 6. Uma linha poligonal fechada é composta por 11 segmentos. Pode uma reta (não contendo um vértice da linha poligonal) intersectar cada um desses segmentos? EUREKA! N°31, 2010

15


Sociedade Brasileira de Matemática

Problema 7. Três bolas de gude, A, B, e C estão no chão. Um movimento permitido é passar uma bola entre as outras duas. É possível, após 25 movimentos, que todas as bolas estejam nas suas posições originais? Dica: Que horas são? (Sentidos horário e anti-horário...) Problema 8. Kátia e seus amigos estão em um círculo. Sabemos qua ambos os vizinhos de cada criança são do mesmo sexo. Determine o número de garotas sabendo que existem 5 garotos no círculo. Dica: Comece a analisar por um vizinho da Kátia. Problema 9. (Rússia 1970) O rei Luis estava desconfiado de alguns de seus cortesãos. Ele fez uma lista completa de cada um dos seus cortesãos e disse a cada um deles para espionar um outro cortesão. O primeiro da lista foi espionar o cortesão que estava espionando o segundo da lista, o segundo da lista foi espionar o cortesão que estava espionando o terceiro da lista, e assim sucessivamente; o penúltimo foi espionar o cortesão que estava espionando o último e o último foi espionar o cortesão que estava espionando o primeiro. Prove que o rei Luis tinha um número ímpar de cortesãos. Solução. Seja n o número de cortesão da lista e suponha que n é par. Coloque-os sentados ao redor de uma mesa circular de modo que cada um esteja espionando o seu vizinho da direita.

Y

1

X 2

n 2

O cortesão 1 espia o cortesão X que espia o cortesão 2, o cortesão 2 espia o n cortesão Z que espia o cortesão 3, e assim sucessivamente até que o cortesão 2 n espia o cortesão Y que espia o cortesão + 1. Como os números 1, 2, 3,...,n devem 2 EUREKA! N°31, 2010

16


Sociedade Brasileira de Matemática

se alternar sobre o círculo, concluímos que o cortesão

n + 1 é igual ao cortesão 1, 2

ou seja, n = 0. Esse absurdo mostra que n é ímpar. Problema 10. Um cubo 1 × 1 × 1 está posicionado em um plano quadriculado de modo que uma de suas faces coincide com um dos quadradinhos do plano. Em cada movimento podemos “tombar” o cubo por uma de suas arestas, fazendo coincidir uma face, que tinha essa aresta, com um dos quadradinhos do plano. È possível fazer o cubo voltar a sua posição inicial após 2005 movimentos? Dica: Alguém aí joga xadrez? Paridade e Contagens Nesta seção, abordaremos duas ideias muito simples: 1. Se contamos os elementos de um conjunto de duas maneiras diferentes, os valores obtidos devem ter a mesma paridade (Porque são iguais!) 2. Se os elementos de um conjunto podem ser pareados então o conjunto tem uma quantidade par de elementos. Problema 11. Em Brasilândia existem apenas 9 casas muito distantes entre si. È possível que cada casa esteja ligada a exatamente 7 outras casas através de estradas? Solução: Não é possível. Some a quantidade de estradas que saem de cada casa. Bem, facilmente obtemos 9 × 7 estradas. Como cada estrada liga duas cidades, a contagem que fizemos contou cada estrada duas vezes. Logo o número obtido teria que ser par. Você deve ter ficado com uma pulga atrás da orelha. Será que cada casa ligada a exatamente 7 outras foi realmente crucial? É possível revolvermos o problema anterior com um eneunciado mais geral: Problema 12. Prove que numa festa com n pessoas, o número de pessoas que conhecem um número ímpar de outras pessoas na festa á par. Solução: Numere as pessoas de 1 até n e denote por d i o número de amigos da pessoa i. Imagine que existe um fio entre duas pessoas que se conhecem. Se E denota a quantidade de fios, temos d1 + d2 + ... + d n = 2 E, EUREKA! N°31, 2010

17


Sociedade Brasileira de Matemática

pois cada fio é contado duas vezes, um para cada ponta. Como o lado direito é par, no lado esquerdo devemos ter uma quantidade par de números ímpares. Problema 13. (Olimpíada de Maio 2000) O conjunto {1, 2, 3, 4} pode ser dividido em dois subconjuntos A = {1,4} e B = {3, 2} sem elementos comuns e tais que a soma dos elementos de A seja igual à soma dos elementos de B. Essa divisão é impossível para o conjunto {1, 2,3, 4,5} e também para o conjunto {1, 2, 3, 4, 5, 6} . Determine todos os valores de n para os quais o conjunto dos primeiros n números naturais pode ser dividido em dois subconjuntos sem elementos comuns tais que a soma dos elementos de cada subconjunto seja a mesma. Solução. Como a soma dos elementos de A deve ser igual à soma dos elementos de B, a soma dos números do conjunto {1, 2,3,...,n} deve ser o dobro da soma dos elementos de A, ou seja, deve ser um número par. Você já deve saber que 1 + 2 + 3 + ... + n =

n ( n + 1)

. 2 Você não sabia disso? Não fique aí parado! Tente descobrir porque isso é verdade! n ( n + 1) Veja que é par se n ( n + 1) é múltiplo de 4. Como estamos interessados no 2 resto na divisão por 4 de algum número, talvez seja interessante procurar quais os possíveis restos de n na divisão por 4. Podemos escrever n na forma n = 4q + r onde r = 0,1, 2 ou 3. Mãos à obra! n ( n + 1) = 2q ( 4q + 1) é par. 1. Se n = 4q então 2 n ( n + 1) 2. Se n = 4q + 1 então = ( 2q + 1)( 4q + 1) é ímpar. 2 n ( n + 1) 3. Se n = 4q + 2 então = ( 2q + 1)( 4q + 1) é ímpar. 2 n ( n + 1) 4. Se n = 4q + 3 então = ( 2q + 2 )( 4q + 3) é par. 2 Podemos concluir que n deve ser da forma 4q ou 4q + 3. Acabou? Não! Precisamos construir EXEMPLOS para cada uma dessas possibilidades mostrando que realmente esses valores satisfazem as condições do problema. Para n = 4q, considere os conjuntos

EUREKA! N°31, 2010

18


Sociedade Brasileira de Matemática

A = {(1, 4 ) ,( 5,8 ) ,( 9 ,12 ) ,...,( 4q − 3, 4q )} .

B = {( 2,3) ,( 6,7 ) ,(10 ,11) ,...,( 4q − 2 ,4q − 1)} .

Para n = 4q + 3, considere os conjuntos A = {( 4,7 ) ,( 8,11) ,(12,15 ) ,...,( 4q, 4q + 3)} ∪ {(1, 2 )} .

B = {( 5,6 ) ,( 9 ,10 ) ,(13,14 ) ,...,( 4q + 1, 4q + 2 )} ∪ {( 3)} .

Note que os conjuntos foram divididos em parêntesis. Cada parêntese de A possui correspondente em B com a mesma soma, facilitando a construção de um exemplo generalizado. Problema 14. Podemos desenhar uma linha poligonal fechada feita por 9 segmentos de reta, cada um deles intersectando exatamente outro segmento? Solução. Se tal construção é possível, então todos os segmentos podem ser agrupados em pares de segmentos intersectantes. Mas o número de segmentos é ímpar! Absurdo! Os próximos dois problemas tratam de dominós. Um dominó consiste de um tabuleiro 1 x 2 com pontos em cada casinha. A quantidade de pontos varia de 0 até 6. Então, o número total de dominós distintos é 28. Problema 15. Todos os dominós são arranjados em uma cadeia de duas pontas (a quantidade de pontos na extremidade de dois dominós consecutivos é a mesma). Se em uma ponta existe o número 5, qual é o número de outra ponta? Problema 16. Em um conjunto de dominós, descartamos todos aqueles que possuem pelo menos uma casinha vazia. É possível arranjarmos todos os restantes em uma cadeia? Problema 17. (Eslovênia 1992) Prove que para quaisquer inteiros positivos a1 ,a2 ,...,an o número: a1 − a2 + a2 − a3 + ... + an − a1

é par. Observação: x − y é chamado de valor absoluto da diferença entre x e y e denota o máximo entre x – y e y – x. Na reta real, ele representa a distância entre os números x e y.

EUREKA! N°31, 2010

19


Sociedade Brasileira de Matemática

Solução: Perceba que x − y = ± x ± y para alguma escolha de sinais. Então a soma total é ± a1 ± a2 ± a2 ± a3 ± ... ± an ± a1 . Como cada número ai aparece duas vezes, basta mostrarmos que cada uma das expressões ± a1 ± ai é par para qualquer escolha de sinais. Vejamos os casos: 1. 2. 3. 4.

± ai ± ai ± ai ± ai

± ai ± ai ± ai ± ai

= + ai + ai = − ai + ai = + ai − ai = − ai − ai

= 2ai é par. = 0 é par. = 0 é par. = −2ai é par.

1.3 Miscelânia Problema 18. Podemos trocar uma nota de 25 reais usando dez notas que podem assumir os valores 1, 3, 5? Solução. Não. Como a soma de um número par de números ímpares é par, a soma dos valores dessas 10 notas só pode ser um número par. Mas 25 é ímpar. Problema 19. Peter comprou um caderno com 96 folhas, e numerou com os números de 1 até 192. Victor rasgou 25 folhas consecutivas do caderno, e adicionou os 50 números. Victor pode ter obtido o número 1990 como resultado da soma? 1 1 1 1 1 1 + + + + + = 1 não admite soluções a b c d e f com todos os números sendo ímpares. Dica: Faça o produto dos denominadores.

Problema 20. Prove que a igualdade

Problema 21. O produto de 21 inteiros é igual a 1. Mostre que sua soma não pode ser zero. Dica: compare as quantidades de números positivos e negativos. Problema 22. Três gafanhotos estão brincando ao longo da uma linha. Na sua vez, cada gafanhoto pode pular sobre um outro gafanhoto, mas não sobre os outros dois. Eles podem retornar para suas posições iniciais após 1991 movimentos?

EUREKA! N°31, 2010

20


Sociedade Brasileira de Matemática

Solução. Sejam A, B, C os três gafanhotos. Estaremos interessados apenas na ordem em que os gafanhotos se dispõem ao longo da reta, digamos que inicialmente eles estão na ordem (A, B, C). Podemos fazer os seguintes movimentos: 1

2

3

4

( A,B,C ) → ( B,A,C ) → ( B,C,A) → ( C,B,A) → ... Em cada passo, disponha as letras A, B e C em um círculo (como mostra a figura) e leia a palavra ABC. Percebeu alguma coisa? Antes de efetuarmos nosso primeiro movimetno, a leitura estava no sentido “horário” e logo em seguida passou para o sentido “anti-horário”. Como cada movimento alternar os sentidos, após 1991 movimentos estaremos em um sentido diferente do original. Logo, não é possível retornarmos para a posição original.

A

C

B

B

C

A

Observação: Compare com o problema 7. Problema 23. Os números de 1 até 10 são escritos em uma linha. Podemos colocar os sinais + e – entre eles de modo que o resultado da expressão resultante seja 0? Solução: Não é possível. Perceba que quando escolhemos um número para trocarmos de sinal, por exemplo, de + para –, a soma total varia o dobro do número escolhido, ou seja, a paridade da soma não muda. Basta ver agora que 1 + 2 + ... + 10 = 55 não tem a mesma paridade que 0. Um INVARIANTE é a paridade da soma. Problema 24. Um gafanhoto pula ao longo de uma linha. No seu primeiro pulo, ele anda 1cm, no segundo 2cm, e assim sucessivamente. Ele pode pular para a esquerda ou para a direita. Mostre que após 1985 pulos, o gafanhoto não pode retornar ao ponto em que começou. Dica: Perceba que você pode associar aos pulos do gafanhoto um número com EUREKA! N°31, 2010

21


Sociedade Brasileira de Matemática

sinal (+ se o pulo é para a esquerda e – se é para a direita). Agora use o problema anterior. Problema 25. Os números 1, 2,...,1984, 1985 são escritos em um tabuleiro. A operação permitida é apagar dois números e colocar sua diferença positiva. Após algumas operações, resta apenas um único número no tabuleiro. Pode este número ser 0? Problema 26. Pode um tabuleiro 8 × 8 ser coberto com dominós 1 × 2 de modo que somente os quadrados a1 e h8 não sejam cobertos? Solução. Não é possível. Pinte o tabuleiro de preto e branco da maneira usual. Cada dominó cobre exatamente um quadrado preto e outro branco (Invariante), portanto, a quantidade de quadrados pretos cobertos é igual à quantidade de quadrados brancos cobertos. Como a1 e h8 têm a mesma cor, sobrariam 30 quadrados de uma cor e 32 de outra para serem cobertos. Absurdo! Problema 27. 45 pontos são escolhidos sobre a reta AB, todos fora do segmento de reta AB. Prove que a soma das distâncias desses pontos ao ponto A não pode ser igual à soma das distâncias ao ponto B. Solução. Sejam A e B dispostos, sem perda de generalidade como na figura abaixo. Tomemos um ponto X. A

B

X

X pode estar à direita de B ou à esquerda de A. Ou ocorre: AX + AB = BX ou BX + AB = AX . Assim, se estivéssemos somando em x as distâncias dos 45 pontos para A e em y para B, estaríamos na verdade, só somando uma diferença de AB em x ou em y. Como 45 é ímpar, não podemos “distribuir” uma igual quantidade de AB´ s para o grupo de A e o de B. Assim, segue que não é possível. Problema 28. Um número de 17 dígitos é somado com o seu reverso (um número com os mesmo dígitos mas escritos na ordem inversa). Mostre que sua soma contém pelo menos um dígito par. Problema 29. Existem 100 soldados em um quartel. Toda noite, três deles ficam de guarda. Após um certo período de tempo, é possível que cada soldado tenha ficado de guarda exatamente uma vez com cada outro soldado? EUREKA! N°31, 2010

22


Sociedade Brasileira de Matemática

Solução: Suponha, por absurdo, que seja possível. Tomemos o Soldado Ryan, ele possui 99 companheiros. Suponha que ele em particular tenha conseguido ficar exatamente uma vez de pernoite com cada um dos outros. A cada dia, Ryan formava 2 duplas diferentes, que não poderiam se repetir nos dias posteriores. Caso Ryan tivesse pernoitado x vezes, a quantidade de duplas que ele teria formado seria 2x, que por hipótese, deve ser igual a 99. Chegando à conclusão que 99 é par. Absurdo! Problema 30. 25 garotos e 25 garotas estão sentados ao redor de uma mesa. Prove que é sempre possível encontrar uma pessoa tal que ambos os seus vizinhos são garotas. Solução: Suponha, por absurdo, que não necessariamente haja uma pessoa que possua duas garotas como vizinhas. Denotemos h para garoto e m para garota. Cada pessoa ou possui como vizinho 2h ou h+m. Somando todas as 50 possibilidades, devemos estar contando cada pessoa duas vezes (já que essa é vizinho de duas pessoas). Assim: x ( 2h ) + y ( h + m ) = 50h + 50m onde x é o número de pessoas que têm 2 garotos como vizinhos e y é o número de pessoas que têm um garoto e uma garota. Notemos ainda que x + y = 50. Obtemos xh = ( 50 − y ) m assim xh = xm. Mas x garotos só serão iguais a x garotas, se x for nulo. Assim, todas as pessoas têm um garoto e uma garota como vizinhos. Pintemos as 50 posições do círculo apenas de branco e preto. E analisemos apenas as pretas. Todas as pretas terão que ter vizinhos sendo um garoto e uma garota. Logo, as casas brancas serão alternadas: garoto, garota, garoto... Absurdo. Pois com 25 casas brancas, na última e na primeira brancas haverá 2 garotos. Absurdo! Segue o resultado. Problema 31. (Ucrânia 1997) Um tabuleiro é colorido de branco e preto da maneira usual, e cada casa contém um inteiro. Sabemos que a soma dos números em cada coluna e a soma dos números em cada linha é par. Mostre que a soma dos números nas casas pretas é par. Solução. Suponha sem perda de generalidade que o quadrado do canto esquerdo superior é preto. A partir desse quadrado, numere as colunas da esquerda para a direita e as linhas de cima para baixo. Some os números das colunas em posições ímpares e os números das linhas em posições pares. Perceba que cada quadrado preto do tabuleiro é contado apenas uma vez nessa soma enquanto que os quadrados brancos das linhas e colunas mencionadas são contados duas vezes. Logo, esse soma tem a mesma paridade que a soma de todos os números escritos EUREKA! N°31, 2010

23


Sociedade Brasileira de Matemática

nos quadrados pretos. Como a soma de quaisquer linhas e colunas é par, a soma dos números nos quadrados pretos é par.

Problema 32. Considere um tabuleiro 1998 × 2002 pintado alternadamente de preto e branco da maneira usual. Em cada casa do tabuleiro, escrevemos 0 ou 1, de modo que a quantidade de 1´s em cada linha e em cada coluna do tabuleiro é ímpar. Prove que a quantidade de 1´s escritos nas casa brancas é par. Dica: Tente imitar a solução anterior. Problema 33. (Austrália 2007) Em cada casa de um tabuleiro 2007 × 2007 escrevemos um número inteiro ímpar. Sejam Z i a soma dos números na i-ésima linha e S j a soma dos números na j-ésima coluna, para 1 ≤ i, j ≤ 2007. Além disso, sejam A = Z i ⋅ Z 2 ...Z 2007 e B = S1 ⋅ S 2 ...S 2007 . Mostre que A + B não pode ser igual a zero. Problema 34. (China 1986) É possível arranjar os números 1, 1, 2, 2, 3, 3,...,1986, 1986 em fila de modo que entre quaisquer dois i´s hajam (i – 1) números? Solução: Vamos tentar fazer alguns casos pequenos. É fácil ver que não conseguimos fazer o que o enunciado pede com os números 1, 1, 2, 2 mas com os números 1, 1, 2, 2, 3, 3, 4, 4 temos um exemplo: 1º. 2º. 3º. 4º. 5º. 6º. 7º. 8º. a3 a4 a2 b3 b2 b4 a1 b1 3 4 2 3 2 4 1 1 Contados da squerda para a direita, denotemos por ai e bi as posições do primeiro e segundo número i, respectivamente. No nosso exemplo, a2 = 3 e b2 = 5. Como existem i – 1 números entre dois números i´s, devemos ter bi − ai = i. Se é possível escrever os números 1, 1, 2, 2, ..., n, n em linha como no enunciado, EUREKA! N°31, 2010

24


Sociedade Brasileira de Matemática

obtemos:

( a1 + a2 + ...an ) + ( b1 + b2 + ...bn ) = 1 + 2 + ... + 2n = n ( 2n + 1) n ( n + 1) . ( b1 − a1 ) + ( b2 − a2 ) + ...( bn − an ) = 1 + 2 + ...+ n = 2

Somando as duas linhas, 2 ( b1 + b2 + ...bn ) =

n ( 5n + 3 )

Como o lado esquerdo é sempre par, a fração

2 n ( 5n + 3) 2

deve ser um inteiro par.

Isso já restringe os possíveis valores de n. Para n = 1986, n ( 5n + 3 ) = 9863469 2 é ímpar e conseqüentemente não é possível dispormos esses números em linha. Uma pergunta natural que você deve tentar responder é: para quais n tal distribuição é possível? Problema 35. É possível arranjar os números de 1 até 9 em uma sequência, de modo que exista uma quantidade ímpar de números entre 1 e 2, entre 2 e 3,..., e entre 8 e 9? Problema 36. (Rússia 1984) O número de todos os inteiros positivos de 64 dígitos sem zeros em sua representação e que são divisíveis por 101 é par ou ímpar? Solução: Precisamos bolar alguma maneira de agrupar os números em pares. Seja A = 11 ...3 110 repetições do número 1. 12 64 vezes

Como 1111 é múltiplo de 101 é fácil ver que A é múltiplo de 101. Para todo número de 64 dígitos a = a1a2 ...a63 a64 , sem zeros em sua representação decimal, considere o seu conjugado b = b1b2 ...b63b64 = (10 − a1 )(10 − a2 ) ...(10 − a64 ). Nenhum dígito de a é igual a zero, portanto, cada número 10 − ai pertence ao conjunto

{1,2,..,9} . Da equação a + b = A obtemos que a é divisível por 101 se e somente se

b é divisível por 101 (lembre-se que A é múltiplo de 101). Como o único número que é igual ao seu conjugado é o número 55 ...55 (que é múltiplo de 101) e os 123 64 vezes

demais números que satisfazem o enunciado podem ser pareados, concluímos que a EUREKA! N°31, 2010

25


Sociedade Brasileira de Matemática

quantidade procurada é ímpar. Problema 37. (Putnam 1997) Seja Bn a quantidade de n – uplas ordenadas de

inteiros positivos ( a1 ,a2 ,...,an ) tais que

1 1 1 + + ... + = 1 a1 a2 an B10 é par ou ímpar?

Solução: Uma ideia natural é tentar agrupar as soluções em pares. Qualquer solução com a1 ≠ a2 pode ser pareada com a outra solução obtida pela troca de posição entre a1 e a2 . Logo, B10 tem a mesma paridade que o número de soluções com a1 = a2 . Das soluções com a1 = a2 , podemos parear aquelas que tem a3 ≠ a4 da mesma maneira. Repetindo esse argumento com ( a5 ,a6 ) ,( a7 ,a8 ) e ( a9 ,a10 ) ,

concluímos que a paridade de B10 é a mesma do número de soluções com a5 = a6 ,a7 = a8 e a9 = a10 , ou seja, das soluções de: 2 2 2 2 2 + + + + = 1. a1 a3 a5 a7 a9 Como anteriormente, podemos nos restrigir à quantidade de soluções com a1 = a3 e a5 = a7 , que é igual ao número de soluções da equação: 4 4 2 + + = 1. a1 a5 a9 Mais uma vez, podemos nos restringir à quantidade de soluções com a1 = a5 , que é igual ao número de soluções da equação: 8 2 + = 1. a1 a9 Agora ficou fácil! Basta contar explicitamente o número de soluções da equação anterior. Como fazer isso? Bem, ela pode ser fatorada como: ( a1 − 8 )( a9 − 2 ) = 16 que admite 5 soluções correspondendo às fatorações de 16 como 2i × 24−i para i = 0,1,2 ,3,4. Então B10 é ímpar. Problema 38: Prove que numa festa com 2n pessoas existem duas com um número par de amigos em comum.

EUREKA! N°31, 2010

26


Sociedade Brasileira de Matemática

Solução: Suponha que quaisquer duas pessoas tenham um número ímpar de amigos em comum e seja A um dos participantes da festa. Seja M = { F1 ,F2 ,...,Fk } o conjunto dos amigos de A. Considere uma nova festa restrita apenas ao conjunto M. Como cada Fi tem um número ímpar de amigos em comum com A, na nova festa, cada Fi possui um número ímpar de amigos. Pelo problema 12, k deve ser par. O mesmo argumento vale para qualquer pessoa na festa e conseqüentemente todos têm um número par de amigos. Peça para cada um dos amigos de A fazerem uma lista de seus amigos diferentes de A. A soma da quantidade de nomes listados é par, pois é uma soma de uma quantidade par (igual a k) de números ímpares (cada Fi possui um número ímpar de amigos diferentes de A). Agora comparemos o número de aparições de cada uma das 2n − 1 pessoas diferentes de A nessas listas. Se cada uma delas aparecer em um número ímpar de listas, a soma total de todos os nomes em todas as listas seria ímpar. (Lembre-se que a soma de uma quantidade ímpar de números ímpares é ímpar!). Mas isso é uma contradição. Logo, existe uma pessoa diferente de A que aparece em um número par de listas, e portanto tem um número par de amigos em comum com A. Problema 39. Alex desenhou uma coleção de K retas no plano em posição geral (quaisquer duas retas se intersectam em um ponto e quaisquer três definem um triângulo não degenerado). Para quais valores de K é sempre possível (não importa como as retas são desenhadas) colocar um elemento do conjunto {1, 2 ,...,K − 1} em cada ponto de interseção das retas de modo que em toda reta não existam números iguais. Problema 40. (Rússia) Em cada planeta de um sistema solar existe um astrônomo observando o planeta mais próximo. As distâncias entre os planetas são distintas duas a duas. Demonstre que se a quantidade de planetas é ímpar, então existe pelo menos um planeta que não é observado. Dica: Procure as cadeias de planetas que um olha para o outro que olha para o outro com mais de 2 planetas. REFERÊNCIAS [1] D. Fomin, S. Genkin e I. Itenberg, Mathematical Circles, MAS (1996). [2] C. Augusto, S. Feitosa, B. Holanda e Y. Lima, treinamento Cone Sul 2007, Fortaleza, Realce (2007). [3] P. J. Taylor, Tournament of the Towns 1980 to 1984, Australian Mathematical Trust (1993). [4] D. Fomin e A. Kirichenko, Leningrand Mathematical Olympiadas 1987-1991, MathPro Press (1994). [5] E. Wagner, Paridade, Eureka! No. 2, pp. 32-38, (1998).

EUREKA! N°31, 2010

27


Sociedade Brasileira de Matemática

GEOMETRIA DO TRIÂNGULO: FATOS E PROBLEMAS Carlos Yuzo Shine 1. O Teorema de Miquel Começamos com o teorema em si, que é um dos vários pequenos milagres dos chamados quadriláteros completos (veja um pouco mais desses “milagres” nos exercícios!), que são os quadriláteros conhecidos unidos com as retas que contêm os lados. Isto é, um quadrilátero completo é a união de quatro retas em vez de quatro segmentos. Teorema de Miquel. Sejam a, b, c, d quatro retas coplanares, de modo que não há duas paralelas nem três concorrentes. Os circuncírculos dos quatro triângulos determinados pelas quatro retas passam por um mesmo ponto, denominado ponto de Miquel das quatro retas.

Demonstração: Seja M a intersecção dos circuncírculos de CEF e BDF na figura acima. Então ∠MEA = ∠MEC = 180° − ∠MFC = ∠BFM = ∠BDM = 180° − ∠ADM , de modo que ∠MEA + ∠ADM = 180° e, portanto, MDAE é inscritível. Isso quer dizer que M pertence ao circuncírculo de ADE. Analogamente, prova-se que M pertence ao EUREKA! N°31, 2010

28


Sociedade Brasileira de Matemática

circuncírculo de ABC. Vamos resolver, a título de exemplo, o problema 6 da olimpíada norteamericana de 2006. Exemplo 1.1. (USAMO 2006, Problema 6) Seja ABCD um quadrilátero e E e F os pontos sobre os lados AD e BC, respectivamente, tal que AE / ED = BF / FC. A semirreta FE corta as semirretas BA e CD em S e T, respectivamente. Prove que os circuncírculos dos triângulos SAE, SBF, TCF e TDE passam por um mesmo ponto. Resolução: Ao fazer a figura, você provavelmente vai notar uma certa semelhança com a figura anterior.

Queremos provar que os pontos de Miquel de ADTS e BCTS coincidem! Isso não é difícil, na verdade: seja M a intersecção dos circuncírculos de SAE e SBF. Mostraremos que os circuncírculos de TED e TFC também passam por M. Um arrastão e uma semelhança dão conta do recado: primeiro, note que ∠AME = ∠ASE = ∠BSF = ∠BMF e ∠MEA = ∠MSA = ∠MSB = ∠MFB. Então os triângulos AME e BMF são semelhantes, e da igualdade AE / ED = BF / FC os triângulos MAD e MBC são semelhantes também. Uma rápida verificação mostra que MAB e MDC também são semelhantes: de fato (pois como MAD e MBC são EUREKA! N°31, 2010

29


Sociedade Brasileira de Matemática

AM DM = (novamente da semelhança). BM CM Você pode imaginar que o triângulo MAD “gira” em torno de M e, após um “acerto de escala”, é transformado no triângulo MBC. Isso é uma transformação geométrica conhecida como roto-homotetia de centro M. Assim, A é levado em B e D é elevado em C. Note que a semelhança obtida anteriormente envolve o centro de roto-homotetia M, os pontos e suas imagens na transformação. Isso na verdade sempre acontece (é uma das semelhanças automáticas). Agora podemos terminar o problema: da semelhança entre MAB e MDC, os ângulos externos ∠MDT e ∠MAS são congruentes. Como M pertence ao circuncírculo de SAE, ∠MAS = ∠MES = ∠MET , ou seja, ∠MDT = ∠MET , o que significa que MEDT é cíclico e, portanto, M pertence ao circuncírculo de TED. Utilizando outra semelhança automática, entre MEF e MDC (pois E é levado em F!), prova-se que M pertence também ao circuncírculo de TFC. Note que se U é a interseção de AB e CD, então pelo teorema de Miquel M também pertence ao circuncírculo de STU. Então na vaerdade cinco círculos passam pelo ponto M! A seguinte versão do teorema de Miquel também é útil:

semelhantes então ∠DMA = ∠CMB ) e

Teorema de Miquel para triângulos. Seja ABC em triângulo e D, E, F pontos sobre as retas BC, CA, AB, respectivamente. Então os circuncírculos de AEF, BFD e CDE têm um ponto em comum. Esse ponto também é chamado de ponto de Miquel.

Demonstração: Seja M a segunda interseção dos circuncírculos de AEF e BFD. Então ∠CDM = ∠BFM = ∠AEM = 180° − ∠CEM . EUREKA! N°31, 2010

30


Sociedade Brasileira de Matemática

Exercícios: 01. Demonstre o teorema de Miquel para quadriláteros utilizando o teorema de Miquel para triângulos. 02. Seja ABCDE um pentágono convexo e F, G, H, I, J as interseções dos prolongamentos de EA, AB, AB, BC, CD, DE e DE, EA respectivamente. Prove que as segundas interseções dos circuncírculos de ABF, BCG, BCG, CDH, DEI, DEI, EAJ, e EAJ, ABF pertencem a uma mesma circunferência. 03. Considere um quadrilátero completo. Seja M o seu ponto de Miquel. Prove que: (a) os circuncentros dos quatro triângulos determinados pelo quadrilátero e M estão sobre uma mesma circunferência. (b) as projeções ortogonais de M sobre as quatro retas do quadrilátero pertencem a uma mesma reta r; além disso, M é o único ponto do plano com essa propriedade. (c) os ortocentros dos quatro triângulos pertencem a uma mesma reta s. (d) as retas r e s são paralelas, e a distância de M e r é metade da distância de M a s. 04. Seja ABCD um quadrilátero convexo e X e Y as interseções dos lados opostos AD e BC e AB e CD, respectivamente. Prove que os pontos médios de AC, BD e XY são colineares. Observação: a reta que passa pelos três pontos é a reta de Gauss do quadrilátero completo. 2. Conjugados isogonais A ideia de conjugado é fazer uma associação entre objetos. Objetos conjugados supostamente têm propriedades semelhantes. Isso é bastante comum em equações: se um número é raiz, então o conjugado também é raiz. Em geometria, também existe a ideia de conjugado. De fato, dado um triângulo, cada ponto tem um conjugado isogonal e um conjugado isotômico. Aqui, trataremos somente de conjugados isogonais. Definição 2.1. Dado um triângulo ABC, o conjugado isogonal em relação a ABC de um ponto T do plano de ABC é obtido refletindo as retas TA, TB e TC em relação às bissetrizes internas de ABC que passam por A, B, e C, respectivamente. As retas resultados são concorrentes no isogonal T −1 de T. A seguir, as linhas pontilhadas são as bissetrizes, e as cevianas cinzas são as reflexões das cevianas pretas.

EUREKA! N°31, 2010

31


Sociedade Brasileira de Matemática

O fato de que as retas isogonais são concorrentes é extremamente importante, tanto que será enunciado novamente. Teorema fundamental dos conjugados isogonais. Dados um triângulo e três retas que passam pelos respectivos vértices e concorrem em um ponto P, as retas isogonais a elas, obtidas através da reflexão em relação à bissetriz interna correspondente, são concorrentes no conjugado isogonal P −1 de P. Demonstração Por que as cevianas cinzas são concorrentes? Isso decorre de duas aplicações do teorema de Ceva trigonométrico: primeiro com as cevianas concorrentes em T e depois, com as cevianas concorrentes em T −1 , que formam os mesmos ângulos que as outras cevianas, porém no sentido contrário. Na verdade, pode ocorrer de as três cevianas serem paralelas. Isso ocorre se, e somente se, T está sobre o circuncírculo de ABC; nesse caso, pensamos projetivamente, ou seja, o conjugado isogonal é um ponto do infinito. 2.1 para que servem isogonais? O que é mais útil em conjugados isogonais é simplesmente que as cevianas são reflexões umas das outras em relação às bissetrizes, e isso costuma levar a algumas igualdades entre ângulos um pouco mais difíceis de obter ou mesmo de se imaginar com contas. EUREKA! N°31, 2010

32


Sociedade Brasileira de Matemática

Exemplos 2.1 No triângulo ABC, P e Q são pontos no interior de ABC tais que ∠CBP = ∠PBQ = ∠QBA = ∠ABC / 3 e ∠BCP = ∠PCQ = ∠QCA = ∠ACB / 3. Sejam D e E as projeções ortogonais de P sobre AB e AC, respectivamente. Prove que AQ é perpendicular a DE. Resolução Seja θ = ∠PAD. Então ∠APD = 90° − θ e, como ∠ADP e ∠AEP são retos, o quadrilátero ADPE é inscritível. Logo ∠AED = ∠APD = 90° − θ.

Olhando a figura, note que basta provarmos que ∠QAC = θ. Aí é que entram os conjugados isogonais. Como ∠PBC = ∠QBA e ∠BCP = ∠QCA, os pares de retas BP;BQ e CP;CQ são simétricos entre si em relação às bissetrizes de ∠ABC e ∠ACB, respectivamente. Ou seja, P e Q são conjugados isogonais e, portanto, ∠PAB e ∠QAC também são iguais. Logo ∠QAC = θ e o ângulo entre as retas AQ e DE é 180° − θ − ( 90 − θ ) = 90°. Note que para provar o resultado na conta, bastaria repetir a demonstração do teorema fundamental dos conjugados isogonais. Mas o mais interessante é que, sabendo da existência dos conjugados isogonais, é natural pensar nessa solução. Em contraste, fazer a conta sem pensar em conjugados isogonais não parece ser tão natural. Então dá para pensar que os conjugados isogonais nos economizaram não só fazer a conta, mas mostraram onde fazer as contas relevantes. EUREKA! N°31, 2010

33


Sociedade Brasileira de Matemática

2.2. Conjugados isogonais dos pontos notáveis Você já deve estar familiarizado com os pontos notáveis do triângulo: o baricentro (encontro das medianas), o incentro (encontro das bissetrizes internas), o ortocentro (encontro das alturas) e o circuncentro (encontro das mediatrizes). Quais são os conjugados isogonais desses pontos? Vamos aproveitar e conhecer mais um ponto notável (mas não tão conhecido). Vamos fazer isso em ordem de dificuldade. Incentro As reflexões coincidem com as próprias bissetrizes. Logo o conjugado isogonal do incentro, que é o encontro das bissetrizes internas, é ele mesmo. O mesmo vale para os ex-incentros (encontros de duas bissetrizes externas e uma bissetriz interna e centros dos ex-incírculos, que são tangentes externamente aos lados ou seus prolongamentos). Pense sobre o assunto! Ortocentro e circuncentro A figura a seguir deve convencê-lo de que o ortocentro e o circuncentro são conjugados isogonais.

Baricentro Os isogonais das medianas são as simedianas (SImétrico + MEDIANA). O ponto de encontro das simedianas é o ponto de Lemoine, também conhecido como ponto simediano. O ponto de Lemoine é costumeiramente denotado por K. Primeiro, vamos aprender a traçá-las de modo mais prático.

EUREKA! N°31, 2010

34


Sociedade Brasileira de Matemática

Lema: Seja D a interseção das retas tangentes ao circuncírculo do triângulo ABC por B e C. Então a reta AD contém a simediana que passa por A. Demonstração

Construa o paralelogramo ABEC. Então AD contém a mediana AM. Afirmamos que D e E são conjugados isogonais. De fato, ∠BCE = ∠B e o ângulo entre AC e CD, pela tangência, é igual a ∠B. Assim, as retas CD e CE são conjugadas isogonais. Analogamente, BD e BE também são, e o resultado segue do teorema fundamental dos conjugados isogonais. Exercícios 05. Sejam P e Q pontos no interior do ângulo ∠BAC tais que BP = CP,BQ = CQ e ∠ABP + ∠AQC = 180°. Prove que ∠BAP = ∠CAQ. 06. As retas obtidas através das reflexões da diagonal BD do quadrilátero ABCD em relação às bissetrizes de ∠B e ∠D passam pelo ponto médio de AC. Prove que as reflexões da diagonal AC do quadrilátero ABCD em relação às bissetrizes de EUREKA! N°31, 2010

35


Sociedade Brasileira de Matemática

∠A e ∠C passam pelo ponto médio de BD.

07. (Prova de Seleção EUA, 2008) Seja ABC um triângulo e G o seu baricentro. O ponto P varia sobre o segmento BC. Os pontos Q e R pertencem aos lados AC e AB respectivamente, e são tais que PQ é paralelo a AB e PR é paralelo a AC. Prove que, ao variar P sobre BC, o circuncírculo de AQR passa por um ponto fixado X tal que ∠BAG = ∠CAX . 08. (IMO 2004, Problema 5) Num quadrilátero convexo ABCD a diagonal BD não é bissetriz do ângulo ∠ABC nem do ângulo ∠CDA. Um ponto P no interior de ABCD satisfaz ∠PBC = ∠DBA e ∠PDC = ∠BDA. Prove que os vértices do quadrilátero ABCD pertencem a uma mesma circunferência se, e somente se, AP = CP. 3. Triângulo Pedal Definição 3.1. Seja P um ponto no plano do triângulo ABC e D, E e F as projeções de P sobre as retas BC, CA e AB. O triângulo DEF é o triângulo pedal de P em relação ao triângulo ABC. O que triângulos pedais têm de especial? Primeiro, aparecem muitos ângulos retos, o que propicia o aparecimento de quadriláteros inscritíveis. Segundo, eles normalmente minimizam áreas. Teorema do mínimo. Dados dois triângulos T e ABC, considere todos os triângulos DEF semelhantes a T, todos na mesma ordem, com D sobre o lado BC, E sobre o lado CA e F sobre o lado AB. Dentre todos esses triângulos, o de menor área é o triângulo pedal de algum ponto P. Demonstração Não provaremos aqui a existência de um triângulo de área mínima (caso você esteja curioso, estude topologia e depois volte!). Seja DEF o triângulo de área mínima. Seja M o ponto de Miquel de ABC e DEF, e sejam P, Q e R as projeções de M sobre os lados.

EUREKA! N°31, 2010

36


Sociedade Brasileira de Matemática

Note que o quadrilátero CPMQ é inscritível (pois ∠MPC e ∠MQC são retos), de modo que ∠DME = ∠PMQ = 180° − ∠C. Portanto, ∠PMD = ∠QME : imagine o ângulo ∠DME girando em torno de M para coincidir com ∠PMQ;MD vira MP e ME vira MQ. Analogamente, ∠RMF = ∠QME. Portanto os triângulos PMD, QME e RMF são semelhantes e induzem uma rotohomotetia (você se lembra o que é isso?) que leva DEF a PQR. A razão de MP homotetia é ≤ 1, de modo que a área de PQR é menor ou igual à área de DEF. MD Como DEF tem área mínima, os triângulos devem ser congruentes e deste modo MP = MD, ou seja, P = D. Analogamente, Q = E e R = F, de modo que DEF é o triângulo pedal de P. Exemplo 3.1. (Prova de seleção EUA, 2008) Sejam P, Q, R pontos sobre os lados BC, CA, AB de um triângulo acutângulo ABC tais que PQR é equilátero e tem área mínima entre todos tais triângulos equiláteros. Prove que a reta perpendicular a QR que passa por A, a reta perpendicular a RP que passa por B e a reta perpendicular a PQ que passa por C têm um ponto comum. Resolução Pelo teorema do mínimo, PQR é triângulo pedal de algum ponto T.

EUREKA! N°31, 2010

37


Sociedade Brasileira de Matemática

Como os ângulos ∠TQA e ∠TRA são ambos retos, o quadrilátero AQTR é inscritível, e o seu circuncentro é o ponto médio A´ de AT . Assim, a reta perpendicular a QR e que passa por A, que contém a altura relativa a QR, é isogonal a AT, que contém o circuncentro, em relação ao triângulo AQR. Como os ângulos ∠BAC e ∠QAR são iguais, a perpendicular e AT são isogonais em relação ao triângulo ABC também. O análogo para as perpendiculares a PR por B e a PQ por C. Como AT, BT e CT são concorrentes em T, seus isogonais são concorrentes no conjugado isogonal de T. A título de curiosidade, o ponto T é o primeiro ponto isodinâmico. Os dois pontos isodinâmicos (adivinhe o nome do outro ponto!) são os pontos de interseção dos círculos de Apolônio de A, B e C (que passam pelos vértices, o pé da bissetriz interna e têm centro sobre o lado oposto). Os seus conjugados isogonais são os pontos de Fermat. O primeiro ponto de Fermat é o ponto cuja soma das distâncias aos vértices é mínima (supondo que os ângulos internos do triângulo são todos menores do que 120° ). Veja [5] para aprender isso e muito, muito mais. 3.1. Voltando às simedianas Uma aplicação interessante da ideia de triângulo pedal está relacionada às simedianas. Uma outra maneira de construir as simedianas é a seguinte: Lema. Construa quadrados ABBc Ac , BCCa Ba e CAAb Cb externamente sobre os lados do triângulo ABC. Prolongue Ac Bc , Ba Ca e Cb Ab para obter o triângulo A´ B´C´. Então as retas AA´,BB´ e CC´ concorrem no ponto simediano K de ABC. EUREKA! N°31, 2010

38


Sociedade Brasileira de Matemática

Demonstração Por simplicidade, sejam BC = a, CA = b e AB = c e L o encontro de AA´,BB´,CC´. Queremos provar que L = K. Primeiro, como os pares de retas AB; A´ B´,BC;B´C´ e CA;C´ A´ são paralelos, os triângulos ABC e A´ B´C´ são semelhantes. Seja k a razão de semelhança. Sejam ka ,kb e kc as distâncias de L a BC, CA e AB, respectivamente. Das semelhanças entre LAB; LA’B, LBC; LB’C’ e LCA; LC’A’, todas de razão k, ka k k k k k k = b = c =k ⇔ a = b = c = ka + a kb + b kc + c a b c 1− k

Isto quer dizer que as distâncias de L a cada um dos lados é proporcional aos seus comprimentos. Além disso, considerando uma semelhança prova-se que um ponto X pertence a, digamos, AL se, e somente, as distâncias de X aos lados AB e AC são proporcionais a seus comprimentos. Basta provar que a simediana por A tem a mesma propriedade. Para isso, considere a construção anterior, sendo D o mesmo ponto definido anteriormente.

EUREKA! N°31, 2010

39


Sociedade Brasileira de Matemática

Sendo x e y as distâncias de D a AB e AC, respectivamente, considerando que o ângulo entre AB e BD é ∠FBD = ∠ACB = ∠C e o ângulo entre AC e CD é ∠DCE = ∠ABC = ∠B (não se preocupe com triângulos obtusângulos; nesse caso, troque o ângulo obtuso por seu suplementar), nos triângulos retângulos BDF e CDE,x = BDsen∠C e y = DCsen∠B. Observando ainda que, sendo DB e DC x sen∠C AB tangentes, DB = DC, temos = = . Logo D pertence a AL e, y sen∠B AC consequentemente, K também. Da mesma forma provamos que K pertence a BL e CL, de modo que L = K. Assim como no teorema das bissetrizes, as simedianas dividem os lados opostos em razões interessantes. 2

BN  AB  Lema. Seja ABC um triângulo e NA uma simediana. Então =  . CN  AC  Demonstração

EUREKA! N°31, 2010

40


Sociedade Brasileira de Matemática

Já provamos anteriormente que as distâncias do ponto simediano K aos lados são proporcionais a seus comprimentos. Então existe t real tal que ka = ta, kb = tb e kc = tc ⋅ c tc 2 tb ⋅ b tb 2 tc. Assim, as áreas de KAB, KAC e KBC são = , = e 2 2 2 2 ta ⋅ a ta 2 = , respectivamente. Logo 2 2 2 BN área ABN área KBN área ABN − área KBN área KAB c 2  AB  = = = = = 2 =  CN área ACN área KCN área ACN − área KCN área KAC b  AC  Lema. Sejam d a ,d b e d c as distâncias de um ponto P aos lados BC, CA, e AB do BN c ⋅ d c triângulo ABC. Se AP corta BC em N , então = . CN b ⋅ db Demonstração Fica a cargo do leitor. 3.2 A desigualdade de Erdös-Mordell Um dos principais teoremas sobre triângulos pedais é a desigualdade de ErdösMordell: Desigualdade de Erdös-Mordell. Seja P um ponto no plano do triângulo ABC e d a ,d b ,d c as distâncias de P às retas BC, CA, AB respectivamente. Então PA + PB + PC ≥ 2 ( d a + d b + d c )

Demonstração Seja PA = d. “Multiplique” a figura original por d e construa triângulos semelhantes aos triângulos obtidos por PA e as projeções de P sobre AB e AC:

EUREKA! N°31, 2010

41


Sociedade Brasileira de Matemática

Note que ∠GDE = 90° − β e ∠HDF = 90° − α , de modo que G, D e H são colineares. Além disso, ∠EGD e ∠FHD são ambos retos, de modo que as retas EG e FH são paralelas. A distância entre essas duas retas é GH = AB ⋅ d b + AC ⋅ d c , que é menor ou igual a EF = d ⋅ BC. Lembrando que d = PA, temos AB AC AB ⋅ db + AC ⋅ d c ≤ PA ⋅ BC ⇔ PA ≥ ⋅ db + ⋅ d c . Analogamente, BC BC AB BC PB ≥ ⋅ da + ⋅ dc AC AC AC BC PC ≥ ⋅ da + ⋅ db AB AB 1 Somando as três desigualdades e lembrando que t + ≥ 2 para todo t real positivo, t  AB AC   AB BC   AC BC  PA + PB + PC ≥  + + +  da +   db +   dc ≥ 2 ( d a + d b + d c )  AC AB   BC AB   BC AC 

Exemplo 3.2. (IMO 1991, Problema 4) Sejam ABC um triângulo e M um ponto interior. Mostre que pelo menos um dos ângulos ∠MAB,∠MBC e ∠MCA é menor ou igual a 30°. Resolução: Sejam P, Q e R as projeções de M sobre BC, CA, e AB, respectivamente.

Pela desigualdade de Erdös-Mordell, MA + MB + MC ≥ 2 ( MP + MQ + MR ) . Se EUREKA! N°31, 2010

42


Sociedade Brasileira de Matemática

1 MR MP MQ são maiores do que , então , , 2 MA MB MC MA < 2 MR,MB < 2 MP e MC < 2 MQ, e MA + MB + MC < 2 ( MP + MQ + MR ) ,

todas

as

razões

contradição. Então uma das razões, digamos,

MR 1 , é menor ou igual a . Todavia, MA 2

MR = sen∠MAB, de modo que ∠MAB ≤ 30°. MA

Exercícios 09. Dado um triângulo com perímetro L, seja P o perímetro de um triângulo pedal. Prove que L ≥ 2 P. Quando ocorre a igualdade? 10. Seja P um ponto interior ao triângulo ABC e T o seu triângulo pedal. Prove que R 2 − OP 2 vezes a área de ABC. a área de T é igual a 4R2 11. Seja G o baricentro do triângulo ABC e D, E, F as projeções ortogonais de G sobre os lados BC, CA e AB, respectivamente. Prove que 4 área DEF 1 < ≤ 27 área ABC 4 12. Seja P um ponto qualquer no plano do triângulo ABC. As projeções de P sobre BC, CA, AB são D, E e F respectivamente. (a) Prove que as perpendiculares a EF, FD, DE por A, B, C respectivamente têm um ponto P´em comum. (b) Sejam Q e Q´ as segundas interseções de AP e AP´ com o circuncírculo de ABC, respectivamente. Prove que as retas QQ´ e BC são paralelas. 13. Os pontos X, Y e Z estão sobre BC, CA, e AB, respectivamente, e são tais que XYZ e ABC são semelhantes, nessa ordem. Prove que o circuncentro de XYZ é equidistante aos ortocentros de ABC e XYZ. 14. (Ibero 2008, Problema 5) Seja ABC um triângulo e X, Y, Z pontos interiores dos lados BC, AC, AB, respectivamente. Sejam A´, B´, C´ os circuncentros dos triângulos AZY, BXZ, CYX, respectivamente. Demonstre que EUREKA! N°31, 2010

43


Sociedade Brasileira de Matemática

( A´ B´C´ ) ≥

( ABC )

4 e que a igualdade ocorre se, e somente se, as retas AA´,BB´,CC´ têm um ponto em comum. Observação: Para um triângulo qualquer RST, denotamos a sua área por (RST).

15. (OPM, 2001)

uuur uuur (a) Na figura acima, considere pontos B1 e C1 sobre as semirretas AB e AC, respectivamente.

(i) Mostre que a soma das áreas dos paralelogramos com lados AB1 e AM e com lados AC1 e AM é igual à área do paralelogramo tal que um de seus lados é B1C1 e o outro é paralelo e igual a AM. (ii) Tomando AB1 = AC e AC1 = AB, conclua que AB ⋅ v + AC ⋅ w ≤ BC ⋅ x (b) Prove a Desigualdade de Erdös-Mordell: 2 ( u + v + w ) ≤ x + y + z Referências Bibliográficas [1] Uma ótima fonte de problemas é o Mathlinks: http://www.mathlinks.ro/ (em inglês). [2] Para quem gosta de Geometria, o Forum Geometricorum é um prato cheio! Tudo sobre quadriláteros completos foi retirado do artigo Steiner´s Theorems on the Complete Quadrilateral, de Jean – Pierre Ehrmann, Volume 4 (2004), pp 35-52. [3] Para quem quer saber mais sobre o teorema de Erdös-Mordell, na Eureka! 18. [4] O livro Modern Geometry of the Triangle, de William Gallatly, contém muita informação interessante, incluindo a maior parte dos fatos sobre simedianas e o ponto simediano. [5] Mais conjugados isogonais? Isso e muito mais no livro Geometry of Conics (o “livro do bode” – Veja a Capa!), de A. V. Akoplyan e A. A. Zaslavsky. EUREKA! N°31, 2010

44


Sociedade Brasileira de Matemática

SÉRIE HARMÔNICA DE NÚMEROS PRIMOS Lenimar Nunes de Andrade UFPB – João Pessoa, PB 1. Série harmônica Há séculos que se sabe que a soma dos recíprocos dos números inteiros positivos 1 1 1 1 H n = 1 + + + + ... + 2 3 4 n pode ultrapassar o valor de qualquer constante positiva pré-estabelecida, bastando, para isso, somar determinada quantidade de parcelas, considerando o valor de n suficientemente grande. Quando consideramos uma infinidade de parcelas desse tipo, temos uma série infinita conhecida pelo nome de série harmônica, e, como a soma H n vai aumentando à medida que n aumenta e ultrapassa qualquer valor préestabelecido, temos que se trata de uma série divergente. Existem pelo menos 20 demonstrações diferentes desse fato (veja, por exemplo, a referência [3]) e algumas demonstrações simples podem ser encontradas em [1] ou [2].* Sabe-se que o crescimento das somas parciais H n da série harmônica é bastante lento. Se somarmos 1000 termos da série, obtemos 7, 4855 como resultado. Se somarmos 1000000 de termos, obtemos 14,3927. Para a soma ultrapassar 100, estima-se que seja necessário somar-se aproximadamente 1,5 × 1043 parcelas – um número de parcelas tão grande que nem os computadores mais modernos de hoje em dia, trabalhando ininterruptamente ao longo de vários milênios, conseguiriam efetuar todos os cálculos. Se for escolhido um determinado algarismo, e retirados da série harmônica todos os termos que contenham esse algarismo, então surpreendentemente, obtem-se uma série infinita na qual a soma dos n primeiros termos é sempre inferior a 80, não

*

Nota do editor: Uma demonstração particularmente simples deste fato é a seguinte:

1 1 1 1 1 1 1 1   1 +  +  +  + + +  + ... +  k −1 + ... + k  ≥ 2 3 4 5 6 7 8 2   2 +1 1 1 1 1 1 1 1 1 1  1 1 1 1  1 ≥ 1 + +  +  +  + + +  + ... +  k + k + ... + k  = 1 + + + + ... + 2  4 4 8 8 8 8 2 2  2 2 2 2 2 H 2k = 1 +

=1+

k , que pode ultrapassar o valor de qualquer constante positiva pré-estabelecida. 2

EUREKA! N°31, 2010

45


Sociedade Brasileira de Matemática

importando qual seja o valor de n**. Obtemos, portanto, o que chamamos de série convergente. 2. Série dos recíprocos de números primos A sequência de números primos 2, 3, 5, 7, 11, 13,... é infinita. Apesar de estarem bem próximos uns dos outros, para valores pequenos, à medida que aumentamos o valor de n, torna-se difícil encontrar números primos maiores do que n, e eles vão ficando cada vez mais distantes uns dos outros em média. Se retirarmos da série hermônica todos os termos cujos denominadores não sejam primos, obtemos ainda assim uma série infinita: 1 1 1 1 1 1 1 + + + + + + + ... 2 3 5 7 11 13 17 vamos denominar a série assim obtida de série harmônica de números primos. O principal objetivo deste artigo é mostrar que essa série harmônica de números primos é divergente, ou seja, a soma dos seus n primeiros termos pode ultrapassar qualquer valor pré-estabelecido. Esse fato foi observado pela primeira vez por Leonhard Euler (1707–1783). O crescimento do valor das somas dos n primeiros termos da série harmônica de números primos é exatamente lento. Muito mais lento do que o da série harmônica. Com os recursos computacionais atuais, é impossível realizar a tarefa de somar uma certa quantidade de termos dessa série e obtermos o resultado igual ou superior a 5,0. 3. Demonstração da divergência Inicialmente, vamos mostrar que dado um n inteiro positivo temos que k

k k2  1 1 +  < 1 + + 2 n n  n para todo inteiro positivo k tal que k ≤ n. Em particular, fazendo k = n, obtemos que n

n n2  1 (1)  1 +  < 1 + + 2 = 3. n n  n Para isso, vamos usar o método da indução matemática. Para k = 1, a desigualdade 1 1 1 reduz-se a 1 + < 1 + + 2 o que é verdadeiro. Suponho a desigualdade válida n n n para k, vamos verificar que vale para k + 1 também:

**

Veja o problema proposto no. 141, na página 60.

EUREKA! N°31, 2010

46


Sociedade Brasileira de Matemática

 1 1 +   n

k +1

2 k k2 1  1   1   k k  1  =  1 +   1 +  <  1 + + 2  1 +  = 1 + + 2 + n n n  n   n   n n  n  k

k 2 − n ( k + 1) k k2 k + 1 ( k + 1) k + 1 ( k + 1) + 2 + 3 =1+ + + <1+ + , 2 3 n n n n n n n2 14 4244 3 2

2

<0

onde utilizamos que k < n ( k + 1) porque k ≤ n. Desse modo, a desigualdade fica demonstrada. Escolhido n, um inteiro positivo qualquer, consideremos r o inteiro tal que 2r ≤ n < 2r +1 , ou seja, r é o expoente da maior potência de 2 que não ultrapassa n. Sejam p1 = 2 , p2 = 3 , p3 = 5 ,..., ps os primos positivos menores ou iguais a n. Se m for um inteiro tal que 1 ≤ m ≤ n, então o Teorema Fundamental da Aritmética nos garante que m pode ser escrito de modo único como um produto de potências dos primos pk com expoentes inteiros não negativos t1 ,t2 ,...,ts : 2

m = p1t1 p2t2 p3t3 ...psts

Note que nenhum dos expoentes tk pode ser maior do que r , pois, se assim fosse, teríamos m ≥ pktk ≥ 2tk ≥ 2r +1 > n, o que seria um absurdo. Assim, para todo k, temos 0 ≤ tk ≤ r. Temos, então, a seguinte desigualdade: 1+

1 1 1 1  1 1 1 + + + ... + <  1 + + + ... + r n  2 4 2 3 4 2

1   1 1  × 1 + + + ... + r 3   3 9

 1 1 1 1   1 1 + ... + r  × ... ×  1 + + 2 + ... + r  1 + + 5  ps ps ps   5 25 

 × 

(2)

Sua demonstração consiste na observação de que cada parcela 1 m que aparece do lado esquerdo da desigualdade pode ser escrita de modo único na forma 1 1 1 1 = t1 × t2 × ... × ts m p1 p2 ps

e que cada fração da forma

1 ocorre uma única vez como uma das parcelas do pktk

 1 1 1  fator 1 + + 2 + ... + r  que aparece no segundo membro da desigualdade. pk pk pk   EUREKA! N°31, 2010

47


Sociedade Brasileira de Matemática

1 2 ≤ e q −1 q calculando a seguinte soma de uma progressão geométrica de razão 1 q , obtemos: 1 1 − r +1 ( q − 1) + 1 = 1 + 1 1 1 1 1 q q 1 + + 2 + ... + r = < = = 1 1 q q q q −1 q −1 q −1 1− 1− q q Usando agora a desigualdade (1) obtida no início desta seção, obtemos

Supondo q ≥ 2 , temos 2q − q ≥ 2 , ou seja, 2 ( q − 1) ≥ q que equivale a

q −1

 1  1 < 3 q −1 . 1 +  < 3 que é equivalente a 1 + q − 1 q − 1   Aplicando-se logaritmos, obtemos:  q1−1   1  log 1 + < log 3 ,     q −1   ou seja,  1  log 3 2 log 3 log 1 + ≤ < q  q −1  q −1 de onde finalmente obtemos  1 1  1  1  2log3 log 1 + + 2 + ... + r  < log  1 + < q  q  q q  q −1  1

(3)

Aplicando-se logaritmos aos dois membros da desigualdade (2), e usando-se propriedade log ( ab ) = log a + log b, obtemos: 1 1  1 1 1  1 1 log 1 + + + + ... +  < log  1 + + + ... + r  + n 2   2 3 4  2 4  1 1 1 1   1 1 log 1 + + + ... + r  + ... + log  1 + + 2 + ... + r  (4) 3  ps ps ps   3 9  Usando-se várias vezes o resultado (3) na desigualdade (4), obtemos: 1  2 log 3 2 log 3 2 log 3 2 log 3  1 1 1 log 1 + + + + ... +  < + + + ... + n 2 3 5 ps  2 3 4

1 1 1 1  = 2 log 3 + + + ... +  . ps  2 3 5

EUREKA! N°31, 2010

48


Sociedade Brasileira de Matemática

Se existisse uma constante L tal que

1 1 1 + + ... + < L para todo inteiro positivo 2 3 ps

1  1 1 1 s, então teríamos log 1 + + + + ... +  < 2 log 3 × L = log ( 32 L ) (5) n  2 3 4 1 1 1 1 (6) o que implicaria 1 + + + + ... + < 32 L , para todo n natural, 2 3 4 n o que seria um absurdo, pois a série harmônica não é limitada e, para algum n, a 1  1 1 soma 1 + + + ... +  ultrapassaria a constante 32 L . n  2 3

4. Algumas somas parciais Sabe-se que quando maior o valor de n, mais próximo de ln ( ln ( n ) ) + B1 será a soma de todos os recíprocos de primo inferiores a n. A constante B1 é conhecida como constante de Mertens e tem valor igual a 0,2614972128.... Com a ajuda de um computador, se somarmos os recíprocos dos números primos inferiores a 1000, obtemos 2,1990 como resultado. Observe que esse valor é próximo de ln ( ln (1000 ) ) + B1 ≈ 2 ,1941. Somando-se todos os recíprocos de númreos primos inferiores a 107 (dez milhões), obtemos uma soma total igual a 3,041449. Calculando-se

(

)

ln ln (107 ) + B1 , obtemos 3,041440 que é muito próximo da soma obtida.

A aproximação ln ( ln ( n ) ) + B1 ≈ S para a soma dos recíprocos de primos inferiores a n pode ser escrita na forma n ≈ ee

S − B1

. 5− B1

Por exemplo, para chegar a 5,0, a soma necessitaria de n = ee = ee ≈ 4, 2 × 1049 parcelas, o que é um número realmente assustador: nem o computador mais rápido de hoje em dia, trabalhando incessantemente por milênios a fio, conseguiria somar tal quantidade de termos. 4 ,7385

Referências Bibliográficas [1] G. Ávila, “As séries infinitas”, RPM 30, 1996. [2] G. Garbi, “A surprendente série harmônica”, RPM 42, 2000. [3] S.J. Kifowit, T. A. Stamps, “The harmonic series diverges again and again”, The AMATYC Review, Vol. 27, No. 2, 2006. [4] D. O. Shkiyarsky, N. N. Chentsov, I. M. Yaglom, “Selected problemas and theorems in elementary Mathematics – Arithmetic and Algebra”, Mir Publishers, Moscow, 1979. EUREKA! N°31, 2010

49


Sociedade Brasileira de Matemática

COMO É QUE FAZ? PROBLEMA PROPOSTO POR WILSON CARLOS DA SILVA RAMOS (BELÉM – PA)

3 85  2 2 = 2  4 xy + 4 ( x + y ) + ( x + y) 3  1) Resolva o sistema   2 x + 1 = 13  x+ y 3

Solução: Da segunda equação, obtemos y =

1 − x. Substituindo esse valor de 13 − 2x 3 2

4x  13  85 + 4 y 2 + 3  − 2 x  = . Fazendo 13 3 3  − 2x 3 13 1  13 1 1 u 13  u = − 2 x, temos x =  − u  , e y = − x = + − , u u 2 6 3 2 3 

y na primeira equação, obtemos

2

26 13  85 26 68 26 4 2 − 2 +  + u −  + 3u 2 = , ou seja, 4u 2 − u − − + = 0. 3u 3 3 3 9 3u u 2 u 1 1 26 140 Fazendo w = u + , temos u 2 + 2 = w2 − 2, donde 4 w2 − w − = 0, e logo u u 3 9 10 7 1 10 1 −7 w= ou w = − . Assim, u + = ou u + = . A primeira dessas 3 6 u 3 u 6 1 equações tem soluções u = 3 e u = , que nos dão as duas soluções 3  1  13 1 1  1  13 1   2  reais  x =  − u  = , y = − x = −  e  x =  − u  = 2, y = − x = 1 . A 2 3 u 3  2 3 u  3    segunda não tem soluções reais, mas tem as soluções complexas conjugadas 7 i 95 , que nos dão as soluções complexas correspondentes u=− ± 12 12  59 + i 95 −73 − i 95   59 − i 95 −73 + i 95  , , ( x, y ) =   e ( x, y ) =   . 24 24 24 24    

donde

EUREKA! N°31, 2010

50


Sociedade Brasileira de Matemática

PROBLEMA PROPOSTO POR MARCÍLIO MIRANDA DE CARVALHO (TERESINA – PI) (teste de seleção da Romênia para IMO de 1978)

2) Para cada n natural, resolva a equação: sen x ⋅ sen 2 x...sen nx + cos x ⋅ cos 2 x...cos nx = 1 SOLUÇÃO DE RENAN HENRIQUE FINDER (JOINVILLE – SC)

Se n ≥ 2, usando a desigualdade triangular e o fato de que max { sen α , cos α } ≤ 1 , temos 1 = sen x sen 2x...sen nx + cos x cos2 x...cos nx ≤ sen x sen 2x...sen nx + cos x cos2 x...cos nx ≤ sen x ⋅ sen 2 x + cos x ⋅ cos 2 x

Pela desigualdade de Cauchy-Schwarz, 1 ≤ sen 2 x + cos 2 x sen 2 2 x + cos 2 2 x = 1. Para que ocorra a igualdade, devemos ter sen x cos 2 x = cos x sen 2 x, logo sen x = sen ( 2 x − x ) = 0, e portanto x = mπ , m ∈ ¢ ⇒ senx = 0. Então cos x cos 2 x...cos nx = 1. Se m é par, isso sempre ocorre. Se m é ímpar, n

cos x = −1, cos 2 x = 1, cos3 x = −1,..., logo cos x...cos nx = ( −1)  2  . Portanto,

n Se n ≥ 2 e   é par (ou seja, se n é da forma 4j – 1 ou 4j), as soluções são 2 x = mπ , m ∈ ¢. n Se n ≥ 2 e   é ímpar (ou seja, se n é da forma 4j + 1 ou 4j + 2), as 2 soluções são x = 2mπ , m ∈ ¢.

Se n = 1 o problema equivale a ⇔ x = 2mπ ou x = 2mπ +

2 2 2 π π  sen x + cos x ⇔ sen = sen  x +  = 2 2 2 4 4 

π , m ∈ ¢. 2

EUREKA! N°31, 2010

51


Sociedade Brasileira de Matemática

SOLUÇÕES DE PROBLEMAS PROPOSTOS

! 123.

Publicamos aqui algumas das respostas enviadas por nossos leitores.

Determine

todas

as

funções

f : ¥* →¥*

tais

que

2 f (m + n ) = f (m) f (n) + f (m) f (n) , para quaisquer m, n ∈ ¥ * distintos. Obs: ¥* = {1, 2,3,...} é o conjunto dos inteiros positivos. 2

2 3

2

2

SOLUÇÃO DE ÍTALO DOWELL LIRA MELO (TERESINA – PI) Primeiro note que se f é uma função constante então f é solução. Agora suponhamos que exista uma função não constante que seja solução. Assim existem naturais a e b com f ( a ) < f ( b ) . Daí temos que 2 f ( a ) = f ( a ) + f ( a ) < f ( a ) f ( b ) + f ( a ) f ( b ) < 2 f ( b ) . 3

3

3

2

2

3

Como 2f ( a2 + b2 ) = f ( a) f ( b) + f ( a) f ( b) , segue que 2 f ( a) < 2 f ( a2 + b2 ) < 2 f ( b) . 3

2

2

3

3

3

Se dividirmos por 2 encontramos que f ( a ) < f ( a 2 + b2 ) < f ( b ) ⇒ 3

3

3

⇒ f ( a ) < f (a 2 + b 2 ) < f ( b ) . Isto nos diz que entre quaisquer dois valores distintos de f podemos encontrar um outro valor de f mas isto não pode ocorer sempre uma vez que f assume valores em ¥*. Esta contradição mostra que tal função não existe. Assim as funções constantes são as únicas soluções. 124.

Considere a seqüência

an =

an −1an −3 + an2− 2 an − 4

(an )n ≥1

definida por

a1 = a2 = a3 = a4 = 1

, ∀n ≥ 5.

Prove que an é um inteiro positivo, para todo inteiro positivo n. SOLUÇÃO DE ZOROASTRO AZAMBUJA NETO (RIO DE JANEIRO – RJ) Vamos provar por indução que, para todo n ≥ 5, valem as seguintes afirmações: •

ak − 4 ak −1ak −3 + ak2− 2 , ∀k ∈ ¥,5 ≤ k ≤ n (e logo ak ∈ ¥, ∀k ≤ n ).

ak −3 ak3− 2 + ak2−1ak − 4 , ∀k ∈ ¥,5 ≤ k ≤ n .

ak − 2 ak −1ak2− 4 + ak3− 3 , ∀k ∈ ¥,5 ≤ k ≤ n .

mdc ( ak −1 , ak ) = 1, ∀k ∈ ¥,1 ≤ k − 1 < k ≤ n.

mdc ( ak − 2 , ak ) = 1, ∀k ∈ ¥,1 ≤ k − 2 < k ≤ n.

EUREKA! N°31, 2010

52

e


Sociedade Brasileira de Matemática

mdc ( ak −3 , ak ) = 1, ∀k ∈ ¥,1 ≤ k − 3 < k ≤ n.

Note que, como a1 = a2 = a3 = a4 = 1, temos a5 = 2 e a6 = 3 , e portanto os itens acima se verificam para todo n ≤ 6 . Vamos agora verificá-los para k= n + 1: queremos mostrar que ( an−1an−3 + an2−2 ) an−2 + an2−1an−4 .  a a + an2− 2  2 an −3 an an − 2 + an2−1 =  n −1 n − 3 a + a =  n−2 n −1 an − 4 an − 4   Como o lado direito é inteiro e mdc ( an −3 , an − 4 ) = 1, isso equivale a mostrar que an −3 ( an −1an − 3 + an2− 2 ) an − 2 + an2−1an − 4 , o que segue de an −3 an3− 2 + an2−1an − 4 .

Também queremos mostrar que

an3−1an2− 4 + ( an −1an − 3 + an2−2 ) an −3  a a + an2− 2  +  n −1 n −3 = a .  n −3 an − 4 an2− 4   2

2

an − 2 a

3 n −1

+a a

2 n n −3

=a

3 n −1

Como o lado direito é inteiro e mdc ( an − 2 , an − 4 ) = 1, isso equivale a mostrar que an − 2 an3−1an2− 4 + ( an −1an −3 + an2−2 ) an −3 , o que equivale a 2

an − 2 an3−1an2− 4 + an2−1an3−3 = an2−1 ( an −1an2− 4 + an3−3 ) , e isso segue de a n − 2 a n −1 a n2− 4 + a n3− 3 .

E também queremos mostrar que

an −1an −3 + an2− 2 ) an2−3 + an3− 2 an − 4 (  an −1an − 3 + an2− 2  2 3 an −1 a a + a =  .  an −3 + an − 2 = an − 4 an − 4   Como o lado direito é inteiro e mdc ( an −1 , an − 4 ) = 1, isso equivale a mostrar que 2 n n −3

3 n−2

an −1 ( an −1an − 3 + an2− 2 ) an2−3 + an3− 2 an − 4 , o que equivale a

an −1 an2− 2 an2− 3 + an3− 2 an − 4 = an2− 2 ( an2−3 + an − 2 an − 4 ) , que segue de an −1 an2−3 + an − 2 an − 4 ,

que por sua vez segue da igualdade an −1an −5 = an2−3 + an − 2 an − 4 , que vem da definição de an −1. Finalmente, de an +1an −3 = an an − 2 + an2−1 , segue que mdc ( an +1 , an ) mdc ( an , an2−1 ) = 1, mdc ( an +1 , an −1 ) mdc ( an −1 , an an − 2 ) = 1 e mdc ( an +1 , an − 2 ) mdc ( an − 2 , an2−1 ) = 1. 125. Considere dois naturais (a0 , a1 , a2 ,..., amn ), ai ∈ {0,1}.

m≥2

As seqüências de tipo m satisfazem as condições: EUREKA! N°31, 2010

53

e

n ≥ 2,

e

as

seqüências


Sociedade Brasileira de Matemática

• ak ak + m = 0, para todo k; • Se ak ak +1 = 1 então m divide k As seqüências de tipo n são definidas analogamente. Prove que existem tantas seqüências do tipo m quanto do tipo n. SOLUÇÃO DE JOSÉ DE ALMEIDA PANTERA (RIO DE JANEIRO – RJ) Considere a matriz m × n, B = ( bij )1≤i ≤ m dada por bij = am (i −1) + j ,1 ≤ i ≤ m,1 ≤ j ≤ n. 1≤ j ≤ n

Temos que ( a0 ,..., amn ) é uma sequência de tipo m se e somente se B é uma matriz m × n cujas entradas pertencem a {0, 1} sem dois termos vizinhos iguais a 1 numa mesma linha ou numa mesma coluna, e tal que b1m = 1 ⇒ a0 = 0. Considere agora a função f que leva uma matriz B m × n na matriz f ( B ) n × m dada por

( f ( B ))

i, j

= B( m +1− j ),( n +1−i ) . Temos que f é uma bijeção entre as matrizes

m × n cujas entradas pertencem a {0, 1} sem dois termos vizinhos iguais a 1 numa mesma linha ou numa mesma coluna e as matrizes n × m cujas entradas pertencem a {0, 1} sem dois termos vizinhos iguais a 1 numa mesma linha ou numa mesma coluna, tal que ( f ( B ) )1,m = B1,n . Isso mostra que o número de seqüências do tipo m é igual ao número de seqüências do tipo n. 126. As circunferências Γi , 0 ≤ i ≤ 5, são tangentes a uma circunferência Γ nos pontos Ai . Além disso, Γi é tangente a Γi+1 para 0 ≤ i ≤ 5 e Γ5 é tangente a Γ0 . Prove que A0 A3 , A1 A4 , A2 A5 são concorrentes.

SOLUÇÃO DE MATHEUS SECCO TORRES DA SILVA (RIO DE JANEIRO – RJ) Sejam Oi os centros de Γi e ri os raios de Γi . Além disso, seja r o raio de Γ , que tem centro O. Vamos supor inicialmente que as circunferências Γi são exteriores a Γ . Usando Ceva trigonométrico no ∆A2 A4 A0 , devemos provar que sen ( < A1 A4 A2 ) sen ( < A3 A0 A4 ) sen ( < A5 A2 A0 ) ⋅ ⋅ =1⇔ sen ( < A1 A4 A0 ) sen ( < A3 A0 A2 ) sen ( < A5 A2 A4 )

 A OA   A OA   AOA  sen  < 1 2  sen  < 3 4  sen  < 5 0  2  2  2     ⋅ ⋅ =1  A OA   A OA   A OA  sen  < 2 3  sen  < 4 5  sen  < 0 1  2  2  2     EUREKA! N°31, 2010

54


Sociedade Brasileira de Matemática

A0 A5

A1

A4

A2 A3

Denotamos < AOA i i +1 = α i ,0 ≤ i ≤ 5, índices módulo 6. Mas no ∆OOi Oi +1 , temos:

( ri + ri +1 )

2

= ( r + ri ) + ( r + ri +1 ) − 2 ( r + ri )( r + ri +1 ) ⋅ cos α i ⇒

cos α i = 1 − 1 − 2 sen 2 sen 2

2

2

2ri ri +1 ⇒ ( r + ri )( r + ri +1 )

αi 2ri ri +1 =1− ⇒ 2 ( r + ri )( r + ri +1 )

ri ri +1 αi = , donde 2 ( r + ri )( r + ri +1 )

α  α  α  sen 2  1  sen 2  3  sen 2  5   2 ⋅  2 ⋅  2  =1⇒ α α α       sen 2  2  sen 2  4  sen 2  6   2   2   2  α  α  α  sen  1  sen  3  sen  5   2 ⋅  2 ⋅  2  = 1, pois sen  α i  > 0, ∀i ≤ 5,   α  α  α   2 sen  2  sen  4  sen  6   2   2   2  donde obtemos o desejado. Isso conclui a prova!. Obs.: Supusemos que as circunferências Γi estavam no exterior de Γ1 , mas se EUREKA! N°31, 2010

55


Sociedade Brasileira de Matemática

fossem interiores, obteríamos sen2

αi ri ri +1 = , o que também nos daria o 2 ( r − ri )( r − ri +1 )

desejado, com um argumento análogo. 129. Um coelho está numa rua infinita dividida em quadrados numerados pelos

inteiros, e começa no quadrado 0. Se num dado momento ele está no quadrado k, ele escolhe, com probabilidade

1 , pular para o quadrado k + 2 ou, também com 2

1 , pular para o quadrado k – 1. Ele continua esse processo 2 indefinidamante. Dado m ∈ ¢ , determine a probabilidade de, em algum momento,

probabilidade

o coelho pisar no quadrado m.

SOLUÇÃO DE ASDRUBAL PAFÚNCIO SANTOS (BOTUCATU – SP) Denotemos por am a probabilidade de, em algum momento, o coelho pisar no quadrado m. Seja n ≠ 0 um inteiro. Após o primeiro passo do coelho, ele pode estar no quadrado 1 1 –1, com probabilidade , ou no quadrado 2, com probabilidade , ficando a 2 2 distâncias respectivamente n + 1 e n – 2 do quadrado n. Assim, para todo 1 1 n ≠ 0, an = an − 2 + an +1 ( *) . 2 2 1 2 n 5  n  ∑   tende a 0 exponencialmente rápido, com probabilidade total 2n k =0  k  temos que, para um certo n0 ∈ ¥, e para todo n ≥ n0 , pelo menos 40% dos n primeiros passos do coelho são para frente, o que faz com que, após n passos, ele 2n 3n n esteja num quadrado de número maior ou igual a 2 ⋅ − = , para todo 5 5 5 n ≥ n0 . Em particular, a probabilidade de o coelho pisar no quadrado m tende a 0 quando m tende a −∞ .

Como

De (*), temos am + 3 = 2am + 2 − am , ∀m ≥ 0, donde existem constantes A, B, C com m

m

1+ 5  1− 5  am = A + B   + C   , ∀m ≥ 0, pois o polinômio característico da  2   2  EUREKA! N°31, 2010

56


Sociedade Brasileira de Matemática

  1 + 5   1− 5  x −  recorrência acima é x3 − 2 x 2 + 1 = ( x − 1)  x −      .    2    2   Como ( am ) é limitada, devemos ter B = 0. Como a0 = 1, devemos ter A + C = 1.

De (*), também temos a− m −3 = 2a− m − a− m +1 , ∀m ≥ 1, ou seja, fazendo bk = a− k , temos bk + 3 = 2bk +1 − bk , ∀k ≥ 0, donde, como   −1 + 5    −1 − 5   °,C ° com x3 − 2 x + 1 = ( x − 1)  x −  x −  A, B     , existem °   2 2       k

k

°  −1 + 5  + C °  −1 − 5  , ∀k ≥ 0. Como b não só é limitado como bk = ° A+ B k    2  2    ° = 0, e como b = a = 1, tende a 0 quando k tende a +∞ , devemos ter ° A=C 0

° = 1. devemos ter B

 1− Assim, am = 1 − C + C   2

−m

m

0

m

 −1+ 5   1+ 5  5  , ∀m ≥ 0 e am = b−m =   =   , ∀m ≤ 0.   2   2  1 1 Fazendo n = 1 em (*), obtemos a1 = a−1 + a2 , donde 2 2   1 − 5  1  −1 + 5  1  3 − 5  1 − C + C   =   + 1 − C + C    , e portanto 2  2  2  2  2  7−3 5 C= . 2  3 5 − 5  7 − 3 5  1 − 5  m  +    , ∀m ≥ 0 2   2   2  Assim, temos am =  . m  1 + 5   , ∀m ≤ 0   2  130. Suponha que a, b, c ∈ ¡ e a equação x 2 − ( a + b + c ) x + ( ab + ac + bc) = 0 não

tem raízes reais. Prove que a, b e c têm todos o mesmo sinal e existe um triângulo de lados

a,

b e

c.

EUREKA! N°31, 2010

57


Sociedade Brasileira de Matemática

SOLUÇÃO DE DANIEL EITI NISHIDA KAWAI (TAUBATÉ – SP) Se a equação x 2 − ( a + b + c ) ⋅ x + ( ab + ac + bc ) = 0 não tem raízes reais, temos ∆ < 0 ⇔ ( a + b + c ) − 4 ⋅ 1 ⋅ ( ab + ac + bc ) < 0 ⇔ 2

⇔ a 2 + b2 + c 2 + 2ab + 2 ac + 2bc − 4ab − 4ac − 4bc < 0 ⇔ ⇔ a 2 + b2 + c 2 − 2ab − 2 ac − 2bc < 0 ⇔

⇔ a 2 + b2 + c 2 + 2ab − 2 ac − 2bc − 4ab < 0 ⇔ ( a + b − c ) − 4ab < 0 ⇔ 2

⇔ ( a + b − c ) < 4ab ⇔ 0 ≤ ( a + b − c ) < 4ab ⇒ 4ab > 0 ⇒ ab > 0 ⇒ a e b têm o mesmo sinal. De maneira análoga, b e c têm o mesmo sinal. Assim, a, b e c têm todos o mesmo sinal, e logo ab = a b , ac = a c e bc = b c . 2

2

Assim, a 2 + b2 + c 2 − 2 ab − 2 ac − 2 bc < 0, donde ⇒ a 2 + b2 + c 2 + 2 ab − 2 ac − 2 bc < 4 ab ⇒

⇒ a + b + c + 2 a b −2 a c −2 b c <4 a b ⇒ 2

2

2

(

) ⇒ −2 a ⋅ b < a + b − c ⇒ b + b ⇒( c ) <( a ) + 2 a ⋅ b +( b ) b ) ⇒ c < a + b.

⇒( a + b − c) < 2 a ⋅ b 2

2

⇒ c < a +2 a ⋅ ⇒

( c ) <( 2

2

a +

2

2

2

De maneira análoga, b <

a +

a, b e

c

e

a <

b +

c . Assim, existe um triângulo de lados

c.

Agradecemos o envio de soluções e a colaboração de: Carlos Alberto da Silva Victor (Nilópolis – RJ) Rodrigo dos Anjos Azevedo (Três Rios – RJ) Vinicius dos Nascimento S. Mano (Petrópolis – RJ) Marcelo Robeiro de Souza (Rio de Janeiro – RJ) Jheimyson Rego Barnabé (Imperatriz – MA) Flávio Antonio Alves (Amparo – SP) Ítalo Dowell Lira Melo (Teresina – PI) Curro Fernández López (Lugo, Espanha) Miguel Amengual Covas (Mallorca, Espanha) Bruno Salgueiro Fanego (Galicia, Espanha)

Prob. 123, 130 Prob. 130 Prob. 130 Prob. 130 Prob. 130 Prob. 130 Prob. 130 Prob. 42 Prob. 110 Prob. 116, 117, 118

Continuamos aguardando soluções para os problemas 131 e 132. EUREKA! N°31, 2010

58


Sociedade Brasileira de Matemática

PROBLEMAS PROPOSTOS *

Convidamos o leitor a enviar soluções dos problemas propostos e sugestões de novos problemas para próximos números.

133) Considere um n–ágono regular inscrito em um círculo unitário, fixe um vértice i e denote por dj a distância entre este vértice i e o vértice j. Prove que n −1

∏ (5 − d ) = F j ≠i j =0

2 j

2 n

onde F1 = 0, F1 = 1 e Fn = Fn −1 , Fn − 2 se n ≥ 2.

134) Considere a operação ⋅ entre dois vetores do ( x, y , z ) ⋅ ( u, v, w ) = ( xu + yw + zv, xw + zu + yv, xv + yu + zw )

¡3

definida por:

Prove que, para todo k ≥ 1, se ( x, y , z ) = ( 0,0,0 ) então x = y = z = 0. k

( x, y , z ) ∈ ¡ 3 , ( x, y , z ) = ( x, y , z ) k k −1 ( x, y , z ) = ( x , y , z ) ⋅ ( x, y , z ) . 1

Obs.: Para qualquer

e, para todo

k > 1,

135) Considere um hemisfério cuja base é um círculo ( C1 ) . Um círculo ( C2 ) do

hemisfério é paralelo a ( C1 ) , de forma que existem n círculos do hemisfério, congruentes, tangentes entre si, a ( C1 ) e a ( C2 ) . Mostre que a razão K(n) entre os raios de ( C2 ) e ( C1 ) é igual a: K ( n ) =

cos 2 π n . 1 + sen 2 π n

136) Sejam R, r1 , r2 e r3 os raios dos círculos de centro O, O1 , O2 e O3 , respectivamente, conforme a figura abaixo. Prove que: R = r1r2 + r1r3 + r2 r3 .

O1 R O R2

O2

O3

137) Sendo A um conjunto de quinze pontos de ¡ 2 tal que a distância de cada ponto à origem é positiva e menor do que 1 e que quaisquer dois deles nunca sejam EUREKA! N°31, 2010

59


Sociedade Brasileira de Matemática

colineares com a origem. Mostre que existe um triângulo com dois vértices em A e 1 um na origem cuja área é menor que . 4 138) Calcule o máximo divisor comum entre todos os números da forma x ⋅ y ⋅ z , onde ( x, y , z ) percorre todas as soluções inteiras da equação x 2 + y2 = z 2 com x ⋅ y ⋅ z ≠ 0. 139) Determine todos os inteiros positivos x, y, z satisfazendo x3 − y3 = z 2 , onde y é primo, z não é divisível por 3 e z não é divisível por y. 140) Mostre que 2903n − 803n − 464n + 261n é divisível por 1897, para todo n ∈ ¥. 141) Dado a ∈ {0,1, 2,3, 4,5,6,7,8,9} , seja X ≠ ∅ um conjunto finito de inteiros positivos, tal que nenhum dos seus elementos possui o algarismo a em sua 1 representação decimal. Prove que ∑ < 80. n∈ X n

Problema 133 e 134 proposto por Evandro Makiyama de Melo (São Paulo – SP) (foram propostos originalmente na IX e na II Olimpíada Iberoamericana de Matemática Universitária, respectivamente); 135 e 136 propostos por Ramilson Medeiros Pitombeira (Rio de Janeiro – RJ); 137 proposto por Ítalo Dowell Lira Melo (Teresina – PI); 138 proposto por Luiz Felipe Silva; 139 proposto por Adriano Carneiro (Caucaia – CE); 140 proposto por Wilson Carlos da Silva Ramos (Belém – PA).

EUREKA! N°31, 2010

60


Sociedade Brasileira de Matemática

AGENDA OLÍMPICA XXXII OLIMPÍADA BRASILEIRA DE MATEMÁTICA NÍVEIS 1, 2 e 3 Primeira Fase – Sábado, 12 de junho de 2010 Segunda Fase – Sábado, 18 de setembro de 2010 Terceira Fase – Sábado, 16 de outubro de 2010 (níveis 1, 2 e 3) Domingo, 17 de outubro de 2010 (níveis 2 e 3 - segundo dia de prova). NÍVEL UNIVERSITÁRIO Primeira Fase – Sábado, 18 de setembro de 2010 Segunda Fase – Sábado, 16 e Domingo, 17 de outubro de 2010

ASIAN PACIFIC MATH OLYMPIAD (APMO) 06 de março de 2010

XVI OLIMPÍADA DE MAIO 08 de maio de 2010

XXI OLIMPÍADA DE MATEMÁTICA DO CONE SUL 13 a 19 de junho de 2010 Águas de São Pedro, SP – Brasil

LI OLIMPÍADA INTERNACIONAL DE MATEMÁTICA 02 a 14 de julho de 2010 Astana, Cazaquistão

XVII OLIMPÍADA INTERNACIONAL DE MATEMÁTICA UNIVERSITÁRIA 24 a 30 de julho de 2010 Blagoevgrad, Bulgária

XXIV OLIMPÍADA IBEROAMERICANA DE MATEMÁTICA 17 a 27 de setembro de 2010 Paraguai

II COMPETIÇÃO IBEROAMERICANA INTERUNIVERSITÁRIA DE MATEMÁTICA 3 a 9 de outubro de 2010 Rio de Janeiro, Brasil

XIII OLIMPÍADA IBEROAMERICANA DE MATEMÁTICA UNIVERSITÁRIA

EUREKA! N°31, 2010

61


Sociedade Brasileira de Matemática

COORDENADORES REGIONAIS Alberto Hassen Raad Américo López Gálvez Andreia Goldani Antonio Carlos Nogueira Benedito Tadeu Vasconcelos Freire Carmen Vieira Mathias Claus Haetinger Cláudio de Lima Vidal Denice Fontana Nisxota Menegais Disney Douglas Lima de Oliveira Edson Roberto Abe Edney Aparecido Santulo Jr. Élio Mega Eudes Antonio da Costa Fábio Brochero Martínez Florêncio Ferreira Guimarães Filho Francinildo Nobre Ferreira Genildo Alves Marinho Graziela de Souza Sombrio Gilson Tumelero Ivanilde Fernandes Saad João Benício de Melo Neto João Francisco Melo Libonati Jose de Arimatéia Fernandes José Luiz Rosas Pinho José Vieira Alves José William Costa Krerley Oliveira Licio Hernandes Bezerra Luciano G. Monteiro de Castro Luzinalva Miranda de Amorim Marcelo Rufino de Oliveira Marcelo Mendes Newman Simões Nivaldo Costa Muniz Nivaldo de Góes Grulha Jr. Osnel Broche Cristo Uberlândio Batista Severo Raul Cintra de Negreiros Ribeiro Ronaldo Alves Garcia Rogério da Silva Ignácio Reginaldo de Lima Pereira Reinaldo Gen Ichiro Arakaki Ricardo Amorim Sérgio Cláudio Ramos Seme Gebara Neto Tadeu Ferreira Gomes Tomás Menéndez Rodrigues Valdenberg Araújo da Silva Vânia Cristina Silva Rodrigues Wagner Pereira Lopes

(UFJF) (USP) FACOS (UFU) (UFRN) (UNIFRA) (UNIVATES) (UNESP) (UNIPAMPA) (UFAM) (Colégio Objetivo de Campinas) (UEM) (Grupo Educacional Etapa) (Univ. Federal do Tocantins) (UFMG) (UFES) (UFSJ) (Centro Educacional Leonardo Da Vinci) (UNOCHAPECÓ) (UTFPR) (UC. Dom Bosco) (UFPI) (Grupo Educacional Ideal) (UFPB) (UFSC) (UFPB) (Instituto Pueri Domus) (UFAL) (UFSC) (Sistema Elite de Ensino) (UFBA) (Grupo Educacional Ideal) (Colégio Farias Brito, Pré-vestibular) (Cursinho CLQ Objetivo) (UFMA) (USP – São Carlos) (UFLA) (UFPB)) (Colégio Anglo) (UFGO) (Col. Aplic. da UFPE) (Escola Técnica Federal de Roraima) (UNIFESP) (Centro Educacional Logos) (IM-UFRGS) (UFMG) (UEBA) (U. Federal de Rondônia) (U. Federal de Sergipe) (U. Metodista de SP) (CEFET – GO)

EUREKA! N°31, 2010

62

Juiz de Fora – MG Ribeirão Preto – SP Osório – RS Uberlândia – MG Natal – RN Santa María – RS Lajeado – RS S.J. do Rio Preto – SP Bagé – RS Manaus – AM Campinas – SP Maringá – PR São Paulo – SP Arraias – TO Belo Horizonte – MG Vitória – ES São João del Rei – MG Taguatingua – DF Chapecó – SC Pato Branco – PR Campo Grande – MS Teresina – PI Belém – PA Campina Grande – PB Florianópolis – SC Campina Grande – PB Santo André – SP Maceió – AL Florianópolis – SC Rio de Janeiro – RJ Salvador – BA Belém – PA Fortaleza – CE Piracicaba – SP São Luis – MA São Carlos – SP Lavras – MG João Pessoa – PB Atibaia – SP Goiânia – GO Recife – PE Boa Vista – RR SJ dos Campos – SP Nova Iguaçu – RJ Porto Alegre – RS Belo Horizonte – MG Juazeiro – BA Porto Velho – RO São Cristovão – SE S.B. do Campo – SP Jataí – GO


CONTEÚDO XXXI OLIMPÍADA BRASILEIRA DE MATEMÁTICA Problemas e soluções da Primeira Fase

2

XXXI OLIMPÍADA BRASILEIRA DE MATEMÁTICA Problemas e soluções da Segunda Fase

14

XXXI OLIMPÍADA BRASILEIRA DE MATEMÁTICA Problemas e soluções da Terceira Fase

34

XXXI OLIMPÍADA BRASILEIRA DE MATEMÁTICA Problemas e soluções da Primeira Fase Nível Universitário

59

XXXI OLIMPÍADA BRASILEIRA DE MATEMÁTICA Problemas e soluções da Segunda Fase Nível Universitário

65

XXXI OLIMPÍADA BRASILEIRA DE MATEMÁTICA Premiados

75

AGENDA OLÍMPICA

81

COORDENADORES REGIONAIS

82


Sociedade Brasileira de Matemática

XXXI OLIMPÍADA BRASILEIRA DE MATEMÁTICA Problemas e soluções da Primeira Fase PROBLEMAS – NÍVEL 1

1 1 5 de um número é , quanto vale desse número? 8 5 8 1 1 8 B) C) 1 D) A) 8 5 5

1. Se

2. Na figura, C é um ponto do segmento BD tal que ACDE é um retângulo e ABCE é um paralelogramo de área 22 cm2. Qual é a área de ABDE, em cm2? A) 28 B) 33 C) 36 D) 42 E) 44

E) 2

A B

E C D

3. Numa festa, o número de pessoas que dançam é igual a 25% do número de pessoas que não dançam. Qual é a porcentagem do total de pessoas na festa que não dançam? A) 50% B) 60% C) 75% D) 80% E) 84% 4. De quantas maneiras dois casais podem sentar-se em quatro cadeiras em fila se marido e mulher devem sentar-se em cadeiras vizinhas? A) 2 B) 4 C) 8 D) 12 E) 24 5. Eliana tem 27 cubos iguais em tamanho, mas 4 são brancos e os demais, pretos. Com esses 27 cubos, ela monta um cubo maior. No máximo, quantas faces inteiramente pretas ela poderá obter? A) 1 B) 2 C) 3 D) 4 E) 5 6. A figura abaixo é o mapa de um bairro: os pontos A, B, C e D são as casas e os segmentos são as ruas. De quantas casas é possível fazer um caminho que passa exatamente uma vez por cada uma das ruas? É permitido passar mais de uma vez por uma mesma casa.

EUREKA! N°32, 2010

2


Sociedade Brasileira de Matemática

A

B

D

C

A) 0

B) 1

C) 2

7. Se a = 240, b = 320 e c = 710, então: A) c < b < a B) a < c < b C) b < a < c

D) 3

E) 4

D) b < c < a

E) c < a < b

8. Esmeralda lançou um dado dez vezes e obteve 57 como soma de todos os pontos obtidos nesses lançamentos. No mínimo, quantas vezes saíram 6 pontos? A) 5 B) 6 C) 7 D) 8 E) 9 9. Usando palitos de fósforos, podemos construir um hexágono regular, formado por seis triângulos equiláteros unitários, como mostra a figura. Juntando mais palitos a esse hexágono, queremos obter outro hexágono regular com o quádruplo da área, também formado por triângulos equiláteros unitários. Quantos palitos deverão ser acrescentados? A) 12 B) 24 C) 30 D) 36 E) 48 10. Cinco cartas iguais têm um lado branco e um lado preto. Elas se encontram em fila com a face branca para cima. Um movimento consiste em escolher um único par de cartas vizinhas e virá-las. No mínimo, quantos movimentos são necessários para que as cartas fiquem como na figura ao lado? B) 3 C) 4 A) 2 obter a configuração acima.

EUREKA! N°32, 2010

3

D) 5

E) Não é possível


Sociedade Brasileira de Matemática

11. Uma barra de chocolate é dividida entre Nelly, Penha e Sônia. Sabendo que 2 1 da barra, Penha ganha e Sônia ganha 70 gramas, o peso da Nelly ganha 5 4 barra, em gramas, é: A) 160 B) 200 C) 240 D) 280 E) 400 12. Numa fila para compra de ingressos para um jogo da seleção brasileira, havia 49 pessoas: 25 corintianos, 14 flamenguistas e 10 gremistas. Sabendo que cada pessoa da fila torce para um único time, dois torcedores do mesmo time não estão em posições consecutivas, podemos concluir que: A) tal fila não existe. B) algum dos torcedores das extremidades da fila é gremista. C) algum dos torcedores das extremidades da fila é flamenguista. D) algum flamenguista é vizinho de um gremista. E) algum gremista é vizinho de dois corintianos. 13. Na figura, P é um ponto da reta CD. A região cinza é comum ao retângulo ABCD e ao triângulo ADP. Se AB = 5 cm, AD = 8 cm e a área da região cinza 3 é da área do retângulo, quanto vale a distância 4 PC? A) 1 cm B) 2 cm C) 3 cm D) 4 cm E) 5 cm

A

B

Q

D

C

P

14. Numa pesquisa sobre o grau de escolaridade, obtiveram-se os resultados expressos no gráfico abaixo: Que fração do total de entrevistados representa o total de pessoas que terminaram pelo menos o Ensino Fundamental?

EUREKA! N°32, 2010

4


Sociedade Brasileira de Matemática

A)

1 17

B)

3 13

C)

5 16

D)

11 13

E)

16 17

15. Um número natural A de três algarismos detona um número natural B de três algarismos se cada algarismo de A é maior do que o algarismo correspondente de B. Por exemplo, 876 detona 345; porém, 651 não detona 542 pois 1 < 2. Quantos números de três algarismos detonam 314? A) 120 B) 240 C) 360 D) 480 E) 600 16. O relógio de parede indica inicialmente meio-dia. Os ponteiros das horas e dos minutos irão formar um ângulo de 90 graus pela primeira vez: A) entre 12h e 12h10min. B) entre 12h10min e 12h15min. C) entre 12h15min e 12h20min. D) entre 12h20min e 12h25min. E) após as 12h25min.

12 9

3 6

17. Eduardo escreveu todos os números de 1 a 2009 numa folha de papel. Com os amigos, combinou o seguinte: cada um deles poderia apagar quantos números quisesse e escrever, no fim da lista, o algarismo das unidades da soma dos números apagados. Por exemplo, se alguém apagasse os números 28, 3, 6, deveria escrever no fim da lista o número 7, pois 28 + 3 + 6 = 37. Após algum tempo, sobraram somente dois números. Se um deles era 2000, qual dos números a seguir poderia ser o outro? A) 0 B) 1 C) 3 D) 5 E) 6 18. Uma folha de caderno de Carlos é um retângulo com dois lados (bordas) amarelos de 24 cm e dois lados (bordas) vermelhos de 36 cm. Carlos pinta cada ponto do retângulo na mesma cor do lado mais próximo desse ponto. Qual é a área da região pintada de amarelo? A) 144 cm2 B) 288 cm2 C) 364 cm2 D) 442 cm2 E) 524 cm2 19. O professor Piraldo aplicou uma prova de 6 questões para 18 estudantes. Cada questão vale 0 ou 1 ponto; não há pontuações parciais. Após a prova, Piraldo elaborou uma tabela como a seguinte para organizar as notas, em que cada linha representa um estudante e cada coluna representa uma questão.

EUREKA! N°32, 2010

5


Sociedade Brasileira de Matemática

Questões→ Estudantes ↓ Arnaldo Bernaldo Cernaldo

1

2

3

4

5

6

0 1 0

1 1 1

1 1 1

1 0 1

1 0 1

0 1 0

Piraldo constatou que cada estudante acertou exatamente 4 questões e que cada questão teve a mesma quantidade m de acertos. Qual é o valor de m? A) 8 B) 9 C) 10 D) 12 E) 14

frente

frente

frente

E) frente

esquerda

D)

vista da frente

esquerda

C)

vista da esquerda

esquerda

B)

esquerda

A)

esquerda

20. Alguns cubos foram empilhados formando um bloco. As figuras ao lado representam a vista da esquerda e da frente desse bloco. Olhando o bloco de cima, qual das figuras a seguir não pode ser vista?

frente

PROBLEMAS – NÍVEL 2

1. Veja o Problema No. 1 do Nível 1. 2. Veja o Problema No. 9 do Nível 1. 3. Veja o problema No. 4 do Nível 1. 1 1 4. Se é: = 4, o valor de x+6 x+5 A)

1 5

B)

1 4

C)

2 3

D)

4 5

E) 1

5. Veja o Problema No. 6 do Nível 1. 6. Os inteiros positivos m e n satisfazem 15m = 20n. Então é possível afirmar, com certeza, que mn é múltiplo de: EUREKA! N°32, 2010

6


Sociedade Brasileira de Matemática

A) 5 B) 10 C) 12 7. Veja o problema No. 15 do Nível 1. 8. Veja o Problema No. 11 do Nível 1. 9. Veja o Problema No. 8 do Nível 1.

D) 15

E) 20

10. Na figura abaixo, α = 18 e AB = AC = AD = AE. O valor do ângulo β é:

A ααα

B β

A) 18o

B) 36o

C

E

D

C) 15o

D) 20o

E) 30o

11. Veja o Problema No. 10 do Nível 1. 12. Na figura abaixo, ABCDE é um pentágono regular, CDFG é um quadrado e DFH é um triângulo equilátero. O valor do ângulo β é: H

F

β

G D

E

C A

A) 30o

B) 36o

B

C) 39o

D) 45o

EUREKA! N°32, 2010

7

E) 60o


Sociedade Brasileira de Matemática

13. Veja o problema No. 12 do Nível 1. 14. Veja o Problema No. 13 do Nível 1. 15. A famosa Conjectura de Goldbach diz que todo número inteiro par maior que 2 pode ser escrito como a soma de dois números primos. Por exemplo, 18 pode ser representado por 5 + 13 ou, ainda, por 7 + 11. Considerando todas as possíveis representações de 126, qual a maior diferença entre os dois primos que a formam? B) 100 C) 92 D) 88 E) 80 A) 112 16. Na figura ao lado, E é o ponto médio de AB, F é o ponto médio de AC e BR = RS = SC. Se a área do triângulo ABC é 252, qual é a área do pentágono AERSF? A) 168 B) 189 C) 200 D) 210 E) 220

A

F

E

B

R

S

C

17. Quantos pares ordenados (x, y) de números reais satisfazem a equação

( x − y ) + ( x − y − 2) 2 2

A) 0

B) 1

C) 2

2

= 0?

D) 3

E) infinitos

18. Veja o Problema No. 19 do Nível 1. 19. Entre os inteiros positivos n + 4018, n = 1, 2,..., 20092 , quantos são quadrados perfeitos? A) 1945 B) 1946 C) 1947 D) 1948 E) 1949 20. Para cada número natural n, seja S n a soma dos dez primeiros múltiplos positivos de n. Por exemplo, S 2 = 2 + 4 + 6 + 8 + 10 + 12 + 14 + 16 + 18 + 20. Quanto é S1 + S 2 + S 3 + + S10 ? A) 2925 B) 3025 C) 3125 D) 3225 E) 3325

EUREKA! N°32, 2010

8


Sociedade Brasileira de Matemática

21. Em uma folha quadriculada em que cada quadrado tem lado 2cm, são desenhados dois círculos como na figura ao lado. A distância mínima entre os dois círculos mede: A) 3cm B) 10 cm

( D) ( E) ( C)

) 10 − 2 ) cm 10 − 3) cm 10 + 3 cm

22. Quantos números naturais de 1 a 100, inclusive, podem ser escritos na forma de potência a b , com a, b ∈ e a, b > 1? A) 10 B) 12 C) 14 D) 16 E) 18 23. Veja o Problema No. 18 do Nível 1. 24. Os inteiros 0 < x < y < z < w < t são tais que w = z(x + y) e t = w(y + z). Sendo w = 9, então t é igual a A) 45 B) 54 C) 63 D) 72 E) 81 25. Veja o Problema No. 20 do Nível 1. PROBLEMAS – NÍVEL 3

1. Veja o problema No. 15 do Nível 1. 2. Veja o problema No. 6 do Nível 2. 3. Se x2 = x + 3 então x3 é igual a: A) x2 + 3 B) x + 4 C) 2x + 2

D) 4x + 3

E) x2 – 2

4. Na figura, o quadrado A’B’C’D’ foi obtido a partir de uma rotação no sentido horário do quadrado ABCD de 25 graus em torno do ponto médio de AB. Qual é o ângulo agudo, em graus, entre as retas AC e B’D’?

EUREKA! N°32, 2010

9


Sociedade Brasileira de Matemática

D’

D

C C’

A’ B

A

B’ A) 5

B) 25

C) 45

D) 65

E) 85

5. Um dos cinco números a seguir é divisor da soma dos outros quatro. Qual é esse número? A) 20 B) 24 C) 28 D) 38 E) 42 6. Sempre que Agilulfo volta para casa depois da escola com uma advertência, se sua mãe está em casa, ela o coloca de castigo. Sabendo-se que ontem à tarde Agilulfo não foi colocado de castigo, qual das seguintes afirmações é certamente verdadeira? A) Agilulfo recebeu advertência ontem. B) Agilulfo não recebeu advertência ontem. C) Ontem à tarde a sua mãe estava em casa. D) Ontem à tarde a sua mãe não estava em casa. E) Nenhuma das afirmações acima é certamente verdadeira. 7. Qual é o menor valor de n > 1 para o qual é possível colocar n peças sobre um tabuleiro n × n de modo que não haja duas peças sobre a mesma linha, mesma coluna ou mesma diagonal? As figuras a seguir mostram pares de peças na mesma linha, na mesma coluna e na mesma diagonal em diversos tabuleiros. •

• • •

• •

A) 3

B) 4

C) 5

D) 6

EUREKA! N°32, 2010

10

E) 7


Sociedade Brasileira de Matemática

8. Na figura a seguir, ABCD é um quadrado de lado 4, K pertence ao lado AD, L pertence ao lado AB, M pertence ao lado BC e KLM é um triângulo retângulo isósceles, sendo L o ângulo reto. Então a área do quadrilátero CDKM é igual a A) 6 B) 8 C) 10 D) 12 E) 14

A

L

B

K

M D

C

9. Veja o Problema No. 6 do Nível 1. 10. Veja o Problema No. 16 do Nível 1. 11. Considere o número inteiro positivo n tal que o número de divisores positivos do dobro de n é igual ao dobro do número de divisores positivos de n. Podemos concluir que n é A) um número primo B) um número par C) um número ímpar D) um quadrado perfeito E) potência inteira de 2 12. Esmeralda tem cinco livros sobre heráldica em uma estante. No final de semana, ela limpou a estante e, ao recolocar os livros, colocou dois deles no lugar onde estavam antes e os demais em lugares diferentes de onde estavam. De quantas maneiras ela pode ter feito isso? A) 20 B) 25 C) 30 D) 34 E) 45 13. Veja o Problema No. 19 do Nível 1. 14. Seja f : → uma função tal que f(0) = 0, f(1) = 1, f(2) = 2 e f(x + 12) = f(x + 21) = f(x) para todo x ∈ . Então f(2009) é: A) 0 B) 1 C) 2 D) 3 E) 2009

EUREKA! N°32, 2010

11


Sociedade Brasileira de Matemática

15. Na figura, CD = BC, ∠BAD = 72 , AB é o diâmetro e O o centro do semicírculo. Determine a medida do ângulo ∠DEC. A) 36o B) 42o C) 54o D) 63o E) 18o

D

C E

O

A

B

16. Sabe-se que 2x2 – 12xy + ky2 ≥ 0 para todos x, y reais. O menor valor real de k é A) 9 B) 16 C) 18 D) 27 E) 36 17. Veja o problema No. 15 do Nível 2. 18. Um subconjunto de {1,2,3,…,20} é superpar quando quaisquer dois de seus elementos têm produto par. A maior quantidade de elementos de um subconjunto superpar é: A) 3 B) 4 C) 6 D) 7 E) 11 19. Veja o problema No. 20 do Nível 2. 20. Os círculos C1 e C2, de raios 3 e 4, respectivamente, são tangentes externamente em T. As tangentes externas comuns tocam C1 em P e Q e C2 em R e S. A tangente interna comum em T corta as tangentes externas nos pontos M e N, como mostra a figura. A razão entre as áreas dos quadriláteros MNPQ e MNRS é: A)

1 7

B)

9 16

C)

P C1

T Q

3 4

D)

EUREKA! N°32, 2010

12

R

N

M 3 2

C2 S E)

13 15


Sociedade Brasileira de Matemática

21. Dois carros deixam simultaneamente as cidades A e B indo de uma cidade em direção à outra, com velocidades constantes, e em sentidos opostos. As duas cidades são ligadas por uma estrada reta. Quando o carro mais rápido chega ao ponto médio M de AB, a distância entre os dois carros é de 96 km. Quando o carro mais lento chega ao ponto M, os carros estão a 160 km um do outro. Qual a distância, em km, entre as duas cidades? A) 320 B) 420 C) 480 D) 520 E) 560 8

22. Seja N = 8 8 , em que aparecem 2009 números 8. Agilulfo ficou de castigo: ele deve escrever a soma dos dígitos de N, obtendo um número M; em seguida, deve calcular a soma dos dígitos de M; e deve repetir o procedimento até obter um número de um único dígito. Vamos ajudar Agilulfo: esse dígito é A) 1 B) 2 C) 3 D) 7 E) 8 23. Veja o Problema No. 20 do Nível 1. 24. Veja o Problema No. 18 do Nível 1. 25. Os lados de um triângulo formam uma progressão aritmética de razão t. Então a distância entre o incentro e o baricentro deste triângulo é: A) t

B)

GABARITO

t 2

C)

t 3

D)

NÍVEL 1 – (6º. ou 7º. Anos) 1) C 6) C 2) B 7) A 3) D 8) C 4) C 9) C 5) D 10) B

11) B 12) E 13) E 14) E 15) B

16) C 17) D 18) B 19) D 20) C

NÍVEL 2 – (8º. ou 9º. Anos) 1) C 6) C 2) C 7) B 3) C 8) B 4) D 9) C 5) C 10) A

11) B 12) C 13) E 14) E 15) B

16) A 17) C 18) D 19) B 20) B

NÍVEL 3 – (Ensino Médio) 1) B 6) E 2) C 7) B 3) D 8) B 4) D 9) C 5) D 10) E

11) C 12) A 13) D 14) C 15) C

16) C 17) B 18) E 19) B 20) E

EUREKA! N°32, 2010

13

2t 3

E) faltam dados

21) E 22) B 23) B 24) A 25) C 21) C 22) A 23) C 24) B 25) C


Sociedade Brasileira de Matemática

XXXI OLIMPÍADA BRASILEIRA DE MATEMÁTICA Problemas e soluções da Segunda Fase PROBLEMAS – NÍVEL 1 – PARTE A (Cada problema vale 5 pontos) 01. A figura ao lado mostra

castelos de cartas de 1, 2 e 3 andares. Para montar esses castelos, foram usadas 2, 7 e 15 cartas, respectivamente. Quantas cartas serão necessárias para montar um castelo de 5 andares? 02. Numa classe do 6º ano, de cada 11 estudantes, 4 são meninas. Se há 15 meninos

a mais que meninas, quantos alunos há na classe? 03. Num curso com duração de cinco dias, a frequência dos alunos foi registrada na tabela abaixo: Dia de aula Quantidade de alunos presentes

1º dia

2º dia

3º dia

4º dia

5º dia

271

296

325

380

168

Cada aluno faltou exatamente dois dias. No dia de menor frequência, de quantos por cento foi o total de faltas? 04. Mariazinha deseja cobrir o tampo de uma mesa retangular de 88 cm por 95 cm colando quadrados de cartolina de lado 10 cm, a partir de um canto, como mostrado na figura. Ela cola os quadrados sem buracos nem superposições, até chegar às bordas opostas. Aí, em vez de cortar as folhas para não ultrapassar as bordas, ela as sobrepõe, formando regiões retangulares com duas folhas de espessura (região cinza) e uma pequena região retangular com quatro folhas de espessura (região preta). Qual é a área da região coberta por quatro folhas? EUREKA! N°32, 2010

14


Sociedade Brasileira de Matemática

05. O número 200920092009... 2009 tem 2008 algarismos. Qual é a menor

quantidade de algarismos que devem ser apagados, de modo que a soma dos algarismos que restarem seja 2008? 06. Dizemos que dois ou mais números, com a mesma quantidade de algarismos, são membros da mesma família, quando todos possuem pelo menos um algarismo comum. Por exemplo, os números 72, 32, 25 e 22 pertencem à mesma família, pois todos possuem o algarismo 2, enquanto que os números 123, 245 e 568 não pertencem à mesma família, pois não há um algarismo que apareça nesses três números. Qual é a maior quantidade de membros de uma família, cujos elementos têm três algarismos?

PROBLEMAS – NÍVEL 1 – PARTE B (Cada problema vale 10 pontos) PROBLEMA 1

Carlinhos tem folhas iguais na forma de triângulos retângulos de lados 6 cm, 8 cm e 10 cm. Em cada triângulo, o ângulo assinalado opõe-se ao menor lado. Fazendo coincidir lados iguais desses triângulos sobre uma mesa, sem superpor as folhas, ele desenha o contorno de cada figura obtida (linha grossa), como nos exemplos ao lado. O perímetro de uma figura é o comprimento do seu contorno. a) Qual é a diferença entre os perímetros das figuras 1 e 2 do exemplo? b) Com figuras de três triângulos, qual é o maior perímetro que pode ser obtido? PROBLEMA 2

Esmeralda ia multiplicar um número A de três algarismos por outro número B de dois algarismos, mas na hora de multiplicar inverteu a ordem dos dígitos de B e obteve um resultado 2034 unidades maior. a) Qual era o número A, se os dígitos de B eram consecutivos? b) Qual seria o número A, se os dígitos de B não fossem consecutivos?

EUREKA! N°32, 2010

15


Sociedade Brasileira de Matemática

PROBLEMA 3

Um campeonato de xadrez de 7 rodadas, com 4 jogos por rodada, tem 8 participantes, cujas pontuações por jogo são as usuais: um ponto por vitória, meio ponto por empate e nenhum ponto por derrota. Cada par de jogadores se enfrenta exatamente uma vez. a) Ao término da terceira rodada, é possível que um grupo de jogadores esteja em primeiro lugar e o restante dos jogadores esteja em segundo lugar? Explique por meio de um exemplo. b) Ao término da terceira rodada, é possível que todos os jogadores tenham pontuações diferentes? Explique. PROBLEMAS – NÍVEL 2 – PARTE A (Cada problema vale 5 pontos) 01. Esmeralda tem uma garrafa com 9 litros de uma mistura que tem 50% de álcool e 50% de água. Ela quer colocar água na garrafa de tal forma que apenas 30% da mistura seja de álcool. Quantos litros de água ela irá colocar? 02. Se a, b, c e d são, em alguma ordem, 1, 2, 3 e 4. Qual é o maior valor possível

de ab + bc + cd + da? 03. Dizemos que dois ou mais números, com a mesma quantidade de algarismos, são membros da mesma família, quando todos possuem pelo menos um algarismo em comum. Por exemplo, os números 32, 25 e 22 pertencem à mesma família, enquanto que 123, 245 e 568 não pertencem à mesma família, pois 123 e 568 não pertencem à mesma família. Qual é a maior quantidade de membros de uma família, cujos elementos têm três algarismos? 04. Determine a quantidade de inteiros de dois algarismos que são divisíveis pelos seus algarismos. 05. Na figura abaixo, ABCD e EFGH são quadrados de lado 48 cm. Sabendo que A

é o ponto médio de EF e G é o ponto médio de DC, determine a área destacada em cm2.

EUREKA! N°32, 2010

16


Sociedade Brasileira de Matemática

E L B

A

H

F K D

C

G

PROBLEMAS – NÍVEL 2 – PARTE B (Cada problema vale 10 pontos) PROBLEMA 1

Sejam m e n dois inteiros positivos primos entre si. O Teorema Chinês dos Restos afirma que, dados inteiros i e j com 0 ≤ i < m e 0 ≤ j < n, existe exatamente um inteiro a, com 0 ≤ a < m⋅n, tal que o resto da divisão de a por m é igual a i e o resto da divisão de a por n é igual a j. Por exemplo, para m = 3 e n = 7, temos que 19 é o único número que deixa restos 1 e 5 quando dividido por 3 e 7, respectivamente. Assim, na tabela a seguir, cada número de 0 a 20 aparecerá exatamente uma vez. Restos por 7

0

1

2

3

4

5

6

Restos por 3

0 19

1 2

Qual a soma dos números das casas destacadas? PROBLEMA 2

Observe:

(x – r)(x – s) = x2 – (r + s)x + rs

Assim, substituindo x por r e por s, obtemos

r 2 − (r + s ) r + rs = 0 s 2 − (r + s ) s + rs = 0

a ( r n + 2 − (r + s ) r n +1 + rs ⋅ r n ) = 0 b( s n + 2 − ( r + s ) s n +1 + rs ⋅ s n ) = 0

EUREKA! N°32, 2010

17


Sociedade Brasileira de Matemática

Somando as duas equações e sendo S n = a ⋅ r n + b ⋅ s n , verifica-se que

S n + 2 = (r + s ) S n +1 − rsS n S1 = ar + bs = 1 ,

Dados 4

S 2 = ar 2 + bs 2 = 2 ,

4

5

S 3 = ar 3 + bs 3 = 5

e

5

S 4 = ar + bs = 6 , determine S 5 = ar + bs . PROBLEMA 3

Seja N é o ponto do lado AC do triângulo ABC tal que AN = 2 NC e M o ponto do lado AB tal que MN é perpendicular a AB . Sabendo que AC = 12 cm e que o baricentro G do triângulo ABC pertence ao segmento MN, determine o comprimento do segmento BG. OBS: Baricentro é o ponto de interseção das medianas do triângulo. PROBLEMA 4

Um campeonato de xadrez de 7 rodadas, com 4 jogos por rodada, tem 8 participantes, cujas pontuações por jogo são as usuais: um ponto por vitória, meio ponto por empate e nenhum ponto por derrota. Cada par de jogadores se enfrenta exatamente uma vez. a) Ao término da terceira rodada, é possível que todos os jogadores tenham pontuações distintas? b) Se no final do campeonato todos os jogadores têm pontuações distintas qual o menor número possível de pontos obtidos pelo primeiro colocado? PROBLEMAS – NÍVEL 3 – PARTE A (Cada problema vale 5 pontos) 01. Veja o problema No. 1 do Nível 2. 02. No triângulo retângulo ABC, ∠A = 90º, AB = 5cm e BC = 9cm. Se I é o incentro de ABC, determine o comprimento do segmento CI. 03. Seja c a maior constante real para a qual

x2 + 3y2 ≥ c⋅(x2 + xy + 4y2). para todos x, y reais. Determine o inteiro mais próximo de 2009⋅c. EUREKA! N°32, 2010

18


Sociedade Brasileira de Matemática

04. No programa de auditório Toto Bola, o apresentador Ciço Magallanes dispõe de

duas caixas idênticas. Um voluntário da platéia é chamado a participar da seguinte brincadeira: ele recebe dez bolas verdes e dez bolas vermelhas e as distribui nas duas caixas, sem que o apresentador veja, e de modo que em cada caixa haja pelo menos uma bola. Em seguida, o apresentador escolhe uma das caixas e retira uma bola. Se a bola for VERDE, o voluntário ganha um carro. Se for VERMELHA, ele ganha uma banana. A máxima probabilidade que o voluntário tem de ganhar um m , em que m e n são inteiros positivos primos entre si. Determine o carro é igual a n valor de m + n. 05. Determine o maior inteiro n menor que 10000 tal que 2n + n seja divisível por 5. PROBLEMAS – NÍVEL 3 – PARTE B (Cada problema vale 10 pontos) PROBLEMA 1

Determine a quantidade de números n = a1a2a3a4a5a6, de seis algarismos distintos, que podemos formar utilizando os algarismos 1, 2, 3, 4, 5, 6, 7, 8, 9 de modo que as seguintes condições sejam satisfeitas simultaneamente: i) a1 + a6 = a2 + a5 = a3 + a4; ii) n é divisível por 9. PROBLEMA 2

Encontre todos os inteiros a > 0 e b > 0 tais que 4 ⋅ 3 a = 11 + 5 b PROBLEMA 3

Para cada inteiro positivo n, seja An = {x ∈ R+ ; x ⋅ x  = n} , em que R+ é o conjunto dos reais positivos e x  é o maior inteiro menor ou igual a x. Determine a quantidade de elementos do conjunto A1 ∪ A2 ∪ A3 ∪ ... ∪ A2009. PROBLEMA 4

No triângulo ABC, temos ∠A = 120° e BC = 12 cm. A circunferência inscrita em ABC tangencia os lados AB e AC, respectivamente, nos pontos D e E. Sejam K e L os pontos onde a reta DE intersecta a circunferência de diâmetro BC. Determine a distância entre os pontos médios dos segmentos BC e KL.

EUREKA! N°32, 2010

19


Sociedade Brasileira de Matemática

SOLUÇÕES NÍVEL 1 – SEGUNDA FASE – PARTE A

Problema Resposta

01 40

02 55

03 65

04 10

05 392

06 252

01. Para fazer um novo andar num castelo já construído, precisamos de três cartas para cada andar anterior mais duas para o topo. Assim, a partir do castelo de 3 andares, para fazer o de 4 andares, precisamos de mais 3 × 3 + 2 = 11 cartas, num total de 15 + 11 = 26 cartas. Portanto, para fazer o castelo de 5 andares, precisamos de 26 + 4 × 3 + 2 = 40 cartas.

Solução alternativa: Para acrescentarmos um quarto andar a um castelo de 3 andares, precisamos de 3 cartas para separar a base dos demais andares e 4 pares de cartas para a base, totalizando 3 + 2.4 = 11 cartas a mais. Veja a figura a seguir:

Analogamente, para acrescentarmos um quinto andar a um castelo de 4 andares, precisamos de 4 cartas para separar a base dos demais andares e 5 pares de cartas para a base, totalizando 4 + 2.5 = 14 cartas a mais. Assim, para montar um castelo de 5 andares, precisamos de 15 + 11 + 14 = 40 cartas. Observação: De fato, o acréscimo de um n-ésimo andar necessita de n − 1 cartas para apoiar a base anterior, e n pares de cartas para a nova base. Portanto, são acrescentadas n − 1 + 2 ⋅ n = 3n − 1 cartas por andar. 02. Seja x a quantidade de meninas. Assim, a quantidade de meninos é x + 15 e a quantidade total de alunos será 2 x + 15 . Fazendo a proporção, temos:

x 4 = 2 x + 15 11 Resolvendo a equação, obtemos x = 20 . EUREKA! N°32, 2010

20


Sociedade Brasileira de Matemática

03. Se cada aluno compareceu exatamente três dias, o número total de alunos do

curso é

271 + 296 + 325 + 380 + 168 1440 = = 480 . A menor frequência foi de 3 3

168 alunos, num total de 480 – 168 = 312 faltas. Portanto, o percentual de faltas nesse dia foi

312 = 0, 65 = 65% . 480

04. Na direção da medida 88 cm, Mariazinha irá usar 9 folhas e na direção da medida 95 cm, irá usar 10 folhas. Mariazinha começa colando as folhas sem sobreposição da esquerda para a direita e de cima para baixo (como na figura) e ao chegar às bordas direita e inferior, desloca, respectivamente, 2 cm à esquerda e 5 cm para cima (as regiões em cinza representam as sobreposições de 2 folhas). A região retangular preta é a intersecção dessas duas faixas de sobreposição, logo é coberta por 4 folhas. Sua área é de 10 cm2.

05. No número existem 502 algarismos 2 e 502 algarismos 9. Para retirar a menor

quantidade possível de algarismos, devemos tentar deixar a maior quantidade possível de algarismos 2. Porém, a soma de todos os algarismos 2 é 1004. Ainda falta 1004 para completar a soma 2008. Como 1004 = 9 × 111 + 5 devemos deixar pelo menos 111 algarismos 9. Porém, é impossível deixar exatamente 111 algarismos 9. Se deixarmos 112 algarismos 9, devemos deixar 500 algarismos 2. Portanto, deve-se retirar no mínimo 2 + 390 = 392 algarismos. 06. Como todos os membros de uma família devem possuir pelo menos um

algarismo comum, a maior quantidade de membros de uma família cujos elementos têm três algarismos é igual ao número de elementos de qualquer conjunto formado EUREKA! N°32, 2010

21


Sociedade Brasileira de Matemática

por todos os números de três algarismos que possuem um determinado algarismo em sua representação decimal. O algarismo das centenas não pode ser zero. Vamos contar então todos os números que têm um determinado algarismo a, não nulo, pois há mais deles. Há 9 × 9 = 81 números em que a aparece uma única vez, como algarismo das centenas. Há 8 × 9 = 72 números em que a aparece uma única vez, como algarismo das dezenas (lembre-se que o das centenas não pode ser 0) e há 72 números em que o a aparece uma única vez, como algarismo das unidades. Há 9 números com a na centena e na dezena, menos na unidade, 9 números com a na centena e na unidade, menos na dezena e 8 números com a na dezena e na unidade, menos na centena e um único número formado inteiramente de a. A quantidade total de números em que figura o algarismo não nulo a é 81 + 72 + 72 + 9 + 9 + 8 + 1 = 252.

Solução alternativa: Para simplificar o raciocínio, vamos contar quantos números de três algarismos não contêm um algarismo a, não nulo, fixado. Assim, nessa situação, existem 8 escolhas para o algarismo das centenas (não pode ser 0 ou a), 9 escolhas para o algarismo das dezenas (não pode ser a), e 9 escolhas para os algarismos das unidades (não pode ser a). Logo, pelo Princípio Fundamental da Contagem, há 8.9.9 = 648 números que não possuem o algarismo a. Assim, como existem 900 números de 3 algarismos, há 900 – 648 = 252 números que possuem o algarismo a ( a ≠ 0 ). Essa é a maior quantidade de membros que uma família pode ter. Observação: Podemos verificar que a família formada por todos os números de três algarismos que possuem o zero tem 900 − 9 ⋅ 9 ⋅ 9 = 171 membros. SOLUÇÕES NÍVEL 1 – SEGUNDA FASE – PARTE B PROBLEMA 1

EUREKA! N°32, 2010

22


Sociedade Brasileira de Matemática

a) O perímetro da primeira figura é 8 + 6 + 6 + 10 + 6 = 36 e da segunda figura é 10 + 8 + 6 + 8 + 8 = 40 . Portanto a diferença é 40 − 36 = 4 . b) A figura de maior perímetro é obtida quando fazemos coincidir os dois menores lados de cada um dos triângulos. Isso é mostrado na figura ao lado cujo perímetro é 10 + 10 + 10 + 8 + 6 = 44 (há outras com o mesmo perímetro). PROBLEMA 2

Seja A o número de três dígitos e B = 10 x + y o número de dois dígitos. Portanto, ao trocar a ordem dos dígitos de B , obtemos o número 10 y + x . Montando a equação segundo as condições do problema, temos:

A(10 x + y ) − A(10 y + x) = 9 A( x − y ) = 2034

Com isso,

A( x − y ) = 226 = 2 ⋅113 Daí, se x, y são consecutivos, A = 226 , caso contrário A = 113 . PROBLEMA 3

a) Sim, é possível. Por exemplo (há outros), podem existir quatro jogadores com pontuação 2 e outros quatro com pontuação 1. Fazendo A, B, C, D o primeiro grupo e E, F, G, H o segundo grupo, temos:

1ª Rodada A vence E B vence F C vence G D vence H 2ª Rodada A empata com B E empata com F C empata com D G empata com H 3ª Rodada A empata com F B empata com E C empata com H D empata com G EUREKA! N°32, 2010

23


Sociedade Brasileira de Matemática

b) Após três rodadas, um jogador pode acumular no máximo 3 pontos. Como as pontuações são múltiplos inteiros de

1 2

3 2

1 , os possíveis valores de pontuação após a 2

5 2

terceira rodada são: 0, ,1, , 2, ,3 (7 resultados possíveis) Como existem 8 jogadores e apenas 7 possibilidades, dois jogadores terão pontuações iguais. SOLUÇÕES NÍVEL 2 – SEGUNDA FASE – PARTE A

Problema Resposta

01 06

02 25

03 252

04 14

05 1704

01. Inicialmente temos 4,5 litros de água e 4,5 litros de álcool. Colocados x litros de

água, para termos 30% de álcool na mistura, basta que

x = 6. 02.

É fácil ver que

30 (9 + x) = 4,5 , então 100

ab + bc + cd + da = b ( a + c ) + b ( c + a ) = ( a + c )( b + d ) .

Suponha sem perda de generalidade que a = 1. . Com isso, {a,c} = {1, 2} ,{1,3} ou

{1,4} e conseqüentemente {b,d } = {3,4} ,{2,4} ou {2,3} , respectivamente. Assim os possíveis valores do produto são 21, 24 e 25 e o máximo é 25. 03. O algarismo das centenas não pode ser zero. Vamos contar então todos os

números que têm um determinado algarismo x, não nulo, pois há mais deles. Há 9 × 9 = 81 números em que x aparece uma única vez, como algarismo das centenas. Há 8 × 9 = 72 números em que x aparece uma única vez, como algarismo das dezenas (lembre-se que o das centenas não pode ser 0) e há 72 números em que o x aparece uma única vez, como algarismo das unidades. Há 9 números com x na centena e na dezena, menos na unidade, 9 números com x na centena e na unidade, menos na dezena e 8 números com x na dezena e na unidade, menos na centena e um único número formado inteiramente de x. A quantidade total de números em que figura o algarismo não nulo x é 81 + 72 + 72 + 9 + 9 + 8 + 1 = 252

EUREKA! N°32, 2010

24


Sociedade Brasileira de Matemática

04. Seja n = 10 A + B o número de dois dígitos. Se A divide n , então

B . Se A > 5 , então B = A , pois B não pode ser 0 e B < 10 < 2 A .

A divide

Listemos as possibilidades: Se A = 1 então AB pode ser 11, 12, 15. Se A = 2 , então AB pode ser 22, 24. Se A = 3 , então AB pode ser 33, 36. Se A = 4 , então AB pode ser 44, 48. Se A = 5 , então AB pode ser 55. Se A = 6 , então AB pode ser 66. Se A = 7 , então AB pode ser 77. Se A = 8 , então AB pode ser 88. Se A = 9 , então AB pode ser 99. Logo, o total de números é 3 + 2 + 2 + 2 + 5 = 14. 05. Sejam K a interseção dos lados AD e FG , e L a interseção dos lados AB e

EH . Por simetria, veja que KD = KF e AK = KG . Considere FK = x . Dessa forma, AK = 48 − x . Usando teorema de Pitágoras no triângulo AFK , temos:

242 + x 2 = ( 48 − x ) . 2

Que nos dá x = 18 . Agora, veja que os triângulos AFK e ALE são semelhantes. Portanto, AE EL . = FK AF Assim, EL = 32 . Para achar a área procurada, basta subtrair a área do quadrado EFGH das áreas dos triângulos AFK e AEL . Portanto a área será 1704. E

L B

A

H

F K D

G

EUREKA! N°32, 2010

25

C


Sociedade Brasileira de Matemática

SOLUÇÕES NÍVEL 2 – SEGUNDA FASE – PARTE B PROBLEMA 1:

0

1

2

3

4

5

6

0

0

15

9

3

18

12

6

1

7

1

16

10

4

19

13

2

14

8

2

17

11

5

20

A resposta é 15 + 8 + 10 + 11 + 12 + 13 = 69. PROBLEMA 2:

S 4 = (r + s ) S 3 − rsS 2 = (r + s ).5 − rs.2 = 5r + 5s − 2rs = 6 S 3 = (r + s ) S 2 − rsS1 = (r + s ).2 − rs.1 = 2r + 2 s − rs = 5 Com isso, encontramos que r + s = −4 e rs = −13 . Daí, S5 = (r + s)S 4 − rsS3 = −24 + 65 = 41. PROBLEMA 3:

Se BP é uma mediana do triângulo então AP = CP = 6 e PN = 2. Como G é o PG 1 PN 1 baricentro do triângulo então e = = , assim, pela recíproca do GB 2 NC 2 teorema de Tales, GN é paralelo a BC e ∠B = 90o . Como o triângulo ABC é retângulo então AP = CP = BP = 6. Com isso, BG = 4 e GP = 2.

EUREKA! N°32, 2010

26


Sociedade Brasileira de Matemática

PROBLEMA 4:

a) Após três rodadas, um jogador pode acumular no máximo 3 pontos. Como as pontuações são múltiplos inteiros de ½ , os possíveis valores de pontuação após a terceira rodada são: 0,1/2, 1, 3/2, 2, 5/2, 3 Como existem 8 jogadores e apenas 7 possibilidades, dois jogadores terão pontuações iguais. b) Se k é a pontuação do primeiro colocado e todas as pontuações são distintas, a soma das pontuações dos oito jogadores será no máximo:

1 3 5 7     k +  k −  + ( k − 1) +  k −  + ( k − 2 ) +  k −  + ( k − 3) +  k −  = 8k − 14 2 2 2 2     Como foram disputados exatamente 4 × 7 = 28 pontos, temos

8k − 14 ≥ 28 1 1 pois as pontuações são múltiplos inteiros de . Basta mostrarmos 2 2 um exemplo onde este valor é atingido.

Logo, k ≥ 5 +

Na tabela abaixo, marcamos na interseção da linha Ai com a coluna A j o número de pontos que Ai ganhou na partida disputada contra Aj.

A1 A2 A3 A4 A5 A6 A7 A8 Total

A1 X

1

1

1

1

1

½

0

5+½

A2 0

x

1

1

1

1

1

0

5

A3 0

0

x

1

1

1

1

½

4+½

A4 0

0

0

X

1

1

1

1

4

A5 0

0

0

0

X

0

0

0

0

A6 0

0

0

0

1

X

½

1

2+½

A7 ½

0

0

0

1

½

x

1

3

A8 1

1

½

0

1

0

0

x

3+½

EUREKA! N°32, 2010

27


Sociedade Brasileira de Matemática

SOLUÇÕES NÍVEL 3 – SEGUNDA FASE – PARTE A

Problema Resposta

01 0069

02 0006

03 1339

04 0033

05 9993

01. [RESPOSTA: 0069] SOLUÇÃO:

0

1

2

3

4

5

6

0

0

15

9

3

18

12

6

1

7

1

16

10

4

19

13

2

14

8

2

17

11

5

20

A resposta é 15 + 8 + 10 + 11 + 12 + 13 = 69. 02. [RESPOSTA: 0006] SOLUÇÃO: Pelo teorema de Pitágoras, é imediato que

AC 2 = 9 2 − 5 2 = 56 ∴ AC = 2 14 . Seja r o raio do círculo inscrito, como mostrado na figura abaixo. B 5–r

9

5–r 5

2 14 − r

I r

r A

r

2 14 − r

C

Como os comprimentos das tangentes ao círculo inscrito partindo de cada vértice são iguais, ficamos com a equação (5 – r) + (2 14 − r ) = 9, de onde obtemos r = 14 − 2 . Novamente pelo teorema de Pitágoras, obtemos:

EUREKA! N°32, 2010

28


Sociedade Brasileira de Matemática

CI 2 = r 2 + ( 2 14 − r ) 2 = ( 14 − 2) 2 + ( 14 + 2) 2 = 36 ∴ CI = 6 . 03. [RESPOSTA: 1339] SOLUÇÃO: Fazendo x = t⋅y, a equação inicial reduz-se a

t2 + 3 ≥ c⋅(t2 + t + 4). Logo, devemos ter (c – 1)t2 + ct + (4c – 3) ≤ 0, para todo t real. Para isto, devemos ter c – 1 < 0 e o discriminante ∆ = c2 − 4⋅(c – 1)⋅(4c – 3) ≤ 0. Da última inequação, obtemos −15c2 + 28c – 12 ≤ 0, cuja solução é c ≤

c≥

2 ou 3

6 . Como c < 1, o maior valor possível de c é 2/3. Daí, 2009⋅c = 1339,333... . 5

04. [RESPOSTA: 0033] SOLUÇÃO: Seja P(a, b) a probabilidade de o voluntário ganhar o carro no caso em

que ele tenha colocado a bolas VERDES e b bolas VERMELHAS na caixa 1. Então, necessariamente haverá (10 – a) bolas VERDES e (10 – b) bolas VERMELHAS na caixa 2. Segue que P ( a, b ) =

a 1 1 10 − a ⋅ + ⋅ . 2 a + b 2 20 − a − b

Podemos supor, sem perda de generalidade, que a + b ≤ 10, já que as caixas são idênticas. Suponha, ainda, que haja alguma bola VERMELHA na caixa 1. Vejamos o que acontece com essa probabilidade se transferirmos uma bola VERDE da caixa 2 para a caixa 1 e uma bola VERMELHA da caixa 1 para a caixa 2. Ficamos com P( a + 1, b − 1) =

1 a +1 1 9−a . ⋅ + ⋅ 2 a + b 2 20 − a − b

Dessa forma,

P( a + 1, b − 1) − P( a, b) =

1  1 1  ⋅ − ≥0, 2  a + b 20 − a − b 

pois a + b ≤ 10.

EUREKA! N°32, 2010

29


Sociedade Brasileira de Matemática

Assim, o voluntário sabe que, enquanto houver bola VERMELHA na caixa que contém menos bolas, a probabilidade pode ser aumentada, bastando, para isto, que ele troque uma das bolas VERMELHAS desta caixa com uma VERDE da outra. Por isso, para maximizarmos a probabilidade, basta considerarmos o caso em que a caixa 1 contém apenas bolas VERDES e a caixa 2 contém o restante das bolas. Teremos 1 1 10 − a P( a,0) = + ⋅ 2 2 20 − a . 1  10 − a  1  10  5 ⋅ = 1 +  =1−  = 2 − 2 20 − a  2  20 − a  20 − a Logo, a probabilidade será máxima quando a for mínimo. Como em cada caixa deve haver pelo menos uma bola, devemos ter a = 1. Neste caso, a probabilidade é: 5 14 P(1,0) = 1 − = . 19 19 Segue que m = 14, n = 19 e m + n = 33. 05. [RESPOSTA: 9993] SOLUÇÃO: Vamos analisar os restos das divisões de 2n e n por 5. n 2n 2n +n

1 1 2

2 2 4

3 3 3

4 4 1

5 0 2

6 1 4

7 2 3

8 3 1

9 4 2

10 0 4

11 1 3

12 2 1

13 3 2

14 4 4

15 0 3

16 1 1

17 2 2

18 3 4

19 4 3

20 0 1

3

1

1

0

2

0

0

4

1

4

4

3

0

3

3

2

4

2

2

1

Veja que os restos das divisões de 2n por 5 formam uma seqüência de período 4, enquanto que os restos das divisões de n por 5 formam uma seqüência de período 5. Logo, os restos das divisões de 2n + n formam uma seqüência de período 20, dada pela última linha da tabela acima. Dessa forma, tomando os números de 1 a 10000 em intervalos de tamanho 20, o maior n tal que 2n + n deixa resto zero na divisão por 5 é o 13o termo do ultimo intervalo, ou seja, o número 9980 + 13 = 9993. SOLUÇÕES NÍVEL 3 – SEGUNDA FASE – PARTE B PROBLEMA 1:

Seja k = a1 + a6 = a2 + a5 = a3 + a4. Temos 3k = a1 + a2 + ... + a6 é múltiplo de 9, uma vez que n é múltiplo de 9. Daí, segue que k é múltiplo de 3. Mas, como os algarismos são distintos, perceba que EUREKA! N°32, 2010

30


Sociedade Brasileira de Matemática

1 + 2 + ... + 6 ≤ a1 + a2 + ... + a6 ≤ 4 + 5 + ... + 9 ⇔ 21 ≤ 3k ≤ 39 ⇔ 7 ≤ k ≤ 13. Como k é múltiplo de 3, temos dois casos: k = 9 e k = 12.

1o caso: k = 9. Veja que é suficiente escolhermos a1, a2 e a3, pois a4 = 9 – a3, a5 = 9 – a2 e a6 = 9 – a1. Como os dígitos devem ser distintos, devemos escolher a1, a2 e a3 de modo que haja no máximo um dígito em cada um dos conjuntos {1, 8}, {2, 7}, {3, 6} e {4, 5}. Esta escolha pode ser feita da seguinte forma: • Escolhemos três dos quatro conjuntos: 4 maneiras; • Em cada um dos três conjuntos acima, escolhemos um dos dois dígitos: 23 = 8 maneiras; • Permutamos os dígitos escolhidos: 3! = 6 maneiras. Logo, o total de números, neste caso, é igual a 4×8×6 = 192.

2o caso: k = 12. Neste caso, os dígitos a1, a2 e a3 devem ser escolhidos do conjunto {3, 4, 5, 7, 8, 9} de modo que haja no máximo um dígito em cada um dos conjuntos {3, 9}, {4, 8} e {5, 7}. Esta escolha pode ser feita da seguinte maneira: • Em cada um dos três conjuntos acima, escolhemos um dos dois dígitos: 23 = 8 maneiras; • Permutamos os dígitos escolhidos: 3! = 6 maneiras. Logo, o total de números, neste caso, é igual a 8 × 6 = 48. O total de números é, portanto, 192 + 48 = 240. PROBLEMA 2:

Analisando a equação módulo 5, obtemos 4 ⋅ 3a ≡ 1( mod 5 ) ⇔ 3a ≡ 4 ( mod 5 ) . Mas os valores de 3a mod 5 são periódicos de período 4: 0 1 2 3 4 5 6 7 a 3ª mod 5 1 3 4 2 1 3 4 2 Assim, concluímos que 3a ≡ 4 ( mod 5 ) ⇔ a = 2 + 4t para t ∈ . Agora, analisando a equação módulo

3,

obtemos

11 + 5 ≡ 0 ( mod 3) ⇔ ( −1) ≡ 1( mod 3) o que ocorre se, e só se, b é par. Portanto a b

b

EUREKA! N°32, 2010

31


Sociedade Brasileira de Matemática

e b são ambos pares, digamos a= 2c e b = 2d para dois inteiros positivos c, d. Assim, 4 ⋅ 3a = 11 + 5b ⇔ ( 2 ⋅ 3c ) − 52 d = 11 2

⇔ ( 2 ⋅ 3c − 5d )( 2 ⋅ 3c + 5d ) = 11

⇔ ⇔

2 ⋅ 3c − 5d = 1 2 ⋅ 3c + 5d = 11 3c = 3 5d = 5

a = 2⋅c = 2 b = 2⋅d = 2

Assim, a única solução é: ( A,B ) = ( 2 , 2 ) PROBLEMA 3:

Vamos

fazer

o

gráfico

da

função

f ( x) = x ⋅ x  . Para cada k natural, se

k ≤ x ≤ k + 1 , temos

x  = k .

Logo, o gráfico

de f é formado por segmentos de reta y = k⋅x, como mostra a figura ao lado: Assim, para um n fixo, a equação f(x) = n tem no máximo uma solução. Portanto, a quantidade de elementos de A1 ∪ A2 ∪ A3 ∪ ... ∪ A2009 é igual à quantidade de inteiros n, tais que 1 ≤ n ≤ 2009, para os quais f (x) = n admite solução, isto é, os n tais que f (k) = k2 ≤ n < k(k + 1) = k2 + k, para algum k ∈ N.

6

4 2 1 1

2

3

PROBLEMA 4

Vamos mostrar inicialmente que BL e CK são as bissetrizes dos ângulos B e C do

∆ABC. Para isto, sejam K´ e L´ as intersecções das bissetrizes de C e B com a circunferência de diâmetro BC , como na figura. Seja ainda I o incentro de ∆ABC e β e γ as medidas de B e C , respectivamente, de modo que β + γ = 60°.

EUREKA! N°32, 2010

32


Sociedade Brasileira de Matemática

L´ 120º

γ /2

A

F β /2 β /2

B

G I

γ /2 γ /2

C

Sejam D´ e E´ as intersecções de K´ L´ com os lados AB e AC do triângulo. Para mostrar que K´ L´ = KL, basta mostrar que E´ e D´ são as projeções ortogonais de I aos lados AC e AB . Como BC é diâmetro, temos que BL´C é reto, assim se mostrarmos que o quadrilátero IE´ L´C é cíclico, provaremos que IE´C é reto, e analogamente para D´. Denote por F e G os encontros das bissetrizes de C e B com os lados opostos. γ β β+γ Temos m GIC = m F IB = m AFC − m F BI = β + − = = 30°. da 2 2 2 β γ β+γ mesma forma, temos m GE´L´ = m BGA − m ( GL´ E´ ) = + γ − = = 30° 2 2 2

) ( ) ( ) ( ) ( ) pois m ( GL´ E´ ) = m ( BL´K´ ) = m ( BCK´ ) já que ambos os ângulos subtendem o mesmo arco BK´. Assim, m ( GE´L´ ) = m ( GIC ) , provando que IE´L´C é cíclico. (

Sendo

(

)

) (

O

(

o

ponto

médio

) (

) m ( LOK ) Assim a distância pedida é LO ⋅ cos

de

BC,

m K OL = 180° − m LOC − m K OB = 180° − β − γ = 120°

2

EUREKA! N°32, 2010

33

=

BC ⋅ cos 60° = 3cm. 2

temos


Sociedade Brasileira de Matemática

XXXI OLIMPÍADA BRASILEIRA DE MATEMÁTICA Problemas e soluções da Terceira Fase TERCEIRA FASE – NÍVEL 1 PROBLEMA 1

A sequência 121, 1221, 12221, ... contém todos os números da forma 122 … 21 . A n dígitos 2

quantidade de dígitos 2 indica a posição do número na sequência. Por exemplo, o número 122222221 é o sétimo termo da sequência. a) Dentre os 2009 primeiros termos da sequência, quantos são divisíveis por 3? b) Qual é o menor número múltiplo de 1001 da sequência? PROBLEMA 2

O hexágono regular ABCDEF tem área de 12 cm2. a) Traçando segmentos a partir de um vértice, o hexágono ABCDEF foi repartido em 4 triângulos, conforme figura. Calcule as áreas desses triângulos.

b) Usando os quatro triângulos em que foi dividido o hexágono, podemos montar o retângulo PQRS, na figura. Qual é a área desse retângulo?

EUREKA! N°32, 2010

34


Sociedade Brasileira de Matemática

PROBLEMA 3

As casas de um tabuleiro 4 × 4 devem ser numeradas de 1 a 16, como mostrado parcialmente no desenho, formando um Quadrado Mágico, ou seja, as somas dos números de cada linha, de cada coluna e de cada uma das duas diagonais são iguais.

a) Que números devem ser escritos no lugar de X e de Y? b) Apresente o Quadrado Mágico completo na sua folha de respostas. PROBLEMA 4

Carlinhos tem várias peças formadas por quatro quadradinhos de lado unitário, na forma de L:

Ele forma figuras maiores com essas peças, fazendo coincidir um ou mais lados dos quadradinhos, como no exemplo, em que foram usadas duas dessas peças, fazendo coincidir um lado unitário. Não é permitido formar buracos nas figuras.

Permitido Não permitido a) Desenhe uma figura cujo perímetro é 14. b) Descreva como formar uma figura de perímetro 2010. c) É possível formar uma figura de perímetro ímpar? Justifique sua resposta. PROBLEMA 5

Um dominó é formado por 28 peças diferentes. Cada peça tem duas metades, sendo que cada metade tem de zero a seis pontos: EUREKA! N°32, 2010

35


Sociedade Brasileira de Matemática

Esmeralda coloca 4 peças de dominó dentro de um estojo, respeitando as regras do jogo, isto é, peças vizinhas se tocam em metades com as mesmas quantidades de pontos. Caso seja possível guardar as quatro peças no estojo, dizemos que o conjunto de quatro peças é precioso.

Por exemplo, a figura acima mostra as maneiras de guardar o conjunto precioso formado pelas peças , , , . a) Mostre que um conjunto precioso não pode conter duas peças duplas. A figura abaixo mostra as peças duplas.

b) Quantos conjuntos preciosos contêm uma peça dupla? c) Determine a quantidade total de conjuntos preciosos.

EUREKA! N°32, 2010

36


Sociedade Brasileira de Matemática

TERCEIRA FASE – NÍVEL 2 PRIMEIRO DIA PROBLEMA 1

Veja o problema No. 5 do Nível 1. PROBLEMA 2

Seja A um dos pontos de interseção de dois círculos com centros X e Y. As tangentes aos círculos em A intersectam novamente os círculos em B e C. Seja P o ponto de plano tal que PXAY é um paralelogramo. Prove que P é o circuncentro do triângulo ABC. PROBLEMA 3

Prove que não existem inteiros positivos x e y tais que x3 + y3 = 22009. SEGUNDO DIA PROBLEMA 4

Resolva, em números reais, o sistema

1 1 1 =y+ =z+ y z x xyz = 1.

x+

PROBLEMA 5

Uma formiga caminha no plano da seguinte maneira: inicialmente, ela anda 1cm em qualquer direção. Após, em cada passo, ela muda a direção da trajetória em 60o para a esquerda ou direita e anda 1cm nessa direção. É possível que ela retorne ao ponto de onde partiu em (a) 2008 passos? (b) 2009 passos? 1 cm 1 cm

60° 60° 1 cm

EUREKA! N°32, 2010

37


Sociedade Brasileira de Matemática

PROBLEMA 6

Seja ABC um triângulo e O seu circuncentro. As retas AB e AC cortam o circuncírculo de OBC novamente em B1 ≠ B e C1 ≠ C , respectivamente, as retas BA e BC cortam o circuncírculo de OAC em A2 ≠ A e C 2 ≠ C , respectivamente, e as retas CA e CB cortam o circuncírculo de OAB em A3 ≠ A e B3 ≠ B , respectivamente. Prove que as retas A2A3, B1B3 e C1C2 passam por um mesmo ponto. TERCEIRA FASE – NÍVEL 3 PRIMEIRO DIA PROBLEMA 1

Esmeralda escreve 20092 números inteiros em uma tabela com 2009 linhas e 2009 colunas, colocando um número em cada casa da tabela. Ela soma corretamente os números em cada linha e em cada coluna, obtendo 4018 resultados. Ela percebeu que os resultados são todos distintos. É possível que esses resultados sejam todos quadrados perfeitos? PROBLEMA 2

Considere um primo q da forma 2p + 1, sendo p > 0 um primo. Prove que existe um múltiplo de q cuja soma dos algarismos na base decimal é menor ou igual a 3. PROBLEMA 3

São colocadas 2009 pedras em alguns pontos (x, y) de coordenadas inteiras do plano cartesiano. Uma operação consiste em escolher um ponto (a, b) que tenha quatro ou mais pedras, retirar quatro pedras de (a, b) e colocar uma pedra em cada um dos pontos (a, b – 1), (a, b + 1), (a – 1, b), (a + 1, b). Mostre que, após um número finito de operações, cada ponto terá no máximo três pedras. Além disso, prove que a configuração final não depende da ordem das operações.

EUREKA! N°32, 2010

38


Sociedade Brasileira de Matemática

SEGUNDO DIA PROBLEMA 4

Mostre que existe um inteiro positivo n0 com a seguinte propriedade: para qualquer inteiro n ≥ n0 é possível particionar um cubo em n cubos menores. PROBLEMA 5

Seja ABC um triângulo e O seu circuncentro. As retas AB e AC cortam o circuncírculo de OBC novamente em B1 ≠ B e C1 ≠ C , respectivamente, as retas BA e BC cortam o circuncírculo de OAC em A2 ≠ A e C 2 ≠ C , respectivamente, e as retas CA e CB cortam o circuncírculo de OAB em A3 ≠ A e B3 ≠ B , respectivamente. Prove que as retas A2A3, B1B3 e C1C2 passam por um mesmo ponto. PROBLEMA 6

Seja n > 3 um inteiro fixado e x1, x2, …, xn reais positivos. Encontre, em função de n, todos os possíveis valores reais de

x3 x1 x2 + + + x n + x1 + x 2 x1 + x 2 + x3 x 2 + x3 + x 4

+

xn−2

x n −1 xn + + x n −1 + x n x n −1 + x n + x1

SOLUÇÕES DA TERCEIRA FASE – NÍVEL 1 PROBLEMA 1: SOLUÇÃO DE PEDRO HENRIQUE ALENCAR COSTA (FORTALEZA – CE)

(a) Um número divisível por 3 tem a soma de seus algarismos como múltiplo de 3. Assim, o primeiro termo múltiplo de 3 é 1221, pois 1 + 2 + 2 + 1 = 6, que é múltiplo de 3. O próximo é o mesmo com 3 algarismos 2 a mais. Então, para saber quantos n−2 múltiplos de 3 escritos dessa forma existem até n, fazemos: + 1. Sendo n= 3 2009 − 2 2007 2009, fica: +1 = + 1 = 669 + 1 = 670. 3 3 (b) Vejamos inicialmente um exemplo de como multiplicar por 1001. Temos 1001 vezes 80 = 80080, pois:

EUREKA! N°32, 2010

39


Sociedade Brasileira de Matemática

0080 0080 + 0080080 7 a lg arismos

O primeiro termo da sequencia que é múltiplo de 1001 possui 7 algarismos, sendo ele desta forma 1222221, que é igual a 1221 × 1001, pois:

1221 +

1221 1222221

É fácil verificar que os termos anteriores não são múltiplos de 1001. PROBLEMA 2: SOLUÇÃO DE ANA BEATRIZ MOTTA ARAGÃO CORTEZ (CAMPINAS – SP) a)

A

B G

F

E

C

D

Seja G o centro do hexágono. A área GDE e GCD é igual a de AFE. Tomando a figura como desenho representativo, podemos dividir o hexágono em seis figuras de áreas iguais: AFE; AGE; GDE; GCD; AGC; ABC. Sabendo que sua área é de 12 cm2, dividimos-na por 6 (número de partes em que o hexágono foi fracionado; assim, cada fração tem 2 cm2 de área (12 cm2 : 6). Para calcularmos a área dos triângulos pedidos, é só fazer: AFE → 2cm 2 AED → AGE+ EGD → 2cm 2 + 2cm 2 → 4cm 2 ADC → AGC+ GCD → 2cm 2 + 2cm 2 → 4cm 2 ABC → 2cm 2

EUREKA! N°32, 2010

40


Sociedade Brasileira de Matemática

Temos então a área dos dois triângulos iguais AFE e ABC como 2cm 2 (cada um) e a área dos outros dois triângulos iguais AED e ADC como 4cm 2 (cada um), totalizando 12cm 2 . Obs. Há outras formas de resolver o problema com este mesmo raciocínio. Poderíamos dividi-lo em 3 losangos, ou 12 pequenos triângulos por exemplo. b) Dividimos a figura, com um raciocínio parecido com o da letra a). P W

U Q

S

V T R

Cada triângulo acima possui a mesma área. Utilizando a informação de que o triângulo em questão (SVR ou PQU) possui área de 2cm2, calculamos a área do quadrilátero multiplicando 2cm 2 pelo número em que foi fracionada a figura, o que dá 2cm 2 ⋅ 8 = 16cm 2 , que é a área do retângulo PQRS. PROBLEMA 3: SOLUÇÃO DE DIMAS MACEDO DE ALBUQUERQUE (FORTALEZA – CE)

a) Veja os quadrados mágicos:

a1

a2

a3

a4

a5

a6

a7

a8

a9

a10

a11

a12

a13

a14

a15

a16

14

11

5

X

8

=

12

3 Y

Vendo-os, posso afirmar que a soma total do quadrado é a1 + a2 + ⋅⋅ ⋅ + a16 o que

equivale a 1 + 2 + ⋅⋅⋅ + 16 que é igual a (16 ⋅ 17 ) ÷ 2 = 136. Sabendo que em cada EUREKA! N°32, 2010

41


Sociedade Brasileira de Matemática

linha a soma é a mesma, a soma de cada uma delas será 136 ÷ 4 = 34 . Como em cada linha, coluna e diagonal a soma será 34 os valores de X e Y serão: X = 34 − (14 + 11 + 5 ) = 34 − 30 = 4 Y = 34 − (14 + 8 + 3) = 34 − 25 = 9.

b) Vamos denominar os espaços vazios do quadrado de: b1 ,b2 , b3 ,b4 ,b5 ,b6 , b7 e b8 como mostra a figura:

D1 C1

C2

C3

C4

L1

14

11

5

4

L2

b1

8

b2

b3

L3

12

b4

3

b5

L4

b6

b7

b8

9

D2 Sabendo que em cada linha, coluna ou diagonal a soma é 34, temos as seguintes equações: b3 +b5 = 22 (as raízes só podem ser 15 e 6, pois alguns dos números dos outros pares já aparecem). b2 +b8 = 26 (as raízes só podem ser 16 e 10, pois alguns dos números dos outros pares já aparecem). b4 +b7 = 15 (as raízes só podem ser 13 e 2, pois alguns dos números dos outros pares já aparecem). b1+b6 = 8 (as raízes só podem ser 7 e 1, pois se fossem 6 e 2 não daria certo, pois o 2 já aparece em b4 ou b7 ). Sendo assim, na linha 3 a única combinação qua dá certo é b4 = 13 e b5 = 6, caso fossem valores diferentes a soma da linha não daria 34. Tendo descoberto esses dois valores eu posso descobrir os outros: Se b3 não é 6, só pode ser 15. Se b7 não é 13, só pode ser 2. Na linha 2 a única combinação que dá certo é b1 = 1 e b2 = 10, pois caso fossem outros valores a soma não daria 34. EUREKA! N°32, 2010

42


Sociedade Brasileira de Matemática

Tendo descoberto esses outros dois valores posso descobrir mais outros: Se b1 = 1 , b6 só pode ser 7. Logo se b6 é 7 e b7 é 2, b8 só pode ser 16. Sabendo todos os valores desconhecidos, o quadrado mágico completo é assim: 14

11

5

4

1

8

10

15

12

13

3

6

7

2

16

9

PROBLEMA 4: SOLUÇÃO DE ISABELLA AYRES PINHEIRO DE LIMA (GOIÂNIA – GO)

a)

P =14

b) Primeiro, vamos utilizar figuras de perímetro 12, nas ‘pontas’ da figura:

Esses dois lados estarão no meio da figura, e por isso, não serão contados, ou seja, o perímetro que essa figura vai ocupar na “grande” figura será de apenas 10. Como são duas desses figuras (nas ‘pontas”), já conseguimos 20 de perímetro dos 2010 que precisamos. Agora colocamos figuras de perímetro 14 no “meio”

⋅⋅⋅ Como 4 lados de cada figura estarão no meio da grande figura, cada uma delas ocupará 10, no perímetro 2010. Teremos que usar 199 destas figuras de perímetro 14, no meio; e 2 figuras de perímetro 12, nas pontas. Ao todo: 2 × 10 + 199 × 10 = 2010 . EUREKA! N°32, 2010

43


Sociedade Brasileira de Matemática

c) Não é possível formar uma figura de perímetro ímpar, porque uma simples peça tem perímetro par e, toda vez que adicionamos outra peça, o perímetro aumentou em 10 – 2. (número de lados usados na colagem), que é sempre par. PROBLEMA 5

Veja a solução do problema 1 do nível 2. SOLUÇÕES DA TERCEIRA FASE – NÍVEL 2 PROBLEMA 1: SOLUÇÃO DE VINÍCIUS CANTO COSTA (SALVADOR – BA)

a) Supondo o contrário, isto é, que seja possível um conjunto precioso com 2 peças duplas, elas estariam intercaladas por uma peça, pois caso contrário, elas se encaixariam e isto não é possível pois não tem números em comum e isto não seria de acordo com a regra. Assim, as peças estariam arrumadas dessa forma:

?

X X ?

? Y

?

Y

e as outras duas como iriam se encaixar com as peças duplas de X e Y, seriam da forma X Y

Mas isto é um absurdo, pois não existem peças iguais no jogo (c.q.d). b) Se formarmos um conjunto precioso com uma peça dupla, ele seria organizado dessa forma, seguindo as regras do jogo:

EUREKA! N°32, 2010

44


Sociedade Brasileira de Matemática

X

X

X X

Y Y

Z

Z

Logo, se nós escolhermos as peças duplas e as que não têm contato com ela, nós formamos o conjunto: Apenas pegamos a peça com o número da peça dupla e um dos números da que não é dupla e a outra com o número da peça dupla e o outro número da que não é dupla e organizamos da maneira certa, que é única, como 6 podemos observar. Logo, a quantidade será 7 ⋅   = 105, pois são 7 peças duplas  2 e a outra peça deve ter números diferentes entre si e da peça dupla também, logo, são 2 números para escolher em 6, já que uma não pode ser usada. c) A quantidade total de conjuntos preciosos será a quantidade que inclui uma peça dupla mais a que não tem esse tipo de peça. Já temos pelo item b) que com peça dupla é 105. Basta contar os conjuntos sem peça dupla. Esses conjuntos serão da forma

X

X

W W

Y Y

Z

Z

com todos os números diferentes dois a dois. Repare que para cada conjunto de 4 números de 0 a 6 temos 3 conjuntos preciosos que seriam:

X

X

W

1º. Y

W

Z

Y

Z

,

X

X

W

2º. Z

W

Y

EUREKA! N°32, 2010

45

Z

Y

,

X

X

Y

3º. Z

Y

W

Z

W


Sociedade Brasileira de Matemática

7 Logo os conjuntos preciosos sem peça dupla totalizam   ⋅ 3, que são as maneiras  4 de escolher 4 números dentre 7 vezes 3. Assim, 7 7 ⋅ 6 ⋅ 5 ⋅ 4 ⋅ 3 ⋅ 2 ⋅1 é a 105 +   ⋅ 3 = 105 + ⋅ 3 = 105 + 7 ⋅ 5 ⋅ 3 = 105 + 105 = 210 4 ⋅ 3 ⋅ 2 ⋅1 ⋅ 3 ⋅ 2 ⋅1  4 quantidade total de conjuntos preciosos.

PROBLEMA 2: SOLUÇÃO DE FRANCISCO MARKAN NOBRE DE SOUZA FILHO (FORTALEZA – CE)

X

X1 A

X

α

β

90° – β 180° – 2α

P

α

Y β C

B

Se P é circuncentro de ABC , então ele deve ser a interseção entre as mediatrizes dos segmentos AB e AC. Como AB e AC são tangentes às circunferências,

B AY = C AX = 90°. Esses dois ângulos têm B AC

B AX = C AY , que chamarei de α : B AX = C AY = α B AC = 90° − α X AY = 90° + α

EUREKA! N°32, 2010

46

em comum e portanto


Sociedade Brasileira de Matemática

Da última igualdade, como PXAY é paralelogramo, temos P X A = 90° − α . Por outro lado, como o triângulo BXA é isósceles ( XA = XB ) , temos

AX B = 180° − 2α , ou seja, PX é bissetriz do ângulo AX B. Usando mais uma vez que BXA é isósceles, PX também é a mediana e altura relativa ao lado AB. Assim, PX é a mediatriz do segmento AB. Pela mesma rzão, PY é a mediatriz do segmento AC, o que conclui a prova. PROBLEMA 3: SOLUÇÃO DE LUCAS CAWAI JULIÃO PEREIRA (CAUCAIA – CE)

Para provarmos o que o enunciado quer, basta analisar a equação módulo 7. Queremos descobrir, então, quais os restos que um cubo qualquer i3 deixa na divisão por 7. Conseguimos isso elevando ao cubo os possíveis restos que um número qualquer deixa por 7, que são 0, 1, 2, 3, 4, 5, e 6. Concluímos que os possíveis restos que um cubo pode deixar são 0, 1 e 6. Agora analisemos as potências de 2 módulo 7. 21 ≡ 2 ( mod 7 ) 22 ≡ 4 ( mod 7 ) 23 ≡ 1( mod 7 ) 24 ≡ 2 ( mod 7 )

Encontramos o período, então dividimos 2009 por 3. Como o resto dessa divisão é 2, logo 22009 ≡ 4 ( mod 7 ) . Daí encontramos um absurdo já que qualquer soma dos possíveis restos de dois cubos jamais será 4. Logo x 3 + y 3 = 22009 não possui solução nos inteiros. PROBLEMA 4: SOLUÇÃO DE ANDRE MACIEIRA BRAGA COSTA (BELO HORIZONTE – MG)

Olhemos para a primeira equação: 1 1 1 x + = y + → Vamos substituir o termo em termos das variáveis x e y. y z z Da segunda equação, temos: xyz = 1 → xy =

x+

1 (substituímos na primeira equação) z

1 = y + xy (multiplicamos tudo por y) y

xy + 1 = y 2 + xy 2 (reordenando)

y 2 ( x + 1) − xy − 1 = 0 (resolvemos pela forma de Bháskara) EUREKA! N°32, 2010

47


Sociedade Brasileira de Matemática

1 . x +1 Separemos em dois casos: 1) Caso y = 1.

y1 = 1 e y2 = −

Substituímos na segunda igualdade: 1 1 1 1 y + = z + →1+ = z + z x z x  xyz = 1  x ⋅1 ⋅ z = 1  1  x ⋅ z = 1 Como xyz = 1, temos z = e daí x  1 z =  x 1 1 + = z + z (multiplicando por z. z ≠ 0 ) z 2 z 2 − z − 1 = 0 (Resolvendo pela fórmula de Bháskara) 1 z1 = 1 e z2 = − . 2 1 Da equação z = ,x1 = 1 e x2 = −2. x 1  Nesse caso, temos as soluções (1, 1, 1) e  −2 ,1,  . 2  1 2) Caso y = − ( x ≠ −1) x +1 Substituímos na segunda equação: 1   x −  z =1  x +1 x 1 − = x +1 z 1 1 x x +1 1 1 x +1 − = −1 = − + =z+ , e ; note que, nesse caso, y = + = − z=− z x +1 x +1 x x x x a segunda igualdade também é satisfeita.

EUREKA! N°32, 2010

48


Sociedade Brasileira de Matemática

Resposta: 1 (1,1,1) ;  −2,1,−  e todas as triplas da forma 2  a ∈ − {0, −1} .

1  Obs.  −2 ,1,−  → esta solução é da forma 2 

1 a +1  ,−  a, −  , com a +1 a  

1 a +1  ,−  a, − . a + a  1 

PROBLEMA 5: SOLUÇÃO DE DANIEL EITI NISHIDA KAWAI (SÃO PAULO – SP)

120° 120°

120°

°

60 60°

120°

120°

Temos o diagrama infinito de possíveis posições em que a formiga pode chegar.

a) Resposta: Sim. Para voltar à posição inicial em 2008 passos, basta seguir as instruções abaixo:

EUREKA! N°32, 2010

49


Sociedade Brasileira de Matemática

Posição inicial

Posição inicial

Dê 333 voltas no hexágono (isso dará 1998 passos e depois siga o trajeto abaixo, em que são usados 10 passos e volta-se à posição inicial. No total, s formiga dará 2008 passos e voltará à posição inicial b) Resposta: Não. Pinte as posições da figura inicial de preto e branco alternadamente. A formiga começa em uma bolinha preta e toda bolinha preta está cercada de bolinhas brancas e toda bolinha branca está cercada de bolinhas pretas. Assim, quando a formiga anda um númro par de passos, ela sempre termina em uma bolinha preta e quando anda um número ímpar de passos, ela sempre terminará em uma bolinha branca. Como 2009 é ímpar, a formiga, se começar em uma bolinha preta, sempre terminará em uma bolinha branca; logo, será impossível voltar à posição inicial depois de 2009 passos. PROBLEMA 6: SOLUÇÃO DE DANIEL EITI NISHIDA KAWAI (SÃO PAULO – SP)

B O1 O A

X M

EUREKA! N°32, 2010

50

C1

C


Sociedade Brasileira de Matemática

Temos AO ≅ BO são ângulos

CO e BO1 ≅ CO1 inscritos do

C1O1 ≅ OO1 . Além disso, C1 BO e C1 CO mesmo arco de circunferência

C1 MO . Como ∆ACO é isósceles (já que AO ≅ CO ), 2 Como ∆ABO é isósceles, C1 AO = C1 CO ⇒ C1 AO = C1 BO. C1 MO ⇒ C1 BO = C1 CO =

B AO = ABO ⇒ B AO + C1 AO = ABO + C1 BO ⇒ B AC1 = ABC1 ⇒ ∆ABC1 é isósceles → C1 ∈ mAB . De maneira análoga, A2 ∈ mBC , A3 ∈ mBC , B1 ∈ mAC , B3 ∈ mAC e C2 ∈ mAB ⇒ A2 A3 ⊂ mBC , B1 B3 ⊂ mAC e C1C2 ⊂ mAB . Como mAB ,mAC e mBC se encontram em 0, as retas A2 A3 ,B1 B3 e C1C2 passam por um mesmo ponto. Obs. mXY é mediatriz do segmento XY . SOLUÇÕES DA TERCEIRA FASE – NÍVEL 3 PROBLEMA 1: SOLUÇÃO DE HUGO FONSECA ARAÚJO (RIO DE JANEIRO – RJ)

Sim, é possível. Considerando a tabela como uma matriz aij tome aij = 0 , para

1 ≤ i, j ≤ 2008, e ai ,2009 = ( 2i − 1) ,a2009 ,i = ( 2i ) , para 1 ≤ i ≤ 2008. 2

2

12 32

22 42

...

d

Então já temos 4016 fileiras cujas somas são quadrados perfeitos e distintos. As duas que faltam são a última linha e última coluna. Seja a2009 ,2009 = d . Queremos que 12 + 32 + ... + 40152 + d = b 2 , 22 + 42 + ... + 40162 + d = c 2 onde b, c são distintos e maiores que 4016. Subtraindo as equações, temos: EUREKA! N°32, 2010

51


Sociedade Brasileira de Matemática

c 2 − b 2 = 1 + 2 + ... + 4016 = 2008 ⋅ 4017 ⇒ ( c − b )( c + b ) = 2008 ⋅ 4017. Tomando c = 502 ⋅ 4017 + 2 e b = 502 ⋅ 4017 − 2 , a igualdade acima é satisfeita. Para concluir, tome

d = b 2 − (12 + 32 + ... + 40152 ) = ( 502 ⋅ 4017 − 2 ) − (12 + 32 + ... + 40152 ) . 2

Desse modo, 12 + 32 + ... + 40152 + d = ( 502 ⋅ 4017 − 2 )

2

22 + 42 + ... + 40162 + d = ( 502 ⋅ 4017 + 2 ) . 2

PROBLEMA 2: SOLUÇÃO DE MATHEUS SECCO TORRES DA SILVA (RIO DE JANEIRO – RJ)

Vamos organizar as idéias. Se p = 2,q = 5 e 10 satisfaz. Se p > 2, o múltiplo de q só poderá ter soma 2 ou 3, pois se tivesse soma 1, seria uma potência de 10, e como q é primo > 5, q não divide 10n ,n ∈ Então, devemos conseguir um múltiplo com soma 2 ou 3. • Múltiplos com soma 2: 10a + 1 • Múltiplos com soma 3: 10a + 10b + 1 ( a ≥ b ) . Pelo Pequeno Teorema de Fermat, 10q −1 ≡ 1( q ) ⇒ 10 q −1 2

Se 10 ≡ −1( q ) ,10 dígitos 2).

q −1 2

Suponha então que 10

q −1 2

+

.

≡ 1( q ) ou 10

q −1 2

≡ −1( q )

+ 1 satisfaz as condições do problema (tem soma dos

q −1 2

≡ 1( q ) ⇒ 10 p ≡ 1( q ) ⇒ ord q 10 = 1 ou p.

Se ord q 10 = 1,q ≠ 9 ⇒ q = 3 ⇒ p = 1, absurdo. Logo, ord q 10 = p. Nesse caso, vamos tentar um múltiplo com soma 3, isto é, vamos procurar inteiros positivos a e b tais que 10a + 10b + 1 ≡ 0 (q). 101 ,102 ,...,10 p são p resíduos distintos módulo q = 2 p + 1. De fato, se

10 x ≡ 10 y ( q ) com y < x ≤ p ,10 ( x − y ) ≡ 1 ( q ) , 0 < x − y < p , contradição,

pois

ord q 10 = p . Se ∃ x tal que 10 x ≡ p ( q ) , tomemos a = b = x ⇒ 2 ⋅ 10 x + 1 ≡ 0 ( q ) e o problema acaba. Suponha então que ∃ x tal que 10 x ≡ p ( q ) . Temos então p resíduos para 101 ,102 ,...,10 p dentre 0 ,1, 2 ,3,..., p − 1, p + 1,..., 2 p. Vamos considerar a lista formada por esses p resíduos. EUREKA! N°32, 2010

52


Sociedade Brasileira de Matemática

0 não está na lista, pois q = 2 p + 1 > 5. Se ∃ y tal que 10 y ≡ −1( q ) , teríamos 102 y ≡ 1( q ) e

ord q 10 = p ⇒ p 2 y ⇒ 2 y = kp ⇒ 2 kp ⇒ 2 k ⇒ k = 2k0 ⇒ y = k0 p ⇒ 10 y = 10k0 p ≡ (10 p ) ≡ 1( q ) , absurdo, pois estamos supondo 10 y ≡ −1( q ) . k0

Logo, 0 e 2p não entram na lista! Considere os pares (1, 2 p − 1) ; ( 2 , 2 p − 2 ) ; ( 3, 2 p − 3) ;...( p − 1, p + 1) . Eles incluem todos os resíduos que 101 ,102 ,...,10 p podem assumir. Temos p – 1 pares e p resíduos a escolher. Pelo Princípio da Casa dos Pombos, escolheremos dois números do mesmo par. Mas a soma de dois números do mesmo par é 2p(mod q). Logo, ∃x, y com 10 x + 10 y ≡ 2 p ( q ) ⇒ A = 10 x + 10 y + 1 ≡ 0 ( q ) ⇒ A é múltiplo de q e tem soma dos dígitos 3. PROBLEMA 3: SOLUÇÃO DE RENAN HENRIQUE FINDER (JOINVILLE – SC) Vamos chamar “operação” o ato de tirar 4 pedras de (a, b) e colocar uma pedra em cada um dos pontos ( a + 1,b ) ,( a − 1,b ) ,( a,b − 1) e ( a,b + 1) . Não faremos distinção de operações no mesmo ponto que usam pedras diferentes; assim, atentaremos para quantas pedras há em cada ponto, e não quais. Provemos por indução o seguinte resultado, trivial para n ≤ 4. Para qualquer n, existe A ( n ) tal que, para quaisquer pedras p1 , p2 ,..., pn , não é

possível realizar mais de A ( n ) operações. Suponhamos que isso valha para todo k < n para fazer o caso em que temos n pedras. É importante observar que A ( n ) depende apenas de n e não da distribuição das pedras. Claramente o centro de massa das pedras é invariante. Logo, podemos fixá-lo como origem (desconsiderando a hipótese de as pedras terem coordenadas inteiras). Dada uma sequência de operações, chamaremos pi ( t ) a posição de pi após t operações , de modo que pi ( 0 ) é a posição inicial de pi . Note que ∀x ∈ ,( x − 1) + ( x + 1) = 2 x 2 + 2 2

2

2 2 2 2 ⇒∀a,b ∈ ,( a + 1) + b2  + ( a −1) + b2  + a2 + ( b −1)  + a2 + ( b + 1)  = 4a2 + 4b2 + 4         Assim, se uma operação em (a, b) move para p1 , p2 , p3 , p4 , com

p1 ( t + 1) = ( a + 1,b) , p2 ( t + 1) = ( a − 1,b) , p3 ( t + 1) = ( a,b + 1) e p4 ( t + 1) = ( a,b − 1) ,

EUREKA! N°32, 2010

53


Sociedade Brasileira de Matemática n

n

⇒ ∑ pi ( t ) = ∑ pi ( t −1) + 4. i =1

2

2

i =1

n

Por indução, ∀t, ∑ pi ( t ) ≥ 4t ⇒∃i : pi ( t ) ≥ 2

2

i =1

4t t ⇒ pi ( t ) ≥ 2 n n

Vamos escolher t grande (veremos que t ≥ 9n3 A ( n − 1) basta). 2

 t  Definimos p = pi ( t ) . Da invariância do centro de massa, ∃j , p j ( t ) ∉ C  p, ,  n  

onde C ( p,r ) = { x ∈

2

: x − p ≤ r} é o círculo de centro p e raio r. Isso ocorre

 t  porque, se todas as pedras estivessem em C  p,  , que é convexo, seu centro de  n    t  t massa também estaria, o que significa ( 0 ,0 ) ∈ C  p, , absurdo.  ⇔ p ≤ n n  Agora vemos as regiões R1 = C ( p,3 A ( n − 1) ) R2 = C ( p,6 A ( n − 1) ) R3 = C ( p,9 A ( n − 1) ) Rn = C ( p,3nA ( n − 1) ) . Uma das n + 1 regiões R1 ,R2 \ R1 ,R3 \ R2 ,...Rn \ Rn −1 e pedra. Como t ≥ 9n3 A ( n − 1) ⇔ 2

2

\ Rn não contém nenhuma

 t t  ≥ 3nA ( n − 1) , teremos p j ∉ C  p,  e n n  

 t  2 Rn ⊂ C  p,  ⇒ p j ∉ Rn ⇒ \ Rn não está vazia. R1 não está vazia porque n   p ∈ R1 . Logo, ∃k ,1 ≤ k ≤ n tal que Rk \ Rk −1 não tem pedras. Assim temos até n – 1 pedras em Rk −1 e até n – 1 pedras em

2

− Rk .

A distância entre uma pedra de Rk −1 e uma fora de Rk é sempre pelo menos

3 A ( n − 1) (vide definição). As pedras em Rk −1 e as fora de Rk se moverão independentemente até que duas delas ocupem a mesma posição. Para que isso EUREKA! N°32, 2010

54


Sociedade Brasileira de Matemática

ocorra, pela hipótese de indução, as pedras fora de Rk não realizarão mais de

A ( n − 1) movimentos, bem como as de dentro de Rk −1 . Portanto, depois de

2 A ( n − 1) rodadas, cada pedra se deslocará no máximo A ( n − 1) unidades, logo uma pedra fora de Rk não poderá ficar no mesmo ponto que uma pedra que estava dentro de Rk −1 , o que torna os dois conjuntos necessariamente independentes. Assim, basta tomar A ( n ) = 9n3 A ( n − 1) + 2 A ( n − 1) . 2

Isso resolve a primeira parte. Para a segunda parte, comecemos lembrando que, se chegarmos à mesma configuração de duas maneiras diferentes, a igualdade n

∑ p (t ) i =1

i

2

n

= 4t + ∑ pi ( 0)

2

i =1

diz que o número de operações, t, é igual nas duas maneiras. Para a prova, suponhamos que na configuração inicial, os pontos com 4 pedras ou mais sejam X1 ,X 2 ,... e X e . Considere também uma sequência de operações que leva o plano a um estado em que não é possível fazer mais operações. Certamente, ocorreram operações com centro em X1 ,X 2 ,... e X e . Considerando duas sequências de operações O1 ,O2 ,...,Oα e O1´ ,O´2 ,...,O´β que terminam em uma configuração na qual não é possível fazer mais operações, provaremos que uma é permutação da outra via indução em min{α,β} , o que resolve o problema. Seja X um ponto em que há mais de quatro pedras no princípio. Seja Oγ a primeira operação em {O1 ,O2 ,...,Oα } com centro em X. Vamos provar que a sequência de operações Oγ ,O1 ,O2 ,...,Oγ−1 ,Oγ+1 ,...Oα leva ao mesmo resultado que O1 ,O2 ,...,Oα . Basta provar que Oγ ,O1 ,...,Oγ−1 ,Oγ+1 ,...Oα é uma sequência de operações válidas, já que cada operação tira o mesmo número de pedras de cada ponto e coloca o mesmo número em cada ponto, independentemente de quando foi realizada, de forma que as operações são comutativas. Pelo mesmo argumento, basta ver que Oγ ,O1 ,...,Oγ−1 é uma sequência possível. Mas começar com Oγ é claramente possível e, da minimalidade de γ , as operações

O1 ,... e Oγ−1 têm centro em um ponto diferente de X. Assim, Oγ só pode ter EUREKA! N°32, 2010

55


Sociedade Brasileira de Matemática

aumentado o número de pedras nos centros de O1 ,... e Oγ−1 , e não diminuído, o que faz com que toda essa sequência seja possível. Em outras palavras, sem perda de generalidade, O1 = Oγ ⇒ O1 tem centro em X. Analogamente, podemos supor que, O1' tem centro em X. Agora, após a realização da operação com centro em X,

min{α,β} diminui, e vemos que as seqüências

´ β

O2 ,...,Oα e O2´,...,O são iguais pela hipótese de indução. PROBLEMA 4: SOLUÇÃO DE MARLEN LINCOLN DA SILVA (FORTALEZA – CE)

Seja S = {n ∈

: é possível particionar um cubo em n cubos menores}.

Lema: Se x, y ∈ S , então x + y − 1∈ S. Prova: Particione o cubo inicial em x cubos menores. Escolha um desses cubos e o particione em y cubos. Daí, o cubo inicial estará particionando em x + y – 1 cubos menores. Claramente, n3 ∈ S , para n ≥ 2 inteiro. Assim 23 ∈ S; portanto, ∀x ∈ S ,x + 8 − 1 = x + 7 ∈ S. Para termos o resultado desejado, basta provarmos que existem a1 ,a2 ,...,a7 ∈ S , tais que ai ≡ a j ( mod 7 ) , para 1 ≤ i < j ≤ 7 (basta escolhermos n0 = max {ai ,1 ≤ i ≤ 7} ). De fato, se x ≥ n0 = max {ai ,1 ≤ i ≤ 7} e x ≡ a j ( mod 7 ) ,x = a j + 7 k ∈ S. Seja S´ = { x ( mod 7 ) ,x ∈ S } . De forma análoga, se x, y ∈ S´ então

( x + y − 1) ( mod 7 ) ∈ S. Claramente 1,6 ,0 ∈ S , já que 23 ,33 e 73 ∈ S. Logo ( 6 + 6 − 1) mod 7 = 4 ∈ S´ e

( 0 + 6 − 1) mod 7 = 5 ∈ S´. Então ( 4 + 0 − 1) mod 7 = 3 ∈ S' e ( 3 + 0 − 1) mod 7 = 2 ∈ S' ⇒ S´ = {0,1,2,3,4,5,6} e o problema está acabado. PROBLEMA 5: SOLUÇÃO DE GUSTAVO LISBOA EMPINOTTI (FLORIANÓPOLIS – SC)

Considere uma inversão com respeito ao circuncírculo do ∆ABC. Como o circuncírculo do ∆AOC passa pelo centro de inversão (O), seu inverso é uma reta pelos inversos de A e C. Mas A e C pertencem ao círculo de inversão, de modo que são seus próprios inversos. Ou seja, a reta AC é o inverso do circuncínculo de ∆AOC . O inverso do ponto A2 é a interseção do inverso do circuncírculo de

∆AOC –que é AC – com o inverso de AB –que é o circuncírculo do ∆AOB – isto EUREKA! N°32, 2010

56


Sociedade Brasileira de Matemática

é, é o ponto A3 . Então A2 e A3 são inversos, logo A2 A3 passa pelo centro de inversão, O. Analogamente, B1 B3 e C1C2 passam por O, como queríamos. PROBLEMA 6: SOLUÇÃO DE RENAN HENRIQUE FINDER (JOINVILLE – SC)

Sendo rn ( a ) o resto da divisão do inteiro a por n (i.e., o único número r em ]0,n ] tal que a ≡ r mod ( n ) , definamos xa = xrn ( a ) . Claramente, a função S : (

)

* n +

dada por: n

S ( x1 ,...xn ) = ∑ i =1

n xi xi =1 satisfaz S ( x1 ,...xn ) > ∑ xi −1 + xi + xi +1 i =1 x1 + x2 + ... + xn

Além disso, f ( t ) = S (1,t,t 2 ,t 3 ,...,t n −1 ) =

( n − 2) t + 1 t n −1 + 1 + t + t n −1 t 2 + t + 1 1 + t n − 2 + t n −1

n e lim f ( t ) = 0 + ( n − 2 ) 0 + 1 = 1, de modo que t →∞ 3  n S ( x1 ,...xn ) pode assumir qualquer valor no intervalo 1,  . Por outro lado, se n é  3 n 1 t  n par, g (1,t,1,t,...,1,t ) =  +  é contínua e tal que g (1) = , enquanto 2  2t + 1 t + 2  3 n n n lim g ( t ) = . Portanto, S ( x1 ,...,xn ) assume todos os valores do intervalo  ,  . t →∞ 2 3 2 Se n for ímpar, definamos a função 2 n −1 t n−3 1 n h ( t ) = S (1,t,1,t,...,1,t,1) = + ⋅ + ⋅ . Temos h (1) = e t+2 2 t+2 2 2t + 1 3 n −1 h (t ) → quando t → ∞. Segue disso que n pode tomar qualquer valor em 2  n n − 1  3 , 2  . No caso em que n é par, não é difícil resolver o problema se notarmos   x x que < ∀x, y,z ∈ *+ . x+ y+z x+ y De fato: 2k k xi x2i −1 x2i = + ∑ ∑ x2i −1 + x2i + x2i +1 i =1 xi −1 + xi + xi +1 i =1 x2 i − 2 + x2 i −1 + x2 i k x2i −1 x2i <∑ + = k. x2i −1 + x2i i =1 x2 i −1 + x2 i É contínua e tal que f (1) =

EUREKA! N°32, 2010

57


Sociedade Brasileira de Matemática

Assim, sob a hipótese de n ser par, os valores possíveis de S ( x1 ,...xn ) são os

 n elementos de 1,  .  2  n − 1 Para o caso n ímpar, queremos mostrar que a imagem de S é 1, .  2  Adaptaremos a idéia usada anteriormente. Veja que essa idéia prova que x5 xn −1 xn x4 n−3 + + ... + + < 2 x3 + x4 + x5 x4 + x5 + x6 xn − 2 + xn −1 + xn xn −1 + xn + xn +1 n −1 , é suficiente termos Logo, para que se garanta que S ( x1 ,x2 ,...xn ) < 2 x3 x + x + x3 x1 x2 + + ≤ 1 2 x0 + x1 + x2 x1 + x2 + x3 x2 + x3 + x4 x1 + x2 + x3 as condições x0 ≥ x3 e x4 ≥ x1 implicam essa desigualdade. Se supusermos por n −1 absurdo S ( x1 ,...xn ) ≥ , concluímos que x0 ≥ x3 ⇒ x1 > x4 e x1 ≤ x4 ⇒ x0 < x3 . 2 Analogamente, supondo x0 ≤ x3 , x1 > x4 ⇒ x2 > x5 x2 > x5 ⇒ x3 > x6 x3 n − 4 > x3 n −1 ⇒ x3 n −3 > x3n O absurdo é que x0 ≥ x3 > x6 > x9 > ... > x3n = x0 . A suposição x1 ≤ x4 pode ser tratada similarmente. Obs: Outra maneira de se fazer o caso x0 ≤ x3 é definir yn = x3− n , o que dá y3 ≤ y0 . Alem disso, n −1 n −1 S ( x1 ,...,xn ) < ⇔ S ( y1 ,..., yn ) < , o que já sabemos provar. 2 2

EUREKA! N°32, 2010

58


Sociedade Brasileira de Matemática

XXXI OLIMPÍADA BRASILEIRA DE MATEMÁTICA Problemas e soluções da Primeira Fase – Nível Universitário PROBLEMA 1

(a) Encontre o valor mínimo da função f : → (aqui e = 2,71828... é a base do logaritmo natural). (b) Qual destes números é maior: e π ou πe ?

dada por f ( x ) = e( x / e ) − x

PROBLEMA 2

Seja ζ ∈

uma raiz de x 7 − 1, com ζ ≠ 1. Existe um polinômio Mônico p de grau 2

com coeficientes inteiros cujas raízes são os números

z2 = ζ + ζ + ζ . Calcule p ( 3) . 3

5

z1 = ζ + ζ 2 + ζ 4

e

6

PROBLEMA 3

A rã Dõ descansa no vértice A de um triângulo equilátero ABC. A cada minuto a rã salta do vértice em que está para um vértice adjacente, com probabilidade p de o salto ser no sentido horário e 1 – p de ser no sentido anti-horário, onde p ∈ ( 0,1) é uma constante. Seja Pn a probabilidade de, após n saltos, Dõ estar novamente no vértice A. (a) Prova que, qualquer que seja p ∈ ( 0 ,1) ,lim Pn = 1 / 3. n →∞

(b) Prove que existe p ∈ ( 0 ,1 / 100 ) tal que, para algum n ∈ ,Pn = 1 / π. PROBLEMA 4

Determine a quantidade de números inteiros positivos n menores ou iguais a 31! Tais que 3n + n é divisível por 31. PROBLEMA 5

Dados os números reais a, b, c, d, considere a matriz

a  d A= c  b

b a d c

Se f ( x ) = a + bx + cx 2 + dx3 , prove que

EUREKA! N°32, 2010

59

c b a d

d  c . d  a


Sociedade Brasileira de Matemática

det A = f (1) f ( i ) f ( −1) f ( −i ) . (Aqui i representa a unidade imaginária.) PROBLEMA 6

Considere a sequência a0 ,a1 ,a2 ,... definida por a0 = 0 ,a1 = π / 3 e, para n ≥ 1,

an +1 =

π ( a0 an + a1an −1 + a2 an − 2 + ... + an a0 )

3 ( n + 1)

.

Calcule ∞

ak

∑2 k =0

k

= a0 +

a1 a2 a3 + + + ... . 2 4 8

SOLUÇÕES NÍVEL UNIVERSITÁRIO – PRIMEIRA FASE PROBLEMA 1

1 a) A derivada da função f é f´ ( x ) = ⋅ e( x e) − 1, que se anula apenas para x = e, e sendo negativa para x < e e positiva para x > e. Assim, o valor mínimo de f é f (e) = 0. b) Pelo resultado do item anterior, como π ≠ e temos que f ( π ) > 0 , logo e( π e) > π , ou seja, e( ) > πe . π

PROBLEMA 2

Um polinômio que satisfaz 2 p ( x ) = x − ( z1 + z2 ) x + z1 z2 .

as

condições

do

enunciado

é

ζ7 − 1 = −1. ζ −1 z1 z2 = ζ 4 + ζ 6 + ζ 7 + ζ 5 + ζ 7 + ζ 8 + ζ 7 + ζ 9 + ζ10 = 3 + ζ + ζ 2 + ζ 3 + ζ 4 + ζ 5 + ζ 6 = 2. z1 + z2 = ζ + ζ 2 + ζ 3 + ζ 4 + ζ 5 + ζ 6 = −1 +

Logo p ( x ) = x 2 + x + 2 e p ( 3) = 14. PROBLEMA 3

a) Sejam A, B, e C os vértices do triângulo no sentido anti-horário. Seja Qn (resp. Rn ) a probabilidade de, após n saltos, Dõ estar no vértice B (resp. C). Temos P0 = 1,Q0 = R0 = 0 e, para todo n ≥ 0 ,

EUREKA! N°32, 2010

60


Sociedade Brasileira de Matemática

 Pn +1 = (1 − p ) Rn + pQn  (*) Qn +1 = (1 − p ) Pn + pRn   Rn +1 = (1 − p ) Qn + pPn Dado n natural, seja

Dn = max { Pn − Qn , Qn − Rn , Rn − Pn } . Vamos provar que,

para todo n, Dn +1 ≤ max { p,1 − p} . Dn . Dado n, há 6 possibilidades para a ordem dos números Pn ,Qn ,Rn . Vamos analisar o caso Pn ≤ Qn ≤ Rn (os outros 5 casos são análogos). Nesse caso, a maior distância Dn entre dois dos números Pn ,Qn e Rn é Rn − Pn . De (*), obtemos: Pn +1 − Qn +1 = (1 − p )( Rn − Pn ) + p ( Qn − Rn ) ≤ max {(1 − p )( Rn − Pn ) , p ( Rn − Qn )} ,

≤ max {1 − p, p} . Dn , pois Rn − Pn e Qn − Rn têm sinais contrários. Qn +1 − Rn +1 = (1 − p )( Pn − Qn ) + p ( Rn − Pn ) ≤ max {(1 − p )( Qn − Pn ) , p ( Rn − Pn )} ≤ max { p,1 − p} ⋅ Dn , pois Pn − Qn e Rn − Pn têm sinais contrários. Rn +1 − Pn +1 = (1 − p )( Qn − Rn ) + p ( Pn − Qn ) = (1 − p )( Rn − Qn ) + p ( Qn − Pn ) ≤

≤ max { p,1 − p} ⋅ ( Rn − Qn + Qn − Pn ) = max { p,1 − p} ⋅ ( Rn − Pn ) = max { p,1 − p} ⋅ Dn . Assim, Dn +1 = max { Pn +1 − Qn +1 , Qn +1 − Rn +1 , Rn +1 − Pn +1 } ≤ max { p,1 − p} ⋅ Dn , para todo n ≥ 0 , donde Dn ≤ ( max { p,1 − p} ) ,∀n ≥ 0. n

Como Pn + Qn + Rn = 1,∀n ∈ ,

Pn −

n P + Qn + Rn ( Pn − Qn ) + ( Pn − Rn ) 2Dn 2 1 = Pn − n = ≤ ≤ ⋅ ( max { p,1 − p} ) ,∀n ≥ 0. 3 3 3 3 3

Como 0 < max { p,1 − p} < 1, segue imediatamente que lim Pn − n →∞

1 = 0 , e que 3

1 lim Pn = . 3

n →∞

b) Para p = 0 teríamos Pn = 1 quando n é múltiplo de 3 e Pn = 0 caso contrário. Por outro lado, tomando p = 1 100 , temos lim P3 k +1 = 1 3 . Em particular, existe k →∞

r∈

1 1 1 tal que P3r +1 > , pois < . π π 3

EUREKA! N°32, 2010

61


Sociedade Brasileira de Matemática

Considerando P3r +1 = P3r +1 ( p ) como função de p, temos que P3r +1 ( p ) é um polinômio (de grau no máximo 3r + 1) em p, e portanto depende continuamente de  1  1 p. Como P3r +1 ( 0 ) = 0 e P3r +1   > , existe, pelo teorema do valor  100  π 1 1 tal que P3r +1 ( p ) = . intermediário, p com 0 < p < 100 π Solução alternativa para o item a): Podemos (Como no início da solução anterior), escrever p 1 − p   Pn   Pn +1   0       0 p  ⋅  Qn  ,∀n ≥ 0.  Qn +1  = 1 − p  R   p 1− p 0   Rn   n +1   1  2 p −1 Os autovalores dessa matriz 3 × 3 são 1 e − ± i 3 ⋅  . 2  2  As normas dos autovalores distintos de 1 são iguais a 1 − 3 p (1 − p ) < 1, donde

Pn ,Qn e Rn convergem a certos números, que denotaremos por x, y, z, respectivamente. Devemos então ter: x = (1 − p ) z + py, y = (1 − p ) x + pz,z = (1 − p ) y + px, donde x = (1 − p ) z + p ( (1 − p ) x + pz ) ⇒ (1 − p + p 2 ) x = (1 − p + p 2 ) z ⇒ x = z ⇒

⇒ y = (1 − p ) x + px = x, e logo x = y = z = 1 3 (pois x + y + z = 1). PROBLEMA 4

Pelo pequeno teorema de Fermat, 330 ≡ 1( mod 31) , e logo

( 3 ( mod 31) ) n

é

periódico com período divisor de 30. Por outro lado, obviamente ( n ( mod 31) ) é periódico com período 31.

Portanto, ( 3n + n ( mod 31) ) é periódico com período divisor de 31 ⋅ 30 = 930.

Pelo teorema chinês dos restos, para cada a com 0 ≤ a ≤ 29 e b com 0 ≤ b ≤ 30 , existe um único c ( a,b ) com 0 ≤ c ( a,b ) ≤ 929 tal que c ( a,b ) ≡ a ( mod 30 ) e

c ( a,b ) ≡ b ( mod 31) . Temos 3c ( a ,b ) + c ( a,b ) ≡ 3a + b ( mod 31) . EUREKA! N°32, 2010

62


Sociedade Brasileira de Matemática

Fixando a e fazendo b variar, 3a + b percorre todas as 31 classes (mod 31). Assim, 3m + m , 0 ≤ m ≤ 929 passa 30 vezes por cada classe (mod 31). Como 930 = 31 ⋅ 30 31 !, 3n + n ≡ 0 ( mod 31) para 31!/31 = 30! inteiros positivos menores ou iguais a 31!. PROBLEMA 5 1ª. Solução

Se x é uma raiz quarta da unidade, temos xf ( x ) = d + ax + bx 2 + cx 3 ,x 2 f ( x ) = c + dx + ax 2 + bx3 e x 3 f ( x ) = b + cx + dx 2 + ax 3 , de modo que

 1   f ( x)  1    xf x    x x ( ) A 2  =  2 = f ( x) 2  .  x   x f ( x)  x    3   3  3  x   x f ( x)  x 

Assim, o vetor (1,x,x 2 ,x3 ) é autovetor de A, com autovalor f(x), para x = 1, i, –1, – i. deduzimos que A possui 4 autovetores independentes e, portanto, det A é o produto dos respectivos autovalores, ou seja, det A = f (1) f ( i ) f ( −1) f ( −i ) . 2ª. Solução

Observamos que det A é um polinômio do quarto grau nas variáveis a, b, c, d, enquanto f(1), f(i), f ( –1), f (– i) são polinômios irredutíveis distintos do primeiro grau nessas mesmas variáveis. Podemos realizar operações lineares nas linhas de A para provar que o polinômio det A é divisível por f(1), f(i), f (–1), f (–i). Isto fica mais rápido utilizando a mesma ideia da primeira Solução: se x é raiz quarta da unidade, multiplicando a segunda coluna por x, a terceira por x2 e a quarta por x3 e somando tudo isso à primeira coluna, obtemos ( f ( x ) ,xf ( x ) ,x 2 f ( x ) ,x3 f ( x ) ) . Assim, temos det A = kf (1) f ( i ) f ( −1) f ( −i ) , onde k é uma constante a ser determinada. Fazendo a = 1 e b = c = d = 0, obtemos det A = 1 e f (1) f ( i ) f ( −1) f ( −i ) = 1, logo k = 1, como queríamos demonstrar. PROBLEMA 6

Defina ∞

f ( x ) = ∑ an x n = a0 + a1 x + a2 x 2 + ... + an x n + ... n =0

Então EUREKA! N°32, 2010

63


Sociedade Brasileira de Matemática

( f ( x ) ) = ∑  ∑ a a 

 n x n =0 l =0  = ( a0 a0 ) + ( a0 a1 + a1a0 ) x + ... 2

n

l

n −l

+ ( a0 an + a1an −1 + a2 an − 2 + ... + an a0 ) x n + ... e ∞

f´ ( x ) = ∑ ( n + 1) an +1 x n n=0

= a1 + 2a2 x + ... + ( n + 1) an +1 x n + ... 2 π f ( x ) ) coincidem, ( 3 2 π π exceto pelo coeficiente constante. Temos portanto f´ ( x ) − ( f ( x ) ) = . Logo 3 3 temos

Pela condição do enunciado, os coeficientes de f´ ( x ) e de

df π 2 df π = ( f + 1) ⇔ ∫ 2 = ∫ dx ⇔ dx 3 f +1 3 π π ( x + C ) ⇒ f ( x ) = tg  ( x + C )  3 3  Como f ( 0 ) = a0 = 0 , concluímos que C = 0, e portanto ⇔ arctg f =

ak

∑2 k =0

k

1 π 1 . = f   = tg   = 3 2 6

EUREKA! N°32, 2010

64


Sociedade Brasileira de Matemática

XXXI OLIMPÍADA BRASILEIRA DE MATEMÁTICA Problemas e soluções da Segunda Fase – Nível Universitário PRIMEIRO DIA PROBLEMA 1

Seja f :[0,1] → [0,1] crescente, derivável e inversível. 1

Se

∫ 0

1

f ( x)dx = ∫ f −1 ( x)dx , prove que existem dois reais diferentes a e b, 0

0 ≤ a < b ≤ 1 , tais que f '(a ) = f '(b) = 1 . Obs.: f −1 denota a inversa da função f . PROBLEMA 2

Seja N = {0,1,2,3,...}. Dados conjuntos A, B ⊂ N , para cada inteiro positivo n denote por r(A, B, n) o número de soluções da equação a + b = n, a ∈ A, b ∈ B . Prove que existe n0 ∈

tal que r(A, B, n + 1)> r(A, B, n) para todo n > n0 se e

somente se N \ A e N \ B são finitos. PROBLEMA 3

Dados

n, a1 , a2 ,..., an

inteiros

positivos,

definimos

q0 = 1, q1 = a1

e

qk +1 = ak +1qk + qk −1 , para 1 ≤ k ≤ n − 1 . Prove que, dado c > 1, existe K > 0 tal que, para todo M > K, existem n inteiro positivo e a1 , a2 ,..., an pertencentes a {1,2} tais que M ≤ qn < c ⋅ M . SEGUNDO DIA PROBLEMA 4

Seja H o hiperboloide de equação 3x 2 + 3y 2 − z 2 − 1 = 0 . i) Prove que todo ponto ( x, y, z ) ∈ H pertence a exatamente duas retas contidas em H. ii) Prove que todas as retas contidas em H formam o mesmo ângulo com o plano de equação z = 0, e determine esse ângulo.

EUREKA! N°32, 2010

65


Sociedade Brasileira de Matemática

PROBLEMA 5

Ache todas as funções f : Z → Z que satisfazem: i) f (f (n)) = f (n + 1), para todo n ∈ Z . ii) f(2009n + 2008) = 2009.f(n) para todo n ∈ Z . PROBLEMA 6

Para n inteiro positivo seja f(n) o número de produtos de inteiros maiores que 1 cujo resultado é no máximo n, isto é, f(n) é o número de k-uplas ( a1 , a2 ,..., ak ) onde k é algum natural, ai ≥ 2 é inteiro para todo i e a1 ⋅ a2 ⋅ ... ⋅ ak ≤ n (contando a 0-upla vazia ( ), cujo produto dos termos é 1). Assim, por exemplo, f(1) = 1, por causa da 0-upla ( ) e f(6) = 9, por causa da 0-upla ( ), das 1-uplas (2), (3), (4), (5) e (6) e das 2-uplas (2, 2), (2, 3) e (3, 2). Seja α > 1 tal que

1

∑ mα

= 2.

m =1

a) Prove que existe uma constante K > 0 tal que f ( n) ≤ K ⋅ nα para todo inteiro positivo n. b) Prove que existe uma constante c > 0 tal que f ( n) ≥ c ⋅ nα para todo inteiro positivo n. PROBLEMA 1: SOLUÇÃO DE MARLON DE OLIVEIRA GOMES (FORTALEZA – CE)

(I) Notemos primeiramente que f (0) = 0 e f (1) = 1. De fato, f é inversível e

portanto sobrejetora, logo, existem a e b ∈ [ 0,1] tais que f ( a ) = 0 e f ( b ) = 1. Se a ≠ 0, b ≠ 1,

f ( t ) < 0, ∀t ∈ [ 0, a ) f ( t ) > 1, ∀t ∈ ( b,1]

Um absurdo. (II) Afirmação: f possui um ponto fixo em (0, 1). Isto é, ∃c ∈ ( 0,1) tal que

f ( c ) = c. Prova: Suponha que seja f ( x ) > x, ∀x ∈ ( 0,1) .

(

)

Então, f −1 f ( x ) = x > f −1 ( x ) ∀x, pois f crescente ⇔ f −1 crescente. Logo, seria f ( x ) > x > f −1 ( x ) , ∀x ∈ ( 0,1) ⇒

EUREKA! N°32, 2010

66


Sociedade Brasileira de Matemática 1

1

1

0 1

0

f ( x ) dx > ∫ xdx > ∫ f −1 ( x ) dx, um absurdo, pois

∫ ∫ f ( x ) dx = ∫ 0 1 0

0

f −1 ( x ) dx.

Se supusermos f ( x ) < x, ∀x ∈ ( 0,1) temos um resultado análogo. Suponha agora que existam x1 e x2 ∈ ( 0,1) tais que f ( x1 ) > x1 e f ( x2 ) < x2 .

Sendo f diferenciável, é também contínua, donde g : [ 0,1] →

, g (t ) = f (t ) − t é

contínua. Note que g ( x1 ) > 0 e g ( x2 ) < 0, logo, pelo teorema do valor intermediário, existe c ∈ ( x1 , x2 ) tal que g ( x ) = 0 ⇒ f ( c ) = c, o que prova o resultado. (III) Pelo teorema do valor médio, existem a ∈ ( 0, c ) e b ∈ ( c,1)

 f (c) =1  f (c) − f ( 0 ) = f ´( a ) ⋅ ( c − 0 ) ⇒ f ´( a ) =  c . tais que   f (1) − f c = f ´ b ⋅ 1 − c ⇒ f ´ b = 1 − f ( c ) = 1 ( ) ( )( ) ( )  1− c PROBLEMA 2: SOLUÇÃO DE RAMON MOREIRA NUNES (FORTALEZA – CE)

Para cada n ∈ , seja q ( A, B, n ) o número de pares ( x, y ) tais que x + y = n e, além disso, x ∉ A ou y ∉ B . Assim, q ( A, B, n ) é o número de pares (x, y) com x + y = n que r ( A, B, n ) não conta. Portanto, q ( A, B, n ) = n + 1 − r ( A, B, n ) . Veja que:

r ( A, B, n + 1) > r ( A, B, n ) ⇔

⇔ r ( A, B, n + 1) ≥ r ( A, B, n ) + 1 ⇔ ⇔ n + 1 − q ( A, B, n + 1) ≥ n − q ( A, B, n ) + 1 ⇔ ⇔ q ( A, B, n + 1) ≤ q ( A, B, n )

∀n ≥ n0 ; daí, q ( A, B, n ) ≤ q ( A, B, n0 ) , ∀n ≥ n0 . Como só existem finitos n menores que n0 , isso nos diz que q ( A, B, n ) é limitada como função de n. Agora, se (x, y) é tal que x + y = n e x ∉ A, então ( x, y ) é contado por

q ( A, B, n ) . Dessa forma, q ( A, B, n ) ≥ # ({1, 2,..., n} \ A) .

EUREKA! N°32, 2010

67


Sociedade Brasileira de Matemática

Se

\ A fosse infinito, poderíamos tomar # ({1, 2,..., n} \ A ) tão grande quanto

quiséssemos, mas como q ( A, B, n ) é limitada isso não pode ocorrer, logo,

\A

é finito. Análogo \ B também é finito. Agora, suponha \ A e \ B finitos.

 \ A = {a1 , a2 ..., ak } ; a1 < ... < ak   \ B = {b1 , b2 ..., bm } ; b1 < ... < bm tome n0 = ak + bm . Se n > n0 , tem-se que (x, y) com x + y = n então x > ak ou y > bm pois x ≤ ak e y ≤ bm ⇒ n = x + y ≤ ak + bm = n0 . Absurdo.

Logo (x, y) não é contado por r ( A, B, n ) apenas quando x ∈

\ A ou y ∈

\B

(ambos não ocorrem simultaneamente); o primeiro caso ocorre n vezes e o segundo m vezes. Assim, r ( A, B, n ) = n + 1 − k − m , e logo r satisfaz a condição do enunciado. PROBLEMA 3: SOLUÇÃO DA BANCA

Vamos escolher dois inteiros positivos grandes r, s e tomar m = r + s, a j = 1 para

1 ≤ j ≤ r e a j = 2 para r + 1 ≤ j ≤ r + s = m. Seja qk = qk ( a1 , a2 ,..., ak ) , para 1 ≤ k ≤ m. Temos qk +1 = qk + qk −1 , para 1 ≤ k ≤ r − 1, e portanto q j = Fj +1 , para

0 ≤ j ≤ r , onde, para j ≥ 1 j j j 1   1 + 5   1 − 5   1 + o (1)  1 + 5  Fj =   −  =   5   2   2   5  2   

é

o

j-ésimo

termo

da

sequência

de

Fibonacci.

Assim,

j

 1+ 5  1+ 5 q j = (1 + o (1) ) c  . Por outro lado, temos  para j grande, onde c = 2 5  2  qk +1 = 2qk + qk −1 para r ≤ k ≤ m − 1, e logo qr + j = u j +1qr + u j qr −1 , onde ( u j ) j ≥0

a sequência dada por u0 = 0, u1 = 1 e uk + 2 = 2uk +1 + uk , para k ≥ 0. Como

uk =

(

1   1+ 2 2 2

) − (1 − 2 ) k

k

 = 1 + o (1) 1 + 2   2 2

obtemos

EUREKA! N°32, 2010

68

(

)

k

, para k ≥ 0,


Sociedade Brasileira de Matemática

qr + j =

1 + o (1) 2 2

((1 + 2 ) q + q ) (1 + 2 )

j

r −1

r

j  1+ 5  1 + o (1)  5 −1  = 1 + 2 +  c 1 + 2   2  2 2   2 

(

)

r

r

j  1+ 5  4 + 10 + 2 c 1 + 2  = (1 + o (1) )  , 8  2  para j e r grandes.

(

(

Como log 1 + 2

 1+ 5  log    2 

)

)

é irracional (pois não é possível termos

m

 1+ 5  1 + 2 =   , com m e n inteiros positivos), a conclusão do problema  2  segue (tirando logaritmos) do seguinte fato, que é provado a seguir:

(

)

n

Dados α , β > 0 tais que α β é irracional, ε > 0 e r > 0, existe x0 > 0 tal que, para

x ∈ , x ≥ x0 ,

qualquer

existem

inteiros

m, n ≥ r

tais

que

mα + nβ − x < ε . Para provar este fato, notemos que, se

(p

n

qn

)

n≥0

é a sequência de reduzidas da

fração contínua de α β , podemos escolher k natural com q 2 k +1 > β ε . Teremos então (ver o artigo sobre Frações Contínuas na Revista Eureka! No. 3)

−ε β < −1 q 2 k + 2 < q 2 k +1 α β − p 2 k +1 < 0 < q 2 k α β − p 2 k < 1 q 2 k +1 < ε β e portanto −ε < q 2 k +1α − p 2 k +1β < 0 < q 2 kα − p 2 k β < ε . Seja X o conjunto de todos

os

números

da

forma

aα + bβ

com

a, b ≥ 0

inteiros.

Se

y = aα + bβ ≥ q 2 k +1α + p 2 k β pertence a X, temos a ≥ q 2 k +1 ou b ≥ p 2 k , e portanto,

δ1 = p 2 k +1β − q 2 k +1α e teremos y + δ1 ∈ X ou

tomando

0 < δ1 , δ 2 < ε

δ 2 = q 2 kα − p 2 k β , temos y + δ 2 ∈ X . Usando este fato e

repetidamente, segue que, para todo z ≥ y, [ z , z + ε ) ∩ X ≠ ∅. Como, para

 q α + p2k β  k0 :=  2 k +1  , temos k0α ∈ X e k0α ≥ q 2 k +1α + p 2 k β , portanto, para α   EUREKA! N°32, 2010

69


Sociedade Brasileira de Matemática

todo z ≥ k0α , [ z , z + ε ) ∩ X ≠ ∅. Assim, tomando x0 := k0α + r (α + β ) , temos que,

x ≥ x0 ,

dado

x − r (α + β ) ≥ k0α ,

temos

e

portanto

 x − r (α + β ) , x − r (α + β ) + ε ) ∩ X ≠ ∅. Assim, existem a, b ≥ 0 inteiros aα + bβ ∈  x − r (α + β ) , x − r (α + β ) + ε ) , e portanto com x ≤ ( r + a ) α + ( r + b ) β < x + ε , o que prova o fato acima.

PROBLEMA 4: SOLUÇÃO DE GABRIEL LUIS MELLO DALALIO (S.J. DOS CAMPOS – SP)

Tomando um ponto

( x, y , z ) ∈ H ,

esse ponto como:

r : {( x + ka, y + kb, z + kc ) , ∀k ∈

podemos representar uma reta que passa por

}. Para ( a, b, c ) que façam r estar contida em

H teremos:

3 ( x + ka ) + 3 ( y + kb ) − ( z + kc ) − 1 = 0, ∀k ∈ 2

2

2

⇒ 3 x 2 + 6 xka + 3k 2 a 2 + 3 y 2 + 6 ykb + 3k 2b 2 − z 2 − 2 zkc − k 2 c 2 − 1 = 0, ∀k ∈ Como 3 x 2 + 3 y 2 − z 2 − 1 = 0, pois ( x, y, z ) ∈ H tem-se: 3 xa + 3 yb − zc = 0 (I)  2 2 2 3a + 3b − c = 0 (II) (I)

(II)

(3x, 3y, –z) vetor normal

30o

Ө

60o Cone

Plano

Como o menor ângulo que a direção (a, b, c) forma com o plano z = 0 é igual a

  c arctg   , por (II) tem-se: 2 2  a +b 

EUREKA! N°32, 2010

70


Sociedade Brasileira de Matemática

   c    = arctg  2  = arctg 3 = 60°.  c      3  Assim está provado ii), qualquer reta contida em H terá ângulo igual a 60° com o plano z = 0. A solução do sistema é a interseção do plano com o cone, que pode ser apenas a origem, o que indicaria que não haveria r possível, ou pode ser igual a duas retas, indicando duas direções possíveis para (a, b, c), ou seja, duas retas r possíveis, ou ainda uma reta apenas de interseção, que é o caso de ( 3 x,3 y , − z ) ter ângulo  c θ = arctg  2 2  a +b

( )

menor de 30° com o plano z = 0. Para que haja interseção de duas retas, o vetor normal do plano ( 3 x,3 y, − z ) deve ter um ângulo formado com o plano z = 0 menor que o ângulo do cone, que é de 30° . Esse ângulo é dado por    z  z  1   < arctg  arctg  = arctg    = 30° 2 2 2  9x + 9 y   3  3z   

(

)

(pois 9 x 2 + 9 y 2 = 3 z 2 + 1 > 3 z 2 ).

 z Então, como arctg   9x2 + 9 y2 

  < 30°, fica provado o item i).  

PROBLEMA 5: SOLUÇÃO DE JOSIAS ELIAS DOS SANTOS ROCHA (MURIBECA - SE)

( f (n) ) = f ( n + 1) e além disso a função f (n) = n + 1 satisfaz (ii) f (2009n + 2008) = 2009 f ( n ) . Suponhamos 2 2 então que f (n1 ) = f ( n2 ) com n1 < n2 ⇒ f ( n1 ) = f ( n2 ) ⇒ f ( n1 +1) = f ( n2 +1) ; indutivamente teremos f ( n1 + k ) = f ( n2 + k ) para todo k ≥ 0 , e assim f será Se f é injetiva, teremos f (n) = n + 1, pois f

periódica a partir de n1 com periodo n2 − n1. Daí f

(

∩ [ n1 , +∞ ) ) é finito, f

(

∩ [ n1 , +∞ ) ) = {a1 ,..., ak } .

{

Suponhamos sem perda de generalidade que a1 ≥ max ai : i = 1,...k

}

e que

f ( m ) = a1 com m > 0 e m > n1 (f é periódica) ⇒ f ( 2009m + 2008 ) = 2009a1 , mas 2009m + 2008 ∈

∩ [ n1 + ∞ ) ⇒ f ( 2009m + 2008 ) ∈ f

EUREKA! N°32, 2010

71

(

∩ [ n1 , +∞ ) )


Sociedade Brasileira de Matemática

⇒ 2009 a1 ≤ a1 ⇒ a1 = 0 ⇒ a1 = 0 ⇒ f Note-se

ainda

f ( −1) = 0

que

pois

⇒ 2009 f ( −1) = f ( −1) ⇒ f ( −1) = 0. periódica a partir de – 1 e

(

∩ [ n1 , +∞ ) ) = {0}. basta

Assim

n = −1

fazer

em

f ( −1) = f ( n1 ) = 0 ⇒ f

f ( n ) = 0, ∀n ≥ −1. Seja

n < −1,

f ( n ) = r ⇒ f ( 2009n + 2008) = 2009r ⇒ f ( 2009 ( n + 1) ) = f ( 2009r ) . ou

r = n +1

2009 ( n + 1) ⇔ f

(

ou

f

é

)

periódica

a

partir

(ii) é com

Assim, de

∩  2009 ( n + 1) , +∞ ) = {0}. Fazendo m1 = 2009 ( n + 1) e

mk = 2009mk −1 + 2008, teremos −1 > m1 > m2 > ..., e além disso temos

f ( mk ) = 0,∀k pois f ( m1 ) = 0 e f ( mk +1 ) = f ( 2009mk + 2008) = 2009 f ( mk ) = 0

se f ( mk ) = 0. Mas para todo n < −1 podemos achar mi e mi +1 tais que

mi +1 ≤ n < mi ⇒ f ( n ) = 0, logo f ( n ) = 0, ∀n. Assim temos apenas as seguintes soluções: (1) f ( n ) = n + 1, ∀n ∈ .

(2) f ( n ) = 0, ∀n ∈ .

 n + 1, se n ≤ −1 . 0, se n ≥ −1

(3) f ( n ) = 

PROBLEMA 6: SOLUÇÃO DA BANCA

Vamos inicialmente estender a função f para [1,+∞) definindo f ( x) = f ( x ) , ∀x ∈ [1,+∞) . Podemos agora mostrar que a função f satisfaz a recorrência seguinte:   x f ( x) = 1 + ∑ f ( ), ∀x ∈ [1, +∞) . De fato, temos o vetor ( ) (correspondente a m m=2 k = 0 ), e, se k ≥ 1 e (a1 , a2 ,..., ak ) é tal que a j é inteiro, a j ≥ 2, ∀j e

x

a1a2 ...ak ≤ x , então 2 ≤ a1 ≤  x  e, se a1 = m , a2 ...ak ≤ possíveis escolhas para (a2 ,..., ak ) . EUREKA! N°32, 2010

72

x x e há f   m m


Sociedade Brasileira de Matemática

Note também que α < 2 , pois a função ζ (α ) =

1

∑ mα

é decrescente e

m =1

1

∑m m =1

=1+

2

= 1+

1 1 1 1 1 1 1 1 1 1 1 1 + + + + + +...<1+ + + + + + +... 4 9 16 25 36 49 4 4 16 16 16 16

1 1 + +...= 2. 2 4

Vamos resolver o item a): Mostraremos que f ( x ) ≤ xα , ∀x ≥ 1, por indução em

x α  x  . Temos f ( x ) = 1 ≤ x , ∀x ∈ (1, 2] . Para x ≥ 2 temos   <  x  , ∀m ≥ 2, m e portanto, por hipótese de indução, α  x   x   1 x x f ( x ) = 1 + ∑ f   ≤ 1 + ∑   = 1 + xα  1 − ∑ α  m m=2 m=2  m   m > x  m ∞

(pois

 ∞ 1 = ∑ α  m =1 m

1

∑ mα

m=2

  

  − 1 = 1).  α

α

α

α

α

1  x   x   x   x   x  Como x ∑ α >   +  +  +  +  ≥  x +1   x + 2   x + 3   x + 4   x + 5  m >  x  m α

α

α

α

α

α

2

2

2

2

2

 2  2  2  2  2  2  2  2  2  2 ≥   +   +   +   +   >   +   +   +   +   > 1, para  3  4  5  6  7  3  4  5  6  7 todo x ≥ 2, segue que f ( x ) < xα , o que prova o resultado. Vamos agora resolver o item b). Mostraremos inicialmente que 2α > 3. De fato,

1 1 1 1 1 1 1 2 4 1 1 1 1  + α + α + ... > α + α + α + ... = α + α + α + ... =  α −1 + α −1 + α −1 + ... α 2 3 4 2 4 4 2 4 8 2 2 4 8  1 1 ⋅ α −1 2 2 = 1 , donde 2α − 2 > 1 e logo 2α > 3. Vamos mostrar que, se = α 1 1 − α −1 2 − 2 2 3k 1 k ≥ 6 e x < 2k , então f ( x ) ≥ cxα ⋅ k ≥ cxα , onde c = o que k 3 −9⋅2 20480 claramente implica o resultado. Note que essa desigualdade vale para k = 6 e x < 26 , pois nesse caso 1=

EUREKA! N°32, 2010

73


Sociedade Brasileira de Matemática

3k xα x2 212 α 81 α cx k = cx < 5⋅c ⋅ x = < < = 1 ≤ f ( x). 3 − 9 ⋅ 2k 17 4096 4096 4096 α

Vamos agora mostrar essa desigualdade por indução em k. Suponhamos que ela vale para um certo k ≥ 6.

3k +1 para todo x com 2k ≤ x < 2k +1. Para 3k +1 − 9 ⋅ 2k +1 x um tal valor de x, temos 1 ≤ < 2k para 2 ≤ m ≤  x  , e logo m α k 2k 2k x  3 3k 1   x  x  x f ( x) =1+ ∑ f   > ∑ f   ≥ c ⋅ k c ⋅ = ⋅ ⋅xα ⋅ 1− ∑ α .  k ∑ k k 3 − 9⋅ 2 m=2  m  3 − 9⋅ 2 m=2  m  m=2  m   m>2k m  1 1 1 1 1 1 1 Temos ∑ α < ∑ α = + +... + + ( k+1)α +... + +... < α α α α k k k k + 1 + 2 m>2k m m≥2k m ( 2 ) ( 2 +1) ( 2 −1) 2 ( 2 −1) Mostraremos que f ( x ) ≥ cxα ⋅

k

<

2k 2 k +1 1 1 2 2 + + ... = k (α −1) + ( k +1)(α −1) + ... <   +   kα k +1)α ( 2 2 2 2 3 3 k +1

k

k +1

+ ... k

 2α 3 2 2 2 α −1 pois 2 = >  , e, como   +   + ... = 3 ⋅   , temos  2 2 3 3 3  k  3k 3 k +1 2  α pois ⋅ ⋅ − > ⋅ ⋅ xα , 1 3 f (x) > c ⋅ k x c      k +1 k +1  3 − 9 ⋅ 2k 3 3 9 2 − ⋅     k k k  3k 3k +1  2   3 − 3⋅ 2 (de fato, essa última ⋅ − ⋅ = > 1 3      k k 3k − 9 ⋅ 2k  3k +1 − 9 ⋅ 2k +1  3   3 −9⋅2 6 ⋅ 2k 9 ⋅ 2 k +1 > 1 + k +1 , que desigualdade pode ser escrita como 1 + k 3 − 9 ⋅ 2k 3 − 9 ⋅ 2 k +1 equivale a 6 3k +1 − 9 ⋅ 2k +1 > 18 3k − 9 ⋅ 2k , que por sua vez equivale a

(

)

(

)

162 ⋅ 2k > 108 ⋅ 2k , que obviamente vale para todo k). Obs: α é aproximadamente igual a 1,7286472389.... Kálmar provou que lim x→∞

f ( x) α

x

=

−1 = 0,3181736521... αζ ´(α )

EUREKA! N°32, 2010

74


Sociedade Brasileira de Matemática

XXXI OLIMPÍADA BRASILEIRA DE MATEMÁTICA PREMIADOS Nível 1 (6º. e 7º. Anos) NOME Alexandre Mendonça Cardoso Daniel de Almeida Souza Pedro Henrique Alencar Costa Ana Beatriz Motta Aragão Cortez Cristhian Mafalda Érika Rizzo Aquino Bianca Lima Barretto Adriana de Sousa Figueiredo Ricardo Ken Wang Tsuzuki João Pedro Sedeu Godoi Leonardo Gomes Gonçalves Leonardo Gushiken Yoshitake Paulo Henrique Omena de Freitas Edgar Kenji Ishida Rodrigo Pommot Berto Kiane Sassaki Menezes Dimas Macedo de Albuquerque Mauricio Najjar da Silveira Murilo Corato Zanarella Victor Almeida Costa Elcio Koodiro Yoshida Carolina Lima Guimarães Bruno da Silveira Dias Emilly Guaris Costa Bruno Almeida Costa Gabriel Averbug Zukin Marcelo Ericsson de Carvalho Sarah Barreto Ornellas Isabella Ayres Pinheiro de Lima Shadi Bavar Viviane Silva Souza Freitas Matheus José Araújo Oliveira Beatriz Miranda Macedo Matheus Uchôa Constante Julio S. Akiyoshi Antonio Wesley de Brito Vieira Vinicius Jóras Padrão Mateus Guimarães Lima de Freitas Vitor Dias Gomes Barrios Marin Mariana Teatini Ribeiro Rodrigo Silva Ferreira Artur Souto Martins Tiago Martins Nápoli

CIDADE – ESTADO Salvador – BA Brasília – DF Fortaleza – CE Campinas – SP Leme – SP Goiânia – GO Salvador – BA Porto Alegre – RS São Paulo – SP Rio de Janeiro – RJ Brasília – DF São Paulo – SP São Paulo – SP São Paulo – SP Brasília – DF Rio de Janeiro – RJ Fortaleza – CE São Paulo – SP Amparo – SP Fortaleza – CE São Paulo – SP Vitória – ES Florianópolis – SC Maceió – AL Fortaleza – CE Rio de Janeiro – RJ São Paulo – SP Salvador – BA Goiânia – GO Blumenau – SC Salvador – BA Recife – PE Niterói – RJ Goiânia – GO São Paulo – SP Cocal de Alves – PI Rio de Janeiro – RJ Fortaleza – CE Presidente Prudente – SP Belo Horizonte – MG Salvador – BA Fortaleza – CE Itú – SP

EUREKA! N°32, 2010

75

PRÊMIO Ouro Ouro Ouro Ouro Ouro Ouro Prata Prata Prata Prata Prata Prata Prata Prata Prata Prata Prata Bronze Bronze Bronze Bronze Bronze Bronze Bronze Bronze Bronze Bronze Bronze Bronze Bronze Bronze Bronze Bronze Bronze Menção Honrosa Menção Honrosa Menção Honrosa Menção Honrosa Menção Honrosa Menção Honrosa Menção Honrosa Menção Honrosa Menção Honrosa


Sociedade Brasileira de Matemática Laís Monteiro Pinto Guilherme Anitele Silva Pedro Papa Paniago Gabriel Yudi Hirata Iago Carvalho de Moraes Adam Yuuki Oyama Luíze Mello D´urso Vianna Enrico Pascucci Loffel Daniel Charles M. Gomes Ellen Tamie Ikefuti Morishigue Ana Emília Hernandes Dib Marcelo Liu Guo Gabriel Queiroz Moura Gabriel Branco Frizzo Ana Jéssyca Mendes Belarmino Mariana Bonfim Moraes Morant de Holanda Ricardo Vidal Mota Peixoto Bruno de Marchi Andrade Juliana Amoedo Amoedo Plácido

Rio de Janeiro – RJ Presidente Prudente – SP Belo Horizonte – MG São Paulo – SP Recife – PE Curitiba – PR Rio de Janeiro – RJ S.B.do Campo – SP Mogi das Cruzes – SP Bastos – SP S.J. do Rio Preto – SP São Paulo – SP Teresina – PI Curitiba – PR Fortaleza – CE Rio de Janeiro – RJ Vassouras – RJ Valinhos – SP Salvador – BA

Menção Honrosa Menção Honrosa Menção Honrosa Menção Honrosa Menção Honrosa Menção Honrosa Menção Honrosa Menção Honrosa Menção Honrosa Menção Honrosa Menção Honrosa Menção Honrosa Menção Honrosa Menção Honrosa Menção Honrosa Menção Honrosa Menção Honrosa Menção Honrosa Menção Honrosa

Nível 2 (8º. e 9º. Anos) NOME André Macieira Braga Costa Gabriel Ilharco Magalhães Daniel Eiti Nishida Kawai Henrique Gasparini Fiuza do Nascimento Marina Pessoa Mota Fellipe Sebastiam S. P. Pereira Liara Guinsberg Lara Timbó Araújo Victor Kioshi Higa Fernando Lima Saraiva Filho Lucas Cawai Julião Pereira Mateus Henrique Ramos de Souza Henrique Vieira Gonçalves Vaz Lucas Nishida Pedro Víctor Falci de Rezende Rafael Kazuhiro Miyazaki Francisco Markan Nobre de Souza Filho Vincent Cherng Hsi Lee Vinícius Canto Costa Thiago Poeiras Silva Victor de Oliveira Bitaraes Victor Hugo Corrêa Rodrigues Luciano Drozda Dantas Martins Breno Leví Corrêa

CIDADE – ESTADO Belo Horizonte – MG Juiz de Fora – MG Atibaia – SP Brasília – DF Fortaleza – CE Rio de Janeiro – RJ São Paulo – SP Fortaleza – CE São Paulo – SP Eusébio – CE Caucaia – CE Pirapora – MG São Paulo – SP Pedreira – SP Juiz de Fora – MG São Paulo – SP Fortaleza – CE São Paulo – SP Salvador – BA Belo Horizonte – MG Betim – MG Rio de Janeiro – RJ Fortaleza – CE Campo Belo – MG

EUREKA! N°32, 2010

76

PRÊMIO Ouro Ouro Ouro Ouro Ouro Ouro Prata Prata Prata Prata Prata Prata Prata Prata Prata Prata Bronze Bronze Bronze Bronze Bronze Bronze Bronze Bronze


Sociedade Brasileira de Matemática Wilson Aparecido Sedano Filho Victor Venturi Daniel Lima Santanelli Felipe Penha Alves Lucas Grimauth Evangelista Gabriel Nogueira Coelho de Togni de Souza Ana Thais Castro de Santana Caio Cesar do Prado Dorea Reis Rafael Rodrigues Rocha de Melo Murilo Freitas Yonashiro Coelho Gabriel José Moreira da Costa Silva Nathalia Novello Fernandes Ribeiro Gabriel Pacianotto Gouveia Pedro Ivo Coêlho de Araújo Elias Brito Oliveira Fernando Tomimura Miyashiro Igor Augusto Marques do Carmo Juliane Trianon Fraga Aimê Parente de Sousa Tadeu Pires de Matos Belfort Neto Yuri Zeniti Sinzato Gabriel Sena Galvão Filipe Mourão Leite Gabriela Loiola Vilar Marcelo Cargnelutti Rossato Pedro Henrique Botolozo Maria Jair Gomes Soares Júnior Maria Clara Cardoso Júlio César Prado Soares Fábio Kenji Arai Julio Barros de Paula Francisco Matheus Gonçalves de Souza Daniel Kantorowitz Ivan Tadeu Ferreira Antunes Filho Vitor Ramos de Paula Victor Santos de Andrade

Paulínia – SP Campinas – SP Rio de Janeiro – RJ São Luís – MA São Paulo – SP Rio de Janeiro – RJ Rio de Janeiro – RJ Nova Iguaçu – RJ Caucaia – CE São Paulo – SP Maceió – AL Rio de Janeiro – RJ São Paulo – SP Fortaleza – CE Brasília – DF São Paulo – SP Juiz de Fora – MG São Paulo – SP Fortaleza – CE Fortaleza – CE Brasília – DF Brasília – DF Teresina – PI Fortaleza – CE Santa Maria – RS Curitiba – PR Montes Claros – MG São Paulo – SP Brasília – DF São Paulo – SP Taubaté – SP João Pessoa – PB Bragança Paulista – SP Lins – SP Belo Horizonte – MG Teresina – PI

Bronze Bronze Bronze Bronze Bronze Bronze Bronze Bronze Menção Honrosa Menção Honrosa Menção Honrosa Menção Honrosa Menção Honrosa Menção Honrosa Menção Honrosa Menção Honrosa Menção Honrosa Menção Honrosa Menção Honrosa Menção Honrosa Menção Honrosa Menção Honrosa Menção Honrosa Menção Honrosa Menção Honrosa Menção Honrosa Menção Honrosa Menção Honrosa Menção Honrosa Menção Honrosa Menção Honrosa Menção Honrosa Menção Honrosa Menção Honrosa Menção Honrosa Menção Honrosa

Nível 3 (Ensino Médio) NOME Renan Henrique Finder Marcelo Tadeu de Sá Oliveira Sales Davi Lopes Alves de Medeiros Matheus Secco Torres da Silva Hugo Fonseca Araújo Marco Antonio Lopes Pedroso Gustavo Lisbôa Empinotti

CIDADE – ESTADO São Paulo – SP Salvador – BA Fortaleza – CE Rio de Janeiro – RJ Rio de Janeiro – RJ Santa Isabel – SP Florianópolis – SC

EUREKA! N°32, 2010

77

PRÊMIO Ouro Ouro Ouro Ouro Ouro Ouro Prata


Sociedade Brasileira de Matemática Marlen Lincoln da Silva Deborah Barbosa Alves Illan Feiman Halpern Thiago Ribeiro Ramos Hanon Guy Lima Rossi João Lucas Camelo Sá Carlos Henrique de Andrade Silva Custódio Moreira Brasileiro Silva Rafael Alves da Ponte Matheus Barros de Paula Bruno Silva Mucciaccia Matheus Araujo Marins Guilherme da Rocha Dahrug Victorio Takahashi Chu Voltaire Laplace dos Reis Jardiel Freitas Cunha Rafael Horimoto de Freitas Lucas de Freitas Smaira Robério Soares Nunes Gabriel Militão Vinhas Lopes Alvaro Lopes Pedroso Alan Anderson da Silva Pereira Maria Clara Mendes Silva Nara Gabriela de Mesquita Peixoto Rodrigo de Sousa Serafim da Silva Rodrigo Rolim Mendes de Alencar João Mendes Vasconcelos Otávio Augusto de Oliveira Mendes Renan Roveri do Amaral Gurgel Fernando Fonseca Andrade Oliveira Caíque Porto Lira Gustavo Haddad Francisco e S. Braga Gustavo Cellet Marques Wagner Rosales Chaves Wagner Carlos Morêto Loyola Filho James Jun Hong Kayo de França Gurgel Nathana Alcântara Lima Gabriel Lima Guimarães Ivan Guilhon Mitoso Rocha Elder Massahiro Yoshida Ruan Alves Pires André Saraiva Nobre dos Santos Luiz Filipe Martins Ramos Felipe Abella C. Mendonça de Souza Eduardo Machado Capaverde Thales Sinelli Lima Tuane Viana Pinheiro Vinícius Cipriano Klein

Fortaleza – CE São Paulo – SP São Paulo – SP Varginha – MG São Paulo – SP Fortaleza – CE Fortaleza – CE Embu – SP Fortaleza – CE Taubaté – SP Vitória – ES São Gonçalo – RJ Santo André – SP São Paulo – SP Manhuaçu – MG Recife – PE São Paulo – SP Guaxupé – MG Riberio Preto – SP Fortaleza – CE Santa Isabel – SP União dos Palmares – AL Pirajuba – MG Fortaleza – CE Itatiba – SP Fortaleza – CE Fortaleza – CE Pilar do Sul – SP Jundiaí – SP Belo Horizonte – MG Fortaleza – CE S.J.dos Campos – SP São Paulo – SP Jundiaí – SP Vitória – ES São Paulo – SP Fortaleza – CE Fortaleza – CE Vitória – ES Fortaleza – CE São Paulo – SP Rio de Janeiro – RJ Fortaleza – CE Niterói – RJ João Pessoa – PB Florianópolis – SC São Paulo – SP Rio de Janeiro – RJ Viçosa – MG

EUREKA! N°32, 2010

78

Prata Prata Prata Prata Prata Prata Prata Prata Prata Bronze Bronze Bronze Bronze Bronze Bronze Bronze Bronze Bronze Bronze Bronze Bronze Bronze Bronze Bronze Bronze Menção Honrosa Menção Honrosa Menção Honrosa Menção Honrosa Menção Honrosa Menção Honrosa Menção Honrosa Menção Honrosa Menção Honrosa Menção Honrosa Menção Honrosa Menção Honrosa Menção Honrosa Menção Honrosa Menção Honrosa Menção Honrosa Menção Honrosa Menção Honrosa Menção Honrosa Menção Honrosa Menção Honrosa Menção Honrosa Menção Honrosa Menção Honrosa


Sociedade Brasileira de Matemática André Austregésilo Scussel Thiago Augusto da Silva Baleixo Marcos Massayuki Kawakami Ana Luísa de Almeida Losnak

Fortaleza – CE Rio de Janeiro – RJ São Paulo – SP São Paulo – SP

Menção Honrosa Menção Honrosa Menção Honrosa Menção Honrosa

CIDADE – ESTADO São Paulo – SP Fortaleza – CE Campinas – SP Campinas – SP São Paulo – SP Fortaleza – CE São Paulo – SP Fortaleza – CE Rio de Janeiro – RJ Belo Horizonte – MG Florianópolis – SC São Paulo – SP São Carlos – SP S.J. dos Campos – SP São Paulo – SP Fortaleza – CE Rio de Janeiro – RJ Fortaleza – CE Rio de Janeiro – RJ São Paulo – SP Fortaleza – CE Aracajú – SE Muribeca – SE S.J. dos Campos – SP Rio de Janeiro – RJ São Paulo – SP Fortaleza – CE São Paulo – SP Fortaleza – CE Rio de Janeiro – RJ S.J. dos Campos – SP S.J. dos Campos – SP Rio de Janeiro – RJ S.J. dos Campos – SP Vitória – ES São Paulo – SP Belford Roxo – RJ Fortaleza – CE S.J. dos Campos – SP

PRÊMIO Ouro Ouro Ouro Ouro Ouro Ouro Ouro Ouro Prata Prata Prata Prata Prata Prata Prata Prata Prata Prata Bronze Bronze Bronze Bronze Bronze Bronze Bronze Bronze Bronze Bronze Bronze Bronze Bronze Bronze Bronze Bronze Bronze Bronze Bronze Bronze Bronze

Nível Universitário NOME Leonardo Ribeiro de Castro Carvalho Régis Prado Barbosa Rafael Assato Ando Rafael Daigo Hirama Guilherme Rodrigues Nogueira de Souza Adenilson Arcanjo de Moura Junior Felipe Rodrigues Nogueira de Souza Ramon Moreira Nunes Thomás Yoiti Sasaki Hoshina Rafael Tupynambá Dutra Luís Fernando Schultz Xavier da Silveira Thiago Costa Leite Santos Gabriel Ponce Carlos Henrique Melo Souza Marcelo Matheus Gauy Antônio Felipe Cavalcante Carvalho Kellem Corrêa Santos Rafael Montezuma Pinheiro Cabral Ricardo Turolla Bortolotti Alexandre Hideki Deguchi Martani Rafael Sampaio de Rezende Maurício de Lemos Rodrigues Collares Neto Joas Elias dos Santos Rocha Gabriel Luís Mello Dalalio Carlos Coelho Lechner Enzo Haruo Hiraoka Moriyama Paulo Sérgio de Castro Moreira Helder Toshiro Susuki Mateus Oliveira de Figueiredo Gabriel Caser Brito Paulo André Carvalho de Melo Caio Ishizara Costa Willy George do Amaral Petrenko Sidney Cerqueira Bispo dos Santos Filho Rafael Endlich Pimentel Guilherme Philippe Figueiredo Bruno da Silva Santos Francisco Osman Pontes Neto Luty Rodrigues Ribeiro

EUREKA! N°32, 2010

79


Sociedade Brasileira de Matemática Renato Rebouças de Medeiros José Olegário de Oliveira Neto Eduardo Fischer Guilherme Lourenço Mejia Jorge Henrique Craveiro de Andrade Edson Augusto Bezerra Lopes Eric Campos Bastos Guedes Thiago da Silva Pinheiro Leandro Farias Maia Pedro Paulo Albuquerque Goes Álvaro Krüger Ramos Alysson Espíndola de Sá Silveira Alfredo Roque de Oliveira Freire Filho Rafael Ghussn Cano Antônio Deromir Neves da Silva Júnior Rafael Sabino Lima Hudson do Nascimetno Lima Diego Andrés de Barros Lima Barbosa Reinan Ribeiro Souza Santos Marcos Victor Pereira Vieira Daniel Ungaretti Borges

Fortaleza – CE S.J. dos Campos – SP Encantado – RS S.J. dos Campos – SP Rio de Janeiro – RJ Fortaleza – CE Niterói – RJ São Paulo – SP Fortaleza – CE Fortaleza – CE Porto Alegre – RS Fortaleza – CE S.J. dos Campos – SP Campinas – SP Fortaleza – CE Rio de Janeiro – RJ Fortaleza – CE Rio de Janeiro – RJ Aracaju – SE Fortaleza – CE Belo Horizonte – MG

EUREKA! N°32, 2010

80

Menção Honrosa Menção Honrosa Menção Honrosa Menção Honrosa Menção Honrosa Menção Honrosa Menção Honrosa Menção Honrosa Menção Honrosa Menção Honrosa Menção Honrosa Menção Honrosa Menção Honrosa Menção Honrosa Menção Honrosa Menção Honrosa Menção Honrosa Menção Honrosa Menção Honrosa Menção Honrosa Menção Honrosa


Sociedade Brasileira de Matemática

AGENDA OLÍMPICA XXXII OLIMPÍADA BRASILEIRA DE MATEMÁTICA NÍVEIS 1, 2 e 3 Primeira Fase – Sábado, 12 de junho de 2010 Segunda Fase – Sábado, 18 de setembro de 2010 Terceira Fase – Sábado, 16 de outubro de 2010 (níveis 1, 2 e 3) Domingo, 17 de outubro de 2010 (níveis 2 e 3 - segundo dia de prova). NÍVEL UNIVERSITÁRIO Primeira Fase – Sábado, 18 de setembro de 2010 Segunda Fase – Sábado, 16 e Domingo, 17 de outubro de 2010

ASIAN PACIFIC MATH OLYMPIAD (APMO) 06 de março de 2010

XVI OLIMPÍADA DE MAIO 08 de maio de 2010

XXI OLIMPÍADA DE MATEMÁTICA DO CONE SUL 13 a 19 de junho de 2010 Águas de São Pedro, SP – Brasil

LI OLIMPÍADA INTERNACIONAL DE MATEMÁTICA 02 a 14 de julho de 2010 Astana, Cazaquistão

XVII OLIMPÍADA INTERNACIONAL DE MATEMÁTICA UNIVERSITÁRIA 24 a 30 de julho de 2010 Blagoevgrad, Bulgária

XXIV OLIMPÍADA IBEROAMERICANA DE MATEMÁTICA 17 a 27 de setembro de 2010 Paraguai

II COMPETIÇÃO IBEROAMERICANA INTERUNIVERSITÁRIA DE MATEMÁTICA 3 a 9 de outubro de 2010 Rio de Janeiro, Brasil

XIII OLIMPÍADA IBEROAMERICANA DE MATEMÁTICA UNIVERSITÁRIA

EUREKA! N°32, 2010

81


Sociedade Brasileira de Matemática

COORDENADORES REGIONAIS Alberto Hassen Raad Américo López Gálvez Andreia Goldani Antonio Carlos Nogueira Benedito Tadeu Vasconcelos Freire Carmen Vieira Mathias Claus Haetinger Cláudio de Lima Vidal Denice Fontana Nisxota Menegais Disney Douglas Lima de Oliveira Edson Roberto Abe Edney Aparecido Santulo Jr. Élio Mega Eudes Antonio da Costa Fábio Brochero Martínez Florêncio Ferreira Guimarães Filho Francinildo Nobre Ferreira Genildo Alves Marinho Graziela de Souza Sombrio Gilson Tumelero Ivanilde Fernandes Saad João Benício de Melo Neto João Francisco Melo Libonati Jose de Arimatéia Fernandes José Luiz Rosas Pinho José Vieira Alves José William Costa Krerley Oliveira Licio Hernandes Bezerra Luciano G. Monteiro de Castro Luzinalva Miranda de Amorim Marcelo Rufino de Oliveira Marcelo Mendes Newman Simões Nivaldo Costa Muniz Nivaldo de Góes Grulha Jr. Osnel Broche Cristo Uberlândio Batista Severo Raul Cintra de Negreiros Ribeiro Ronaldo Alves Garcia Rogério da Silva Ignácio Reginaldo de Lima Pereira Reinaldo Gen Ichiro Arakaki Ricardo Amorim Sérgio Cláudio Ramos Seme Gebara Neto Tadeu Ferreira Gomes Tomás Menéndez Rodrigues Valdenberg Araújo da Silva Vânia Cristina Silva Rodrigues Wagner Pereira Lopes

(UFJF) (USP) FACOS (UFU) (UFRN) (UNIFRA) (UNIVATES) (UNESP) (UNIPAMPA) (UFAM) (Colégio Objetivo de Campinas) (UEM) (Grupo Educacional Etapa) (Univ. Federal do Tocantins) (UFMG) (UFES) (UFSJ) (Centro Educacional Leonardo Da Vinci) (UNOCHAPECÓ) (UTFPR) (UC. Dom Bosco) (UFPI) (Grupo Educacional Ideal) (UFPB) (UFSC) (UFPB) (Instituto Pueri Domus) (UFAL) (UFSC) (Sistema Elite de Ensino) (UFBA) (Grupo Educacional Ideal) (Colégio Farias Brito, Pré-vestibular) (Cursinho CLQ Objetivo) (UFMA) (USP – São Carlos) (UFLA) (UFPB)) (Colégio Anglo) (UFGO) (Col. Aplic. da UFPE) (Escola Técnica Federal de Roraima) (UNIFESP) (Centro Educacional Logos) (IM-UFRGS) (UFMG) (UEBA) (U. Federal de Rondônia) (U. Federal de Sergipe) (U. Metodista de SP) (CEFET – GO)

EUREKA! N°32, 2010

82

Juiz de Fora – MG Ribeirão Preto – SP Osório – RS Uberlândia – MG Natal – RN Santa María – RS Lajeado – RS S.J. do Rio Preto – SP Bagé – RS Manaus – AM Campinas – SP Maringá – PR São Paulo – SP Arraias – TO Belo Horizonte – MG Vitória – ES São João del Rei – MG Taguatingua – DF Chapecó – SC Pato Branco – PR Campo Grande – MS Teresina – PI Belém – PA Campina Grande – PB Florianópolis – SC Campina Grande – PB Santo André – SP Maceió – AL Florianópolis – SC Rio de Janeiro – RJ Salvador – BA Belém – PA Fortaleza – CE Piracicaba – SP São Luis – MA São Carlos – SP Lavras – MG João Pessoa – PB Atibaia – SP Goiânia – GO Recife – PE Boa Vista – RR SJ dos Campos – SP Nova Iguaçu – RJ Porto Alegre – RS Belo Horizonte – MG Juazeiro – BA Porto Velho – RO São Cristovão – SE S.B. do Campo – SP Jataí – GO


Issuu converts static files into: digital portfolios, online yearbooks, online catalogs, digital photo albums and more. Sign up and create your flipbook.